Download as docx, pdf, or txt
Download as docx, pdf, or txt
You are on page 1of 73

Accident ....................................................................

31
Table of Contents

Toledo v. People (439 SCRA 94) ............................ 31

Art.3: Felonies ................................................................. 4

People v. Concepcion (386 SCRA 74) .................... 31

Classification of Felonies According to the Means of


Commission ................................................................. 4

Irresistible Force/Uncontrollable Fear...................... 32

Calimutan v. People (G.R. No. 152133) ................... 4


Manuel v. People (G.R. No. 165842) ....................... 4
Mistake of Fact ............................................................ 5
U.S. v. Ah Chong (15 Phil. 488) ................................ 5
People v. Fernando (G.R. No. L-24978) ................... 6
Diego v. Castillo (A.M. No. RTJ-02-1673) ................ 6
Estrada v. Sandiganbayan (G.R. No. 148560).......... 6
People v. Go Shiu Ling (G.R. No. 115156) ............... 7
Art. 4: Criminal Liability ................................................... 7
Elements of Criminal Liability...................................... 7
Quinto v. Andres (453 SCRA 511).......................... 10
Intent to Kill............................................................... 16
Intod v. CA (215 SCRA 52) ..................................... 16
Valenzuela v. People (G.R. No. 160188)................ 17
People v. Campuhan (G.R. No. 129433) ................ 19
Self-defense .............................................................. 22
Urbano v. People (G.R. No. 182750) ..................... 22
People v. Sanchez (G.R. No. 161007) .................... 23
Defense of Relatives.................................................. 24
Balunueco v. CA and People (G.R. No. 126968) ... 24
Fulfilment of a Duty .................................................. 27
Mamangun v. People (GR 149152) ....................... 27

Ty v. People (G.R. No. 149275) ............................. 32


Entrapment v. Instigation ......................................... 33
People v. Sta. Maria (G.R. No. 171019) ................ 33
Chang v. People (G.R. No. 165111) ....................... 33
Art. 13: Mitigating Circumstances ................................ 34
Incomplete Justifying or ........................................... 34
Exempting Circumstances......................................... 34
People v. CA and Tangan (G.R. No. 103613) ......... 34
Vindication of a Grave offence ................................. 36
Passion or Obfuscation ............................................. 36
People v. Malejana (G.R. No. 145002) .................. 36
People v. Bates (G.R. No. 139907) ........................ 38
Confession of Guilt ................................................... 38
People v. Monttinola 360 scra 631 ...................... 38
People v. Dawaton (G.R. No. 146247) .................. 38
Similar and Analogous Circumstances ...................... 39
Canta v. People (G.R. No. 140937) ....................... 39
Art. 14: Aggravating Circumstances ............................. 39
Classes of Aggravating Circumstances ...................... 39
People v. Evina (405 SCRA 152) ............................ 39
People v. Palaganas (501 SCRA 533)..................... 40
People v. Mendoza (327 SCRA 695)...................... 40

Baxinela v. People (G.R. No. 149652).................... 28

In Contempt or With Insult to the Public Authorities


.................................................................................. 41

Obedience to an Order ............................................. 28

People v. De Mesa (G.R. No. 137036) ................... 41

Tabuena v. Sandiganbayan (G.R. Nos. 103501-03)


............................................................................... 28

People v. Tac-an (G.R. No. 76338-39) ................... 41

Insanity/Imbecility .................................................... 29
People v. Valledor (G.R. No. 129291).................... 29
Minority..................................................................... 30
Llave v. People (G.R. No. 166040) ........................ 30
Jose v. People (G.R. No. 162052) .......................... 30

Abuse of Public Position ........................................... 41


Fortuna v. People (G.R. No. 135784) .................... 41
People v. Villamor (G.R. Nos. 140407-08) ............ 42
Night time, Uninhabited or Obvious Place or Band.. 42
People v. Villanueva (G. R. No. 135330) ............... 42

People v. Ancheta (G.R. No. 70222) ...................... 43

People v. Mariano (G.R. No. L-40527) .................. 57

Recidivism ................................................................. 44

Arts. 21-24: Penalties in General .................................. 58

People v. Dacillo (G.R. No. 149368) ...................... 44

Retroactive Effect of Penal Laws .............................. 58

By Means of Inundation, fire, etc.............................. 45

People v. Evina (G.R. No. 124830-310) ................. 58

People v. Malngan (G.R. No. 170470) ................... 45

People v. Lazaro (G.R. No. 112090) ...................... 58

People v. Comadre (G.R. No. 153559) .................. 45

Pardon by Offended Party ........................................ 59

Craft, Fraud or Disguise............................................. 46

Sta. Catalina v. People (G.R. No. 167805) ............. 59

People v. Labuguen (G.R. No. 127849) ................. 46

Balderama v. People (G.R. No. 147578-85) .......... 59

Abuse of Superior Strength ....................................... 47

People v. Dimaano (G.R. No. 168168) .................. 60

People v. Calpito (416 SCRA 491) .......................... 47

Arts. 25-45: Penalties.................................................... 61

Treachery .................................................................. 47

Reclusion Perpetua ................................................... 61

People v. Piliin (515 SCRA 207) ............................. 47

People v. Novio (G.R. No. 139332) ....................... 61

Ignominy ................................................................... 48

People v. Zacarias (G.R. No. 138990) .................... 61

Aid of Minor or By Means of Motor Vehicles ........... 48

People v. Ramirez (G.R. No. 138261) .................... 62

People v. Mallari (404 SCRA 170) .......................... 48

Arts. 46-77: Application of Penalties ............................ 62

People v. Enguito (326 SCRA 508) ......................... 49

Complex Crime.......................................................... 62

Cruelty ....................................................................... 50

People v. Pineda (G.R. No. L-26222) ..................... 62

People v. Guerrero (389 SCRA 389) ...................... 50

People v. Sanidad (G.R. No. 146099) .................... 63

Simangan v. People (434 SCRA 38) ....................... 50

Delito Continuado..................................................... 63

Art. 15: Alternative Circumstances ............................... 51


Relationship .............................................................. 51

Ramiscal v. Sandiganbayan (G.R. Nos. 169727-28)


.............................................................................. 63

People v. Calongui (G.R. No. 170566) ................... 51

Santiago v. Garchitorena (G.R. No. 109266) ......... 64

People v. Marcos (G.R. No. 132392) ..................... 51

Habitual Delinquency ............................................... 65

Intoxication ............................................................... 51

People v. Espina (G.R. No. 43556) ........................ 65

Arts. 16-20: Persons Criminally Liable for Felonies....... 51

People v. De Jesus (G.R. No. 45198) ..................... 65

Principals ................................................................... 51

Arts. 89-93: Total Extinction of Criminal Liability ......... 66

People v. Vasquez (G.R. No. 123939) .................... 52

Death of the Accused................................................ 66

People v. Dacillo (G.R. No. 149368) ...................... 53

De Guzman v. People (G.R. No. 154579) .............. 66

Accomplices .............................................................. 53

People v. Bayotas (G.R. No. 102007) .................... 66

Abarquez v. People (G.R. No. 150762) .................. 53

People v. Abungan (G.R. No. 136843) .................. 67

Accessories ................................................................ 54

Prescription of Offenses ........................................... 67

People v. Tolentino (G.R. No. 139179) .................. 54

Panaguiton v. DOJ (G.R. No. 167571) ................... 67

People v. Cui (G.R. No. 121982) ............................ 55

Recebido v. People (346 SCRA 881) ...................... 68

People v. Verzola (G.R. No. L-35022) .................... 56

Amnesty .................................................................... 68

Accessories Exempt from Criminal Liability .............. 57

People v. Patriarcha (G.R. No. 135457) ................ 68

Arts. 100-103: Civil Liability........................................... 69


Subsidiary Civil Liability of Other Persons ................. 69
Nueva Espana v. People (460 SCRA 547) .............. 69
Pangonorom v. People (455 SCRA 211) ................ 70
Quinto v. Andres (453 SCRA 511).......................... 71
Probation Law (P.D. No. 968) ........................................ 72
Francisco v. CA (G.R. No. 108747) ......................... 72
Anti-Fencing Law (P.D. No. 1612) ................................. 72
Francisco v. People (434 SCRA 122) ...................... 72
Tan v. People (313 SCRA 220) ............................... 73

Art.3: Felonies
Classification of Felonies According to the Means of
Commission
Calimutan v. People (G.R. No. 152133)
Facts:
Victim Cantre and Saano, together with two other
companions had a drinking spree in a videoke bar at ten
oclock in the morning of February 4, 1996. Thereafter, they
decided to part ways and went to their respective houses. On
their way home, Cantre and Sanano met the petitioner and
Michael Bulalacao. Cantre suddenly punched Bulalacao
because he is suspecting the latter as the one responsible for
throwing stones at his house on previous night. After being
hit, bulalacao ran away. Petitioner picked-up a stone which is
as big as mans fist, ran toward Cantre, and threw it to the
latter, hitting him at the left side of his back. When Cantre
turned his attention to the petitioner, Sanano tried pacify the
two. Both Cantre and petitioner calmed down and went to
their houses. When Cantre arrived at his house, he
complained of the pain in the left side of his back which was
hit by the stone. At that night, he again complained of
backache and also of stomachache. Hes condition
immediately became worst, and at around three oclock in the
following morning, Cantre died.
Right after his death, Cantre was examined by Dr.
Conchita S. Ulanday, the Municipal Health Officer and made a
findings that the cause of death was cardio-respiratory arrest
due to suspected food poisoning. Unsatisfied, the Cantre
family requested for an exhumation and autopsy of the body
of the victim by the NBI. Dr. Mendez conducted an
exhumation and autopsy and reported that the cause of the
death was traumatic injury of the abdomen. The victim
suffered from an internal hemorrhage and there was massive
accumulation of blood in his abdominal cavity due to his
lacerated spleen caused by any blunt instrument, such as a
stone.
Petitioner alleged that he only attempted to pacify
the victim but the latter refused and pulled out eight-inch
Balisong. When he saw the victim was about to stab
Bulalacao, he picked up a stone and threw it at the victim
Cantre. He was able to hit the victim. He contended that the
throwing of the stone was in defense of his companion.
The RTC rendered a decision, which was later
affirmed by the CA, holding that petitioner was criminally
liable for homicide and that the act of throwing a stone from
behind was a treacherous one and the accused committed a
felony which caused the death of the victim and held that the
accused is criminally liable for all the direct and natural
consequences of this unlawful act even if the ultimate result
had not been intended. Hence, these case.
Issue:
Whether or not the petitioner has the intent to kill
the victim and thus liable for homicide?
Decision:
While the Supreme Court is in accord with the
factual findings of the RTC and the CA and affirms that there
is ample evidence proving that the death of the victim Cantre

was caused by his lacerated spleen which is the result by the


stone thrown at him by petitioner Calimutan, it nonetheless,
is at variance with the RTC and the CA as to the determination
of the appropriate crime or offense for which the petitioner
should have been convicted for.
Article 3 of the Revised Penal Code classifies felonies
according to the means by which they are committed, in
particular: (1) intentional felonies, and (2) culpable felonies.
These two types of felonies are distinguished from each other
by the existence or absence of malicious intent of the
offender.
In intentional felonies, the act or omission of the
offender is malicious. In the language of Art. 3, the act is
performed with deliberate intent (with malice). The offender,
in performing the act or in incurring the omission, has the
intention to cause an injury to another. In culpable felonies,
the act or omission of the offender is not malicious. The injury
caused by the offender to another person is "unintentional, it
being
simply
the
incident
of
another
act
performed without malice." (People vs. Sara, 55 Phil. 939). As
stated in Art. 3, the wrongful act results from imprudence,
negligence, lack of foresight or lack of skill.
In the Petition at bar, this Court cannot, in good
conscience, attribute to petitioner any malicious intent to
injure, much less to kill, the victim Cantre; and in the absence
of such intent, this Court cannot sustain the conviction of
petitioner Calimutan for the intentional crime of homicide, as
rendered by the RTC and affirmed by the Court of Appeals.
Instead, this Court finds petitioner Calimutan guilty beyond
reasonable doubt of the culpable felony of reckless
imprudence resulting in homicide under Article 365 of the
Revised Penal Code. The prosecution did not establish that
petitioner Calimutan threw the stone at the victim Cantre
with the specific intent of killing, or at the very least, of
harming the victim Cantre. What is obvious to this Court was
petitioner Calimutans intention to drive away the attacker
who was, at that point, the victim Cantre, and to protect his
helper Bulalacao who was, as earlier described, much
younger and smaller in built than the victim Cantre.

Manuel v. People (G.R. No. 165842)


Facts:
This is a case filed against Eduardo Manuel for bigamy by Tina
B. Gandalera. Complainant allege that she met the petitioner
in Dagupan City sometime in January 1996. When he visited
her in Baguio, as one thing led to another, they went to a
motel where, Eduardo succeeded in having his way with her.
Petitioner proposed marriage and even brought his parents to
assure that he is single. Tina finally accepted the marriage
proposal and they were married on April 22, 1996. In their
marriage contract, it appeared that Eduardo is single.
However, their happy relationship turns into a disaster,
Manuel started making himself scarce and went to their
house only twice or thrice a year. One day, petitioner took all
of his cloths, left and never returned. Out of curiousity, Tina
went to NSO in Manila where she found out that petitioner

had been previously married to Rubylus Gaa. She was so


embarrassed and humiliated when she learned that Eduardo
was in fact already married when they exchanged their own
vows.
For his part, Eduardo testified that he informed Tina
of his previous marriage, but she nevertheless agreed to
marry him. He abandoned her when he noticed that she had
a "love-bite" on her neck, suspecting it that it come from
another man. Eduardo further testified that he declared he
was "single" in his marriage contract with Tina because he
believed in good faith that his first marriage was invalid. He
did not know that he had to go to court to seek for the
nullification of his first marriage before marrying Tina..
Rubylus was charged with estafa in 1975 and thereafter
imprisoned. He visited her in jail after three months and
never saw her again. He insisted that he married Tina
believing that his first marriage was no longer valid because
he had not heard from Rubylus for more than 20 years. After
trial, the court rendered judgment finding Eduardo guilty
beyond reasonable doubt of bigamy. It declared that
Eduardos belief, that his first marriage had been dissolved
because of his first wifes 20-year absence, even if true, did
not exculpate him from liability for bigamy and that even if
the private complainant had known that Eduardo had been
previously married, the latter would still be criminally liable
for bigamy. Eduardo appealed the decision to the CA
maintaining his contentions. He insisted that conformably to
Article 3 of the Revised Penal Code, there must be malice for
one to be criminally liable for a felony. He was not motivated
by malice in marrying the private complainant because he did
so only out of his overwhelming desire to have a fruitful
marriage. Hence, these case.
Issue:
Whether or not the petitioner has criminal intent to
contract on the second marriage to be liable for bigamy?
Decision:
The Supreme Court ruled that the prosecution proved that
the petitioner was married to Gaa in 1975, and such
marriage was not judicially declared a nullity; hence, the
marriage is presumed to subsist. The prosecution also proved
that the petitioner married the private complainant in 1996,
long after the effectivity of the Family Code. The petitioner is
presumed to have acted with malice or evil intent when he
married the private complainant. As a general rule, mistake of
fact or good faith of the accused is a valid defense in a
prosecution for a felony by dolo; such defense negates malice
or criminal intent. However, ignorance of the law is not an
excuse because everyone is presumed to know the
law. Ignorantia legis neminem excusat. It was the burden of
the petitioner to prove his defense that when he married the
private complainant in 1996, he was of the well-grounded
belief that his first wife was already dead, as he had not
heard from her for more than 20 years since 1975. He should
have adduced in evidence a decision of a competent court
declaring the presumptive death of his first wife as required
by Article 349 of the Revised Penal Code, in relation to Article
41 of the Family Code. Such judicial declaration also
constitutes proof that the petitioner acted in good faith, and

would negate criminal intent on his part when he married the


private complainant and, as a consequence, he could not be
held guilty of bigamy in such case. The petitioner, however,
failed to discharge his burden.
Article 3, paragraph 2 of the Revised Penal Code
provides that there is deceit when the act is performed with
deliberate intent. Indeed, a felony cannot exist without intent.
Since a felony by dolo is classified as an intentional felony, it is
deemed voluntary. Although the words "with malice" do not
appear in Article 3 of the Revised Penal Code, such phrase is
included in the word "voluntary."
Malice is a mental state or condition prompting the
doing of an overt act without legal excuse or justification from
which another suffers injury. When the act or omission
defined by law as a felony is proved to have been done or
committed by the accused, the law presumes it to have been
intentional. Indeed, it is a legal presumption of law that every
man intends the natural or probable consequence of his
voluntary act in the absence of proof to the contrary, and
such presumption must prevail unless a reasonable doubt
exists from a consideration of the whole evidence.
Mistake of Fact
U.S. v. Ah Chong (15 Phil. 488)
Facts:
Defendant herein a chinese man named Ah Chong is
employed us a cook at Fort Mckinley. At that time there were
rumours and accounts of frequent robbing of homes in the
area.
On the night of the killing, Ah chong before going to
bed, and afraid of the rumoured robberies taking place in the
vicinity locked himself in their room by placing wooden blocks
and chairs for the purpose of thwarting robbers in case they
tried to rob him
After having gone to bed, he was awakened by the
noise of someone trying to open the door. Ah Chong for his
part called out twice, Who is there, but to no avail. Fearing
that the person trying to enter was robber Ah Chong leaped
from his bed and shouted If you enter the room I will kill
you. But at that precise moment, he was suddenly struck by
the chair that he had placed in the door, and believing that he
was being attacked he seized a knife and struck it on the
supposed assailant/robber, who was killed by the blow.
However the deceased was not a robber not intruder it
turned out that the person was his roommate, trying to enter
their room.
Issue: Whether or not Ah Chong is criminally liable?
Decision:
NO. Ah Chong must be acquitted on the basis of
honest mistake of fact. Where the facts been as Ah Chong
perceived them to be, he would have been justified in killing
the intruder under Article 11 of the Revised, par. 1 of the
Revised Penal Code, which provides for a valid self-defense of
his person. If the intruder was indeed a robber, forcing his
way to enter the room, unlawful aggression would be
present. Also the necessity means to avoid or to repel the

attack would be reasonable. Using the knife to defend


himself. And lastly Ah Chong gave no provocation at all to
warrant such aggression. The Supreme Court Held that there
is nothing unlawful in the intention as well in the act of Ah
Chong, his act would not have been a felony if the real
scenario was the facts he believed them to be.

ordinary precaution that he should have used before talking


such fatal action. Hence he is liable for homicide through
reckless negligence.

Diego v. Castillo (A.M. No. RTJ-02-1673)


Facts:

People v. Fernando (G.R. No. L-24978)


Facts:
The residents of Barrio of Municahan of the
Municipality of Zamboanga were alarmed by the presence of
3 suspicious looking persons prowling around the town,
suspecting them as moro prisoners who recently escaped
from Jail.
Fernando the accused herein was a policeman, when
passing in front of the house of Remigio Delgado he was
called by the latters daughter and said to him that her father
wanted to talk to him. Remigio told Fernando that 3 unknown
and suspicious looking fellows were prowling around the
house, dressed in blue same as those purportedly worn by
the escapees. Fernando stayed in the house talking to the
daughter of Remigio,, both seated in a bench near the
window. At about 7 oclock in the evening , there appeared a
figure in the dark about 4 meters from the stairs, a person in
dark clothes, calling Mang Miong. Fernando and the daughter
of Remegio had no idea who was calling. Fernando asked the
man what he wanted but instead of answering the question
the man continued to the walk with bolo in hand. Fernando
upon seeing this took out his revolver and fired a warning
shot. Thereafter having fired a shot into the air the man
continued his ascend to the stairs, Fernando took a shot at
him. However it was found out that the unknown man was
Buenaventura Paulino, nephew of Remigio.
The trial court held that Fernando was guilty of the
crime of murder. Hence this appeal.
Issue: Whether or not Fernando is criminally liable for his
acts?
Decision:
Yes. But not for the crime of murder. The accused
being agent of the law, to whom notice was given of the
presence of the suspicious looking persons who might be the
escapees. The appearance of a man unknown to him,
dressed in clothes as that of the escaped convicts, and calling
to the owner of the house, of which the daughter of the
owner of the house did not also recognized, caused the
accused to suspect that the unknown man was one of the
escaped convicts, and after firing a warning shot, the man still
did not halt his advance with bolo in hand. In the midst of the
circumstances and believing that the man was a wrongdoer
he tried to perform his duty and first fired into the air and
then at the allege intruder. At that psychological moment
when the forces of far and the sense of duty were at odds,
the accused was not able to take full account of the true
situation. However, a circumstance that should have made
him suspect that the man was not only a friend but a relative
when the man called Nong Miong, and in not asking the
daughter of the owner of the house who was it who was
calling to her father with such familiarity, he did not use the

This is an administrative complaint filed against


herein respondent for Gross ignorance of the law in rendering
his decision in a criminal complaint for bigamy.
On 1965 Lucena Escoto contracted marriage with
Jorge de Perio Jr. Both of which were Filipino Citizens.
However on February 15, 1978 the two acquired a Decree of
Divorce in Texas, USA.
On June 4, 1987 the same Lucena Escoto contracted
marriage with herein complainants brother Manule P. Diego,
celebrated at Dagupan.
Judge Castillo held in this case the acquittal of Ms.
Escoto on the basis of good faith on her part. That Ms. Escoto
believing that her previous marriage had been validly
dissolved by the divorce decree acquired in a foreign country
and that she was legally free to contract the second marriage.
That according to Judge Castillo as an ordinary laywoman ,
she entertains the impression that she can contract a
subsequent marriage. Furthermore Judge Castillo stressed
that knowledge of the law should not be exacted strictly from
her since she is a lay person, and that ineptitude should not
be confused with criminal intent.
Issue: Whether or not mistake of fact to cut-off the criminal
liability of Ms. Escoto was validly taken up by Judge Castillo?
Decision:
No. As carefully distinguished by the Supreme Court
in its previous decisions that mistake of fact, which would
could be a valid defense of good faith in a bigamy case, from
mistake of law, which does not excuse a person, even a lay
person, from liability. In People vs. Bidtu the Supreme Court
held that even if the accused, who had obtained a divorce
decree under Mohammedan custom, honestly believed that
in contracting her second marriage she was not committing
any violation of law, and that she had no criminal intent, the
same does not justify the her act. The Court further that it is
sufficient to say that everyone is presumed to know the law,
and the fact that one doe not know that his act constitutes a
violation of law does not exempt him from the consequence
thereof.

Mala in se v. Mala prohibita


Estrada v. Sandiganbayan (G.R. No. 148560)
Facts:
Petitioner Former President Joseph Estrada was
prosecuted for a crime of violation of RA 7080 (An Act
Defining and Penalizing the Crime of Plunder), as amended by
RA 7659. Thus, he questions the constitutionality of the said
Law. One of the issues that was raised in the petition is
whether Plunder as defined in RA 7080 is a malum

prohibitum, and if so, whether it is within the power of


Congress to so classify it.
Issue: Whether or not Plunder is a crime malum prohibitum?
Decision:
The legislative declaration in R.A. No.7659 that
plunder is a heinous offense implies that it is a malum in
se. For when the acts punished are inherently immoral or
inherently wrong, they are mala in se and it does not matter
that such acts are punished in a special law, especially since in
the case of plunder the predicate crimes are mainly mala in
se. Indeed, it would be absurd to treat prosecutions for
plunder as though they are mere prosecutions for violations
of the Bouncing Check Law (B.P. BIg. 22) or of an ordinance
against jaywalking, without regard to the inherent wrongness
of the acts.
People v. Go Shiu Ling (G.R. No. 115156)
Facts:
The Regional Trial Court of Pasay City finds accusedappellant Antonio Comia guilty of conspiring with four others
to import regulated drugs in violation of Art. III, Section 14 in
relation to Article IV, Section 21 of the Dangerous Drugs Act
(Rep. Act No. 6425, as amended).
Issue: Whether or not a crime for violation of Dangerous
Drugs Act is a crime malum prohibitum?
Decision:
Even granting that Comia
acted in good faith, he cannot escape criminal responsibility.
The crime with which he is charged is a malum prohibitum.
Lack of criminal intent and good faith are not exempting
circumstances. As held inPeople v. Lo Ho Wing:
Moreover, the act of transporting a prohibited drug
is a "malum prohibitum" because it is punished as an
offense under a special law. It is a wrong because it
is prohibited by law. Without the law punishing the
act, it cannot be considered a wrong. As such, the
mere commission of said act is what constitutes the
offense punished and suffices to validly charge and
convict an individual caught committing the act so
punished, regardless of criminal intent.
Likewise, in People v. Bayona, it was held:
The rule is that in acts mala in se there must be a
criminal intent, but in those mala prohibita it is
sufficient if the prohibited act was intentionally
done. "Care must be exercised in distinguishing the
difference between the intent to commit the crime
and the intent to perpetrate to act."
People Vs. Bayona (61 Phil 181)
Facts:
Defendant was driving his automobile on a road in front
electoral precinct No. 1 in barrio de Aranguel, Pilar, Capiz. He
had a revolver with him. He was called by his friend, Jose D.
Benrilo. He alighted from his automobile and approached him
to find out what he wanted. He did not leave his revolver in
the automobile, because there were many people in the road
in front of the polling place and he might lose it. He was
within the fence surrounding the polling place when Jose E.

Desiderio, a representative of the Department of the Interior,


took possession of the revolver defendant was carrying.
The solicitor General was for his Acquittal.
Held: The law which defendant violated is a statutory
provision, and the intent with which be violated is immaterial.
It may be conceded that the defendant did not intended to
intimidate any elector of to violate the law in any other way,
but when he got out of his car and carried his revolver inside
of the fence surrounding the polling place, he committed an
act complained of, and he committed it wilfully. The Election
law does not require for its violation that the offender has
the intention to intimidate the voters to interfere otherwise
with the election. The rule is that in the acts mala in se, there
must be a criminal intent; but in those mala prohibita, it is
sufficient if the prohibited act was intentionally done. Since
the election code prohibits and punishes the carrying of a
firearm inside the polling place, and that person did the
prohibited act freely and consciously, he had the intent to
perpetuate the act.
Art. 4: Criminal Liability
Elements of Criminal Liability
US v Mallari 29 phil 14, 19
G.R. No. L-10037 December 23, 1914
THE UNITED STATES, plaintiff-appellee,
vs.
MAXIMO MALLARI, defendant-appellant.
Ledesma, Lim and Irureta Goyena for appellant.
Office of the Solicitor-General Corpus for appellee.
TORRES, J.:
This case has been brought up on appeal filed by the
defendant from the judgment dated April 22, 1914, whereby
the Honorable Julio Llorente, judge, sentenced him to the
penalty of twelve years and one day of reclusion temporal,
accessories, the payment of an indemnity of P1,000 to the
heirs of the deceased, and the costs.
On the morning of September 25, 1913, in the barrio of
Batasan, municipality of Macabebe, Province of Pampanga,
before going to his work, the defendant Maximo Mallari went
to the house of the married couple, Vicente Sunga and
Canuta Flores, and from the shed outside asked Vicente
Sunga to cure his wife of a sickness from which she had been
suffering for several days, and which he thought was due to
enchantment on the part of the said Vicente. As the latter
refused, averring that he was not a wizard and that he had
not caused the illness of defendant's wife, the former became
enraged and insulted the said spouses. Threatening to kill
them, he ascended the stairway carrying in his hand a thin,
sharp bolo. At his wife's suggestion Vicente Sunga tried to get
out to report the matter to the teniente of the barrio, who
lived at some paces from their house, but as he met the

defendant on the stairway, he immediately went back inside


and jumped out of a window. He was straightway pursued by
the defendant and on arriving almost in front of the house of
the teniente saw that the defendant was following closely
behind him. He therefore turned to face his pursuer and
defend himself as well as he could with his hands. Thereupon
the defendant with a single slash of the bolo wounded
Vicente Sunga in the abdomen, so that his intestines
protruded therefrom. In this condition the victim sat down,
endeavoring with his hands to keep his intestines from falling
out, while his assailant took to flight.
The justice of the peace of Macabebe arrived on the scene a
few moments later and in his presence the wounded man
declared that his assailant was Mallari, who had been in his
house, and who had inflicted the serious wound he had in the
abdomen. As a consequence of this wound he died three days
later.
An autopsy was held on the corpse by the physician who is
president of the municipal board of health of Macabebe, and
as a result of the examination made, it appeared that the
deceased had received an incised wound in the epigastric
region, which penetrated the gastrocolic epiplon, the middle
part of the transverse colon and the rectoabdominal muscles,
and which had cause severe peritonitis, the entire length of
both intestines having become gangrened with sanious
discharges. This wound was necessarily fatal, especially in
view of the scarcity of antiseptics available in the towns of
the provinces.
The facts set forth appear to have been duly proven in the
case and constitute the crime of homicide, provided for and
penalized in article 404 of the Penal Code, for the reason that
the defendant appeared at the house of deceased with the
demand that the latter treat his wife, whom he believed to be
bewitched by the artifices of the deceased. As the latter
refused to do so, saying that he was not a wizard, the
defendant Mallari insulted the Sunga spouses, threatened
them with death and straightway went up into the house of
the deceased with a bolo in his hand. When the deceased
saw this he immediately leaped out of the window in flight
but the defendant pursued him and upon coming up with him
in front of the house of the teniente of the barrio, to whom
he was going to complain, struck him a blow in the abdomen
with the bolo, inflicting a serious and fatal wound that caused
death on the third day thereafter.
Defendant declared that on the morning of the occurrence
his wife, who was ill, told him before going to work to stop in
at Isidro Sunga's house and ask the latter's wife to treat her,
for it was reported that she was a witch; that when he
reached the yard of Isidro Sunga's house he asked the latter's
son, Silvino Sunga, for his mother; that at Silvino's invitation
he entered the yard and Isidro Sunga asked him from the
window what he wanted and invited him to come in; but that
he refused to enter, saying that he could tell what he wanted
from outside; that thereupon Isidro Sunga's wife appeared at

the window and upon seeing him called to her children,


saying that a bad man had come. Then they began to insult
him, and when tried to get away, Vicente Sunga, Silvino
Sunga, Isidro Sunga, and Florentino Sunga, armed with long
bolos, pursued him. At that instant Vicente Sunga, who was
foremost among them, struck him a blow with a pocket-knife
on one of his rumps. Upon feeling the wound he turned and
slashed his assailant in the belly with his bolo, and then
forthwith took to flight, for the others continued to pursue
him. Defendant attempts to show by this testimony that he
was through necessity defending himself from the unlawful
assault made upon him by the deceased.
But it appears fully proven in the case that the defendant
Mallari did not go to Isidro Sunga's house to perform his
wife's errand but that he went directly to another house, in
which Vicente Sunga lived apart, to compel the latter to cure
his sick wife; that when the defendant pursued him in the
direction of the house of the teniente of the barrio, Sunga's
wife, Canuta Flores, and her brother, Dalmacio Flores, came
out of the house unarmed and saw the assault; that upon
approaching Sunga, whom they found seated on the ground
with a wound in his belly, they rendered him aid. At that time
the defendant, who had assaulted him, was no longer there,
as he had straightway taken to flight. The agents of the
authorities and the justice of the peace who went to the
place of the assault found no weapon in possession of the
deceased, or of his wife and his brother-in-law.
Isidro Sunga, who lived in another house with Silvino Sunga
and Florentino Sunga, was not in his house at the time of the
occurrence and none of these persons saw the affair, so they
could not have pursued the defendant with weapons, as he
affirms. The two neighbors, Bernardo Saual and Esteban
Yama, who lived in houses adjoining the deceased's, stated
that when they heard the cries of Canuta Flores calling for
help they left their houses and thereupon saw the defendant
pursuing the deceased Vicente Sunga; that upon arriving in
front of the house of the teniente of the barrio, at the
moment when Sunga turned toward the defendant, who was
pursuing him, said defendant slashed him in the belly and
straightway took to flight. They did not at that time see Sunga
carrying any weapon, or that he attacked his assailant before
being wounded in the belly.
The witnesses for the defense, Bonifacio Ignacio and Diego
Yabut, assert that they saw four armed men pursuing the
defendant on that occasion, the foremost of whom was the
deceased, who was carrying a pocket-knife, while the others
were provided with bolos, wherefore the defendant, upon
being wounded in the right rump by the deceased, defended
himself and slashed the latter with his bolo. To offset these
declarations, the case affords positive proof that the
defendant, without provocation or prior aggression, attacked
the deceased with a bolo and wounded him in the belly.
The affirmations of said witnesses cannot prevail against the
testimony of two impartial neighbors who came up at the

cries of the deceased's wife calling for help and who, along
with the brother-in-law of the wounded man, witnessed the
occurence and rendered him aid, without having seen those
two witnesses of the defense on the spot or the alleged prior
aggression of the deceased against the defendant.
Moreover, when the defendant was asked by the chief of
police how he came to be wounded in the rump, he replied
that he did not know who had inflicted that wound, and this
statement of the chief was not impugned at the trial, so it
may be that the defendant in preparing his defense wounded
himself with pocket-knife, for the wound was of a trifling
nature; and at the investigation held by the justice of the
peace who went to the defendant's house, the latter made
no statement regarding the person who had inflicted said
wound, while the physician who examined him said that it
must have been received when he was standing still.
It does not therefore appear to be duly proven in the case
that the defendant was attacked and wounded with a pocketknife by the deceased, and consequently that there was any
prior unlawful aggression on the part of the deceased, to
justify the finding that the defendant was compelled to
wound him in the belly with a bolo in lawful self-defense. The
plea or circumstance of exemption from responsibility must
be fully proven in the same way as the principal fact, in order
to hold that the perpetrator of the crime is not responsible
therefor.
With reference to the classification of the criminal act, it does
not appear in the case that this was erroneous, for in spite of
the statement of the health officer that the deceased might
have been saved if the wound had been aseptically treated
from the first, its seriousness and fatal character being due to
lack of antiseptics, still the person inflicting it is responsible
for all the consequences of his criminal action, and therefore
for the death that occured some days after the deceased
received the wound.
With respect to the third error assigned to the court for
having held that the declarations made by the deceased
before the justice of the peace had the character of ante
mortem declarations, when the death of the deceased did
not occur for three days and the wound was not in itself of a
fatal nature, it must be remembered that as a result of the
wound inflicted upon the deceased his intestines protruded
and for this reason the wound was of a serious if not fatal
nature. Therefore the opinion of the court with reference to
the nature and force of the statements made by the victim
before the justice of the peace at the time of the
investigation is quite proper, because the credibility of
statements made by a person severely wounded rests not
only on the serious situation resulting from his wound but
also on his physical and mental condition, which, given the
depressed state of his mind, has induced the profound
conviction that his life is actually slipping away, and that he is
in positive and imminent danger of dying sooner or later from
the wound; nor can the force of such declaration be affected

by the circumstance that he died later, hours or days after it


was inflicted, for when the patient did finally die his death
was due to the wound whose gravity did not diminish from
the time he made his declaration until the hour of his death.
In the case of Moore vs. State (96 Tenn., 209) the principle
was maintained that "a dying declaration, otherwise
competent, will not be excluded because it was made five
days before death."
In the syllabus of the decision in the case of
Daughdrill vs. State (113 Ala., 7, 9) it is held: "Where it is
shown that deceased had received a dangerous wound and
stated several times that he was dying, and wanted witness
to take a message to his wife, and was gasping for breath at
the time he was talking, and trembling from fear or
excitement, a sufficient predicate is laid for the admission of
a statement made at that time, as a dying declaration; and it
is no objection to its admission that such declaration took the
form of a message to another."
In the syllabus of the decision rendered in the case of the
United Stated vs. Castellon (12 Phil. Rep., 160) it is held:
"Notwithstanding the fact that hearsay evidence is not
admissible at a trial, the statement made by an individual
who is seriously wounded, at a moment when he was dying,
being conviced that there was no hope of recovery,
constitute per se at least a grave, conclusive and decisive
indication of the culpability of the persons designated by the
dying man, inasmuch as it must be assumed that he, being in
so precarious a condition, spoke truthfully, and that he was
not induced by a desire to tell a lie and to injure an innocent
person."
For the foregoing reasons, whereby the errors assigned to the
judgment appealed from are refuted and said judgment
found to be in accordance with the law and the merits of the
case, it is affirmed, with the costs against the appellant.

Bataclan v Medina 102 phil 181


After one midnight in September 1952, Juan Bataclan rode a
bus owned by Medina from Cavite to Pasay. While on its way,
the driver of the bus was speeding through and when he
applied the brakes it cause the bus to be overturned. The
driver, the conductor, and some passengers were able to free
themselves from the bus except Bataclan and 3 others. The
passengers called the help of the villagers and as it was dark,
the villagers brought torch with them. The driver and the
conductor failed to warn the would-be helpers of the fact
that gasoline has spilled from the overturned bus so a huge
fire ensued which engulfed the bus thereby killing the 4
passengers trapped inside. It was also found later in trial that
the tires of the bus were old.
ISSUE: Whether or not the proximate cause of the death of
Bataclan et al was their burning by reason of the torches
which ignited the gasoline.

HELD: No. The proximate cause was the overturning of the


bus which was caused by the negligence of the driver because
he was speeding and also he was already advised by Medina
to change the tires yet he did not. Such negligence resulted to
the overturning of the bus. The torches carried by the wouldbe helpers are not to be blamed. It is just but natural for the
villagers to respond to the call for help from the passengers
and since it is a rural area which did not have flashlights,
torches are the natural source of lighting. Further, the smell
of gas could have been all over the place yet the driver and
the conductor failed to provide warning about said fact to the
villagers.
WHAT IS PROXIMATE CAUSE?
Proximate cause is that cause, which, in natural and
continuous sequence, unbroken by any efficient intervening
cause, produces the injury, and without which the result
would not have occurred.
And more comprehensively, the proximate legal cause is that
acting first and producing the injury, either immediately or by
setting other events in motion, all constituting a natural and
continuous chain of events, each having a close causal
connection with its immediate predecessor, the final event in
the chain immediately effecting the injury as a natural and
probable result of the cause which first acted, under such
circumstances that the person responsible for the first event
should, as an ordinary prudent and intelligent person, have
reasonable ground to expect at the moment of his act or
default that an injury to some person might probably result
therefrom.
Quinto v. Andres (453 SCRA 511)
Facts:
On Nov. 13,1995, Dante Andres and Randyven
Pacheco invited Wilson Quinto and Edison Garcia to go fishing
with them inside a drainage culvert. However, only Quinto
joined the two, Garcia remain in a grassy area about two
meters from the entrance of the drainage system. After a
while, Pacheco came out, went back again, and emerged
again carrying Wilson who was already dead. He laid the
boys lifeless body down in the grassy area and went to the
house of Wilsons mother and informed her that her son had
died. After more than three months, the cadaver of Wilson
was exhumed and the NBI performed an autopsy thereon. An
information was later filed with the RTC changing Andres and
Pacheco with homicide.
Issue:
Whether or not the accused has criminal liability for
the death of the victim?
Decision:
The prosecution failed to prove the guilt of the
accused beyond reasonable doubt. It failed to prove the guilt
of the accused is criminality liable although the wrongful act
done be different from that which he intended. The Supreme
Court agreed with the trial and appellate courts that the
proximate cause of the death of the victim was not cause by
any wrongful act of the accused. It is the burden of the
prosecution to prove the corps delicti which consists of

criminal act and the defendants agency in the commission of


the act. This, the prosecution failed to do.

People v Pilola 405 scra 134


Facts:
Accused Edman Aguilos, Odilon Lagliba and Rene Pilola were
charged with murder for the death of Joselito Capa. Rene
Pilola devised stabbing the victim and interposed the defense
of alibi. The trial count found all the accused guilty and
sentenced them to reclusion perpetua. Rene Pilola appealed
the decision by contending that there was no conspiracy and
he may not be held criminally liable as principal by direct
participation. He argued that the prosecution failed to prove
that he conspined with the others in stabbing the victim to
death. He asserts that he is merely an accomplice.
ISSUE: Whether or not the appellant may be held criminally
liable as principal by direct participation in the absence of
proof of conspiracy?
HELD:
The court in applying paragraph 1, Article 4 of the Revised
Penal Code ruled that even if two or more offenders do not
conspire to commit homicide or murder, they may be held
criminally liable as principals by direct participation if they
perform overt acts which immediately cause or accelerate the
death of the victim. They are all criminally liable although the
wrongful act done be different from that which he intended
by reason of their individual and separate overt criminal acts.
WHErEFORE, this Court finds RENE GAYOT PILOLA of 606
Nueve de Febrero Street, Mandaluyong City, GUILTY beyond
reasonable doubt of Murder punished under Article 248 of
the Revised Penal Code, and there being no mitigating nor
aggravating circumstances, he is hereby sentenced
to reclusion perpetua. Pilola is hereby ordered to indemnify
the heirs of deceased Joselito Capa alias Jessie in the amount
of FIFTY THOUSAND PESOS (P50,000.00) as indemnity for his
death jointly and solidarily with Odilon Lagliba who was
earlier convicted herein. With cost against the accused.
People vs. Villacorta 657 scra 270

Republic of the Philippines


SUPREME COURT
Manila
FIRST DIVISION
G.R. No. 186412

September 7, 2011

PEOPLE OF THE PHILIPPINES, Plaintiff-Appellee,


vs.
ORLITO VILLACORTA, Accused-Appellant.
DECISION
LEONARDO-DE CASTRO, J.:

On appeal is the Decision dated July 30, 2008 of the Court of


Appeals in CA-G.R. CR.-H.C. No. 02550, which affirmed the
2
Decision dated September 22, 2006 of the Regional Trial
Court (RTC), Branch 170, of Malabon, in Criminal Case No.
27039-MN, finding accused-appellant Orlito Villacorta
(Villacorta) guilty of murder, and sentencing him to suffer the
penalty of reclusion perpetua and to pay the heirs of Danilo
Cruz (Cruz) the sum ofP50,000.00 as civil indemnity, plus the
costs of suit.
3

On June 21, 2002, an Information was filed against Villacorta


charging him with the crime of murder, as follows:
That on or about 23rd day of January 2002, in Navotas, Metro
Manila, and within the jurisdiction of this Honorable Court,
the above-named accused, armed with a sharpened bamboo
stick, with intent to kill, treachery and evident premeditation,
did then and there willfully, unlawfully and feloniously attack,
assault and stab with the said weapon one DANILO
SALVADOR CRUZ, thereby inflicting upon the victim serious
wounds which caused his immediate death.
When arraigned on September 9, 2002, Villacorta pleaded
4
not guilty.
During trial, the prosecution presented as witnesses Cristina
Mendeja (Mendeja) and Dr. Domingo Belandres, Jr. (Dr.
Belandres).
Mendeja narrated that on January 23, 2002, she was tending
her sari-sari store located at C-4 Road, Bagumbayan, Navotas.
Both Cruz and Villacorta were regular customers at Mendejas
store. At around two oclock in the morning, while Cruz was
ordering bread at Mendejas store, Villacorta suddenly
appeared and, without uttering a word, stabbed Cruz on the
left side of Cruzs body using a sharpened bamboo stick. The
bamboo stick broke and was left in Cruzs body. Immediately
after the stabbing incident, Villacorta fled. Mendeja gave
chase but failed to catch Villacorta. When Mendeja returned
to her store, she saw her neighbor Aron removing the broken
5
bamboo stick from Cruzs body. Mendeja and Aron then
6
brought Cruz to Tondo Medical Center.
Dr. Belandres was Head of the Tetanus Department at the
San Lazaro Hospital. When Cruz sustained the stab wound on
January 23, 2002, he was taken to the Tondo Medical Center,
where he was treated as an out-patient. Cruz was only
brought to the San Lazaro Hospital on February 14, 2002,
where he died the following day, on February 15, 2002. While
admitting that he did not personally treat Cruz, Dr. Belandres
was able to determine, using Cruzs medical chart and
diagnosis, that Cruz died of tetanus infection secondary to
7
stab wound. Dr. Belandres specifically described the cause of
Cruzs death in the following manner:
The wound was exposed x x spurs concerted, the patient
developed difficulty of opening the mouth, spastivity of the

body and abdominal pain and the cause of death is hypoxic


encephalopathy neuro transmitted due to upper G.I.
8
bleeding x x x. Diagnosed of Tetanus, Stage III.
The prosecution also intended to present Dr. Deverni Matias
(Dr. Matias), who attended to Cruz at the San Lazaro Hospital,
but the prosecution and defense agreed to dispense with Dr.
Matias testimony based on the stipulation that it would only
corroborate Dr. Belandres testimony on Cruz dying of
tetanus.
For its part, the defense presented Villacorta himself, who
denied stabbing Cruz. Villacorta recounted that he was on his
way home from work at around two oclock in the morning of
January 21, 2002. Upon arriving home, Villacorta drank coffee
then went outside to buy cigarettes at a nearby store. When
Villacorta was about to leave the store, Cruz put his arm
around Villacortas shoulder. This prompted Villacorta to box
Cruz, after which, Villacorta went home. Villacorta did not
notice that Cruz got hurt. Villacorta only found out about
9
Cruzs death upon his arrest on July 31, 2002.
On September 22, 2006, the RTC rendered a Decision finding
Villacorta guilty of murder, qualified by treachery. The
dispositive portion of said Decision reads:
WHEREFORE, in the light of the foregoing, the Court finds
accused Orlito Villacorta guilty beyond reasonable doubt of
the crime of Murder and is hereby sentenced to suffer the
penalty of reclusion perpetua and to pay the heirs of Danilo
Cruz the sum of P50,000.00 as civil indemnity for the death of
10
said victim plus the costs of suit.
Villacorta, through his counsel from the Public Attorneys
Office (PAO), filed a notice of appeal to assail his conviction
11
by the RTC. The Court of Appeals directed the PAO to file
Villacortas brief, within thirty days from receipt of notice.
12

Villacorta filed his Appellants Brief on May 30, 2007; while


the People, through the Office of the Solicitor General (OSG),
13
filed its Appellee's Brief on October 2, 2007.
On July 30, 2008, the Court of Appeals promulgated its
Decision affirming in toto the RTC judgment of conviction
against Villacorta.
Hence, Villacorta comes before this Court via the instant
appeal.
Villacorta manifested that he would no longer file a
supplemental brief, as he was adopting the Appellant's Brief
14
he filed before the Court of Appeals. The OSG, likewise,
15
manifested that it was no longer filing a supplemental brief.
In his Appellants Brief, Villacorta raised the following
assignment of errors:

I
THE COURT A QUO GRAVELY ERRED IN FINDING THE
ACCUSED-APPELLANT GUILTY OF THE CRIME
CHARGED DESPITE THE FAILURE OF THE
PROSECUTION TO PROVE HIS GUILT BEYOND
REASONABLE DOUBT.
II
THE TRIAL COURT GRAVELY ERRED IN APPRECIATING
THE QUALIFYING CIRCUMSTANCE OF TREACHERY.
III
ASSUMING ARGUENDO THAT THE ACCUSED
COMMITTED A CRIME, HE COULD ONLY BE HELD
16
LIABLE FOR SLIGHT PHYSICAL INJURIES.
Villacorta assails the credibility of Mendeja, an eyewitness to
the stabbing incident. It was Mendeja who positively
identified Villacorta as the one who stabbed Cruz in the early
morning of January 23, 2002. Villacorta asserts that
Mendejas account of the stabbing incident is replete with
inconsistencies and incredulities, and is contrary to normal
human experience, such as: (1) instead of shouting or calling
for help when Villacorta allegedly stabbed Cruz, Mendeja
attempted to run after and catch Villacorta; (2) while, by
Mendejas own account, there were other people who
witnessed the stabbing and could have chased after
Villacorta, yet, oddly, only Mendeja did; (3) if Cruz was
stabbed so swiftly and suddenly as Mendeja described, then
it would have been physically improbable for Mendeja to
have vividly recognized the perpetrator, who immediately ran
away after the stabbing; (4) after the stabbing, both Villacorta
and Cruz ran in opposite directions; and (5) Mendeja had said
that the bamboo stick, the alleged murder weapon, was left
at her store, although she had also stated that the said
bamboo stick was left embedded in Cruzs body. Villacorta
maintains that the aforementioned inconsistencies are
neither trivial nor inconsequential, and should engender
some doubt as to his guilt.

In this case, both the RTC and the Court of Appeals gave full
faith and credence to the testimony of prosecution witness
Mendeja. The Court of Appeals rejected Villacortas attempts
to impugn Mendejas testimony, thus:
Appellants reason for concluding that witness Mendejas
testimony is incredible because she did not shout or call for
help and instead run after the appellant, fails to impress the
Court because persons who witness crimes react in different
ways.
"x x x the makings of a human mind are unpredictable;
people react differently and there is no standard form of
behavior when one is confronted by a shocking incident.
Equally lacking in merit is appellants second reason which is,
other persons could have run after the appellant after the
stabbing incident. As explained by witness Mendeja, the
other person whom she identified as Aron was left to assist
the appellant who was wounded. Further, the stabbing
occurred at 2:00 oclock in the morning, a time when persons
are expected to be asleep in their house, not roaming the
streets.
His *Villacortas+ other argument that the swiftness of the
stabbing incident rendered impossible or incredible the
identification of the assailant cannot likewise prosper in view
of his admission that he was in the store of witness Mendeja
on January 23, 2002 at 2:00 oclock in the morning and that
he assaulted the victim by boxing him.
Even if his admission is disregarded still the evidence of
record cannot support appellants argument. Appellant and
the victim were known to witness Mendeja, both being her
friends and regular customers. There was light in front of the
store. An opening in the store measuring 1 and meters
enables the person inside to see persons outside, particularly
those buying articles from the store. The victim was in front
of the store buying bread when attacked. Further,
immediately after the stabbing, witness Mendeja ran after
the appellant giving her additional opportunity to identify the
malefactor. Thus, authorship of the attack can be credibly
18
ascertained.

We are not persuaded.


To begin with, it is fundamental that the determination by
the trial court of the credibility of witnesses, when affirmed
by the appellate court, is accorded full weight and credit as
well as great respect, if not conclusive effect. Such
determination made by the trial court proceeds from its firsthand opportunity to observe the demeanor of the witnesses,
their conduct and attitude under grilling examination,
thereby placing the trial court in the unique position to assess
the witnesses' credibility and to appreciate their truthfulness,
17
honesty and candor.

Moreover, Villacorta was unable to present any reason or


motivation for Mendeja to fabricate such a lie and falsely
accuse Villacorta of stabbing Cruz on January 23, 2002. We
have ruled time and again that where the prosecution
eyewitness was familiar with both the victim and accused,
and where the locus criminis afforded good visibility, and
where no improper motive can be attributed to the witness
for testifying against the accused, then her version of the
19
story deserves much weight.
The purported inconsistencies in Mendejas testimony
pointed out by Villacorta are on matters that have no bearing
on the fundamental fact which Mendeja testified on: that

Villacorta stabbed Cruz in the early morning of January 23,


2002, right in front of Mendejas store.
In the face of Mendejas positive identification of Villacorta as
Cruzs stabber, Villacorta could only muster an
uncorroborated denial. Denial, like alibi, as an exonerating
justification, is inherently weak and if uncorroborated,
regresses to blatant impotence. Like alibi, it also constitutes
self-serving negative evidence which cannot be accorded
greater evidentiary weight than the declaration of credible
20
witnesses who testify on affirmative matters.

germs when he returned to work on his farm only two (2)


weeks after sustaining his injury. The Court granted Urbanos
appeal.
We quote extensively from the ratiocination of the Court in
Urbano:
The issue, therefore, hinges on whether or not there was an
efficient intervening cause from the time Javier was wounded
until his death which would exculpate Urbano from any
liability for Javier's death.

Hence, we do not deviate from the foregoing factual findings


of the RTC, as affirmed by the Court of Appeals.

We look into the nature of tetanus-

Nevertheless, there is merit in the argument proffered by


Villacorta that in the event he is found to have indeed
stabbed Cruz, he should only be held liable for slight physical
injuries for the stab wound he inflicted upon Cruz. The
proximate cause of Cruzs death is the tetanus infection, and
not the stab wound.

"The incubation period of tetanus, i.e., the time between


injury and the appearance of unmistakable symptoms, ranges
from 2 to 56 days. However, over 80 percent of patients
become symptomatic within 14 days. A short incubation
period indicates severe disease, and when symptoms occur
within 2 or 3 days of injury the mortality rate approaches 100
percent.

Proximate cause has been defined as "that cause, which, in


natural and continuous sequence, unbroken by any efficient
intervening cause, produces the injury, and without which
21
the result would not have occurred."
In this case, immediately after he was stabbed by Villacorta in
the early morning of January 23, 2002, Cruz was rushed to
and treated as an out-patient at the Tondo Medical Center.
On February 14, 2002, Cruz was admitted to the San Lazaro
Hospital for symptoms of severe tetanus infection, where he
died the following day, on February 15, 2002. The
prosecution did not present evidence of the emergency
medical treatment Cruz received at the Tondo Medical
Center, subsequent visits by Cruz to Tondo Medical Center or
any other hospital for follow-up medical treatment of his stab
wound, or Cruzs activities between January 23 to February
14, 2002.
22

In Urbano v. Intermediate Appellate Court, the Court was


confronted with a case of very similar factual background as
the one at bar. During an altercation on October 23, 1980,
Urbano hacked Javier with a bolo, inflicting an incised wound
on Javiers hand. Javier was treated by Dr. Meneses. On
November 14, 1980, Javier was rushed to the hospital with
lockjaw and convulsions. Dr. Exconde, who attended to
Javier, found that Javiers serious condition was caused by
tetanus infection. The next day, on November 15, 1980, Javier
died. An Information was filed against Urbano for homicide.
Both the Circuit Criminal Court and the Intermediate
Appellate Court found Urbano guilty of homicide, because
Javier's death was the natural and logical consequence of
Urbano's unlawful act. Urbano appealed before this Court,
arguing that Javiers own negligence was the proximate cause
of his death. Urbano alleged that when Dr. Meneses
examined Javiers wound, he did not find any tetanus
infection and that Javier could have acquired the tetanus

"Non-specific premonitory symptoms such as restlessness,


irritability, and headache are encountered occasionally, but
the commonest presenting complaints are pain and stiffness
in the jaw, abdomen, or back and difficulty swallowing. As the
disease progresses, stiffness gives way to rigidity, and
patients often complain of difficulty opening their mouths. In
fact, trismus is the commonest manifestation of tetanus and
is responsible for the familiar descriptive name of lockjaw. As
more muscles are involved, rigidity becomes generalized, and
sustained contractions called risus sardonicus. The intensity
and sequence of muscle involvement is quite variable. In a
small proportion of patients, only local signs and symptoms
develop in the region of the injury. In the vast majority,
however, most muscles are involved to some degree, and the
signs and symptoms encountered depend upon the major
muscle groups affected.
"Reflex spasm usually occur within 24 to 72 hours of the first
symptoms, an interval referred to as the onset time. As in the
case of the incubation period, a short onset time is associated
with a poor prognosis. Spasms are caused by sudden
intensification of afferent stimuli arising in the periphery,
which increases rigidity and causes simultaneous and
excessive contraction of muscles and their antagonists.
Spasms may be both painful and dangerous. As the disease
progresses, minimal or inapparent stimuli produce more
intense and longer lasting spasms with increasing frequency.
Respiration may be impaired by laryngospasm or tonic
contraction of respiratory muscles which prevent adequate
ventilation. Hypoxia may then lead to irreversible central
nervous system damage and death.
"Mild tetanus is characterized by an incubation period of at
least 14 days and an onset time of more than 6 days.Trismus
is usually present, but dysphagia is absent and generalized

spasms are brief and mild. Moderately severe tetanus has a


somewhat shorter incubation period and onset time; trismus
is marked, dysphagia and generalized rigidity are present, but
ventilation remains adequate even during spasms. The
criteria for severe tetanus include a short incubation time,
and an onset time of 72 hrs., or less, severe trismus,
dysphagia and rigidity and frequent prolonged, generalized
convulsive spasms. (Harrison's Principle of Internal Medicine,
1983 Edition, pp. 1004-1005; Emphasis supplied)
Therefore, medically speaking, the reaction to tetanus found
inside a man's body depends on the incubation period of the
disease.
In the case at bar, Javier suffered a 2-inch incised wound on
his right palm when he parried the bolo which Urbano used in
hacking him. This incident took place on October 23, 1980.
After 22 days, or on November 14, 1980, he suffered the
symptoms of tetanus, like lockjaw and muscle spasms. The
following day, November 15, 1980, he died.
If, therefore, the wound of Javier inflicted by the appellant
was already infected by tetanus germs at the time, it is more
medically probable that Javier should have been infected with
only a mild case of tetanus because the symptoms of tetanus
appeared on the 22nd day after the hacking incident or more
than 14 days after the infliction of the wound. Therefore,
the onset time should have been more than six days. Javier,
however, died on the second day from the onset time. The
more credible conclusion is that at the time Javier's wound
was inflicted by the appellant, the severe form of tetanus that
killed him was not yet present. Consequently, Javier's wound
could have been infected with tetanus after the hacking
incident. Considering the circumstance surrounding Javier's
death, his wound could have been infected by tetanus 2 or 3
23
or a few but not 20 to 22 days before he died.
The incubation period for tetanus infection and the length of
time between the hacking incident and the manifestation of
severe tetanus infection created doubts in the mind of the
Court that Javier acquired the severe tetanus infection from
the hacking incident. We explained in Urbano that:
The rule is that the death of the victim must be the direct,
natural, and logical consequence of the wounds inflicted upon
him by the accused. (People v. Cardenas, supra) And since we
are dealing with a criminal conviction, the proof that the
accused caused the victim's death must convince a rational
mind beyond reasonable doubt. The medical findings,
however, lead us to a distinct possibility that the infection of
the wound by tetanus was an efficient intervening cause later
or between the time Javier was wounded to the time of his
death. The infection was, therefore, distinct and foreign to
the crime. (People v. Rellin, 77 Phil. 1038).
Doubts are present. There is a likelihood that the wound was
but the remote cause and its subsequent infection, for failure
to take necessary precautions, with tetanus may have been

the proximate cause of Javier's death with which the


petitioner had nothing to do. As we ruled in Manila Electric
Co. v. Remoquillo, et al. (99 Phil. 118).
"A prior and remote cause cannot be made the basis of an
action if such remote cause did nothing more than furnish the
condition or give rise to the occasion by which the injury was
made possible, if there intervened between such prior or
remote cause and the injury a distinct, successive, unrelated,
and efficient cause of the injury, even though such injury
would not have happened but for such condition or occasion.
If no danger existed in the condition except because of the
independent cause, such condition was not the proximate
cause. And if an independent negligent act or defective
condition sets into operation the instances, which result in
injury because of the prior defective condition, such
subsequent act or condition is the proximate cause." (45 C.J.
24
pp. 931-932). (at p. 125)
We face the very same doubts in the instant case that compel
us to set aside the conviction of Villacorta for murder. There
had been an interval of 22 days between the date of the
stabbing and the date when Cruz was rushed to San Lazaro
Hospital, exhibiting symptoms of severe tetanus infection. If
Cruz acquired severe tetanus infection from the stabbing,
then the symptoms would have appeared a lot sooner than
22 days later. As the Court noted in Urbano, severe tetanus
infection has a short incubation period, less than 14 days; and
those that exhibit symptoms with two to three days from the
injury, have one hundred percent (100%) mortality.
Ultimately, we can only deduce that Cruzs stab wound was
merely the remote cause, and its subsequent infection with
tetanus might have been the proximate cause of Cruz's death.
The infection of Cruzs stab wound by tetanus was an efficient
intervening cause later or between the time Cruz was
stabbed to the time of his death.
However, Villacorta is not totally without criminal
liability.1wphi1 Villacorta is guilty of slight physical injuries
under Article 266(1) of the Revised Penal Code for the stab
wound he inflicted upon Cruz. Although the charge in the
instant case is for murder, a finding of guilt for the lesser
offense of slight physical injuries may be made considering
that the latter offense is necessarily included in the former
since the essential ingredients of slight physical injuries
constitute and form part of those constituting the offense of
25
murder.
We cannot hold Villacorta criminally liable for attempted or
frustrated murder because the prosecution was not able to
establish Villacortas intent to kill. In fact, the Court of
Appeals expressly observed the lack of evidence to prove
such an intent beyond reasonable doubt, to wit:
Appellant stabbed the victim only once using a sharpened
bamboo stick, hitting him on the left side of the body and
then immediately fled. The instrument used is not as lethal as
those made of metallic material. The part of the body hit is

not delicate in the sense that instant death can ensue by


reason of a single stab wound. The assault was done only
once. Thus, there is doubt as to whether appellant had an
intent to kill the victim, which should be resolved in favor of
26
the appellant. x x x.
The intent must be proved in a clear and evident manner to
exclude every possible doubt as to the homicidal (or
murderous) intent of the aggressor. The onus probandi lies
not on accused-appellant but on the prosecution. The
inference that the intent to kill existed should not be drawn
in the absence of circumstances sufficient to prove this fact
beyond reasonable doubt. When such intent is lacking but
wounds were inflicted, the crime is not frustrated murder but
27
physical injuries only.
Evidence on record shows that Cruz was brought to Tondo
Medical Center for medical treatment immediately after the
stabbing incident.1avvphi1 Right after receiving medical
treatment, Cruz was then released by the Tondo Medical
Center as an out-patient. There was no other evidence to
establish that Cruz was incapacitated for labor and/or
required medical attendance for more than nine days.
Without such evidence, the offense is only slight physical
28
injuries.
We still appreciate treachery as an aggravating circumstance,
it being sufficiently alleged in the Information and proved
during trial.
The Information specified that "accused, armed with a
sharpened bamboo stick, with intent to kill, treachery and
evident premeditation, did then and there willfully,
unlawfully and feloniously attack, assault and stab with the
said weapon one DANILO SALVADOR CRUZ x x x."
Treachery exists when an offender commits any of the crimes
against persons, employing means, methods or forms which
tend directly or especially to ensure its execution, without
risk to the offender, arising from the defense that the
offended party might make. This definition sets out what
must be shown by evidence to conclude that treachery
existed, namely: (1) the employment of such means of
execution as would give the person attacked no opportunity
for self-defense or retaliation; and (2) the deliberate and
conscious adoption of the means of execution. To reiterate,
the essence of qualifying circumstance is the suddenness,
surprise and the lack of expectation that the attack will take
place, thus, depriving the victim of any real opportunity for
self-defense while ensuring the commission of the crime
29
without risk to the aggressor. Likewise, even when the
victim was forewarned of the danger to his person, treachery
may still be appreciated since what is decisive is that the
execution of the attack made it impossible for the victim to
30
defend himself or to retaliate.
Both the RTC and the Court of Appeals found that treachery
was duly proven in this case, and we sustain such finding.

Cruz, the victim, was attacked so suddenly, unexpectedly, and


without provocation. It was two oclock in the morning of
January 23, 2002, and Cruz, who was out buying bread at
Mendejas store, was unarmed. Cruz had his guard down and
was totally unprepared for an attack on his person. Villacorta
suddenly appeared from nowhere, armed with a sharpened
bamboo stick, and without uttering a word, stabbed Cruz at
the left side of his body, then swiftly ran away. Villacortas
treacherous mode of attack left Cruz with no opportunity at
all to defend himself or retaliate.
Article 266(1) of the Revised Penal Code provides:
ART. 266. Slight physical injuries and maltreatment. The
crime of slight physical injuries shall be punished:
1. By arresto menor when the offender has inflicted physical
injuries which shall incapacitate the offended party from
labor from one to nine days, or shall require medical
attendance during the same period.
The penalty of arresto menor spans from one (1) day to thirty
31
(30) days. The Indeterminate Sentence Law does not apply
since said law excludes from its coverage cases where the
32
penalty imposed does not exceed one (1) year. With the
aggravating circumstance of treachery, we can sentence
Villacorta with imprisonment anywhere within arresto menor
in the maximum period, i.e., twenty-one (21) to thirty (30)
days. Consequently, we impose upon Villacorta a straight
sentence of thirty (30) days of arresto menor; but given that
Villacorta has been in jail since July 31, 2002 until present
time, already way beyond his imposed sentence, we order his
immediate release.
Under paragraph (1), Article 2219 of the Civil Code, moral
damages may be recovered in a criminal offense resulting in
physical injuries. Moral damages compensate for the mental
anguish, serious anxiety, and moral shock suffered by the
victim and his family as being a proximate result of the
wrongful act. An award requires no proof of pecuniary loss.
Pursuant to previous jurisprudence, an award of Five
Thousand Pesos (P5,000.00) moral damages is appropriate
33
for less serious, as well as slight physical injuries.
WHEREFORE, the Decision dated July 30, 2008 of the Court of
Appeals in CA-G.R. CR.-H.C. No. 02550, affirming the Decision
dated September 22, 2006 of the Regional Trial Court, Branch
170, of Malabon, in Criminal Case No. 27039-MN, is
REVERSED and SET ASIDE. A new judgment is entered finding
Villacorta GUILTY beyond reasonable doubt of the crime of
slight physical injuries, as defined and punished by Article 266
of the Revised Penal Code, and sentenced to suffer the
penalty of thirty (30) days arresto menor. Considering that
Villacorta has been incarcerated well beyond the period of
the penalty herein imposed, the Director of the Bureau of
Prisons is ordered to cause Villacortas immediate release,
unless Villacorta is being lawfully held for another cause, and
to inform this Court, within five (5) days from receipt of this

Decision, of the compliance with such order. Villacorta is


ordered to pay the heirs of the late Danilo Cruz moral
damages in the sum of Five Thousand Pesos (P5,000.00).
SO ORDERED.
TERESITA
J.
Associate Justice

LEONARDO-DE

CASTRO

WE CONCUR:
RENATO
Chief
Chairperson

C.

CORONA
Justice

LUCAS
P.
BERSAMIN MARIANO C. DEL CASTILLO
Associate Justice
Associate Justice
MARTIN
Associate Justice

S.

VILLARAMA,

JR.

Impossible Crimes
People vs. Balmores 85 Phil 493
Facts: Balmores was found guilty of attempted estafa through
falsification of a government obligation. He attempted to
cash in a sweepstakes ticket that was obviously falsified (the
ticket was split into , and the winning ticket number
written in ink at the bottom left part of the halved ticket). He
presented his falsified ticket to a PCSO booth. The PCSO
employee manning the booth saw that the ticket was
obviously falsified, and had Balmores arrested. Balmores
waived the right to counsel, and pleaded guilty to the crime
of attempted estafa.
Issue: WON Balmores committed an impossible crime.
Held: No; The recklessness and clumsiness of the act of
falsification did not make the crime an impossible one under
Paragraph 2 Article 4 of the RPC. The alteration of a losing
sweepstakes ticket would constitute a crime only if an
attempt to cash it were done, which is what occurred in this
case.
Intod v. CA (215 SCRA 52)

CERTIFICATION
Pursuant to Section 13, Article VIII of the Constitution, I
certify that the conclusions in the above Decision had been
reached in consultation before the case was assigned to the
writer of the opinion of the Courts Division.
RENATO C. CORONA

Intent to Kill
Appellant stabbed the victim only once using a sharpened
bamboo stick, hitting him on the left side of the body and
then immediately fled.The instrument used is not as lethal as
those made of metallic material. The part of the body hit is
not delicate in the sense that instant death can ensue by
reason of a single stab wound. The assault was done only
once. Thus, there is doubt as to whether appellant had an
intent to kill the victim, which should be resolved in favor of
the appellant People of the Philippines vs. Orlito Villacorta,
G.R. No. 186412. September 7, 2011).

Facts:
In the morning of February 4, 1979, Intod,
Pangasian, Tubio and Daligdig went to Mandaya's house in
Lopez Jaena, Misamis Occidental and asked him to go with
them to the house of Palangpangan. Thereafter, Mandaya
and Intod, Pangasian, Tubio and Daligdig had a meeting with
Aniceto Dumalagan. He told Mandaya that he wanted
Palangpangan to be killed because of a land dispute between
them and that Mandaya should accompany the four men,
otherwise, he would also be killed.
At about 10:00 o'clock in the evening of the same
day, Petitioner, Mandaya, Pangasian, Tubio and Daligdig, all
armed with firearms, arrived at Palangpangan's house in
Katugasan, Lopez Jaena, Misamis Occidental. At the instance
of his companions, Mandaya pointed the location of
Palangpangan's bedroom. Thereafter, Petitioner, Pangasian,
Tubio and Daligdig fired at said room. It turned out, however,
that Palangpangan was in another City and her home was
then occupied by her son-in-law and his family. No one was in
the room when the accused fired the shots. No one was hit
by the gun fire.
Issue: Whether or not said act constitutes an impossible
crime?
Decision:
Yes. The factual situation in the case at bar present a
physical impossibility which rendered the intended crime
impossible of accomplishment and under Article 4, paragraph
2 of the Revised Penal Code, such is sufficient to make the act
an impossible crime.
To be impossible under this clause, the act intended
by the offender must be by its nature one impossible of
accomplishment. There must be either impossibility of
accomplishing the intended act in order to qualify the act an

impossible crime. Legal impossibility occurs where the


intended acts, even if completed, would not amount to a
crime. Factual impossibility occurs when extraneous
circumstances unknown to the actor or beyond his control
prevent the consummation of the intended crime. The case at
bar belongs to this category.
In our jurisdiction, impossible crimes are recognized.
The impossibility of accomplishing the criminal intent is not
merely a defense, but an act penalized by itself. Furthermore,
the phrase "inherent impossibility" that is found in Article
4(2) of the Revised Penal Code makes no distinction between
factual or physical impossibility and legal impossibility.
To uphold the contention of respondent that the
offense was Attempted Murder because the absence of
Palangpangan was a supervening cause independent of the
actor's will, will render useless the provision in Article 4,
which makes a person criminally liable for an act "which
would be an offense against persons or property, were it not
for the inherent impossibility of its accomplishment . . ." In
that case all circumstances which prevented the
consummation of the offense will be treated as an accident
independent of the actor's will which is an element of
attempted and frustrated felonies.

Jacinto v People Gr. 162540, july 13, 2009


Petitioner had been convicted of qualified theft and is now
seeking for a reversal of the decision.
Facts:
Jacinto along with Valencia and Capitle was charged with
qualified theft for having stole and deposited a check with an
amount of 10,000 php. Such check was issued by Baby Aquino
for payment of her purchases from Mega Foam, but the
check bounced. Dyhengco found out about the theft and filed
a complaint with the NBI. An entrapment operation was
conducted, with the use of marked bills. The entrapment was
a success and the petitioner along with her co-accused was
arrested.
Issue:
Whether this can constitute as an impossible crime and not
as qualified theft
Held:
This constitutes as an impossible crime.
The requisites of an impossible crime are:
1. that the act performed would be an offense against
persons or property (all acts to consummate the crime of
qualified theft was consummated crime against property)
2. that the act was done with evil intent (mere act of unlawful
taking showed intent to gain)
3. that its accomplishment was inherently impossible or the
means employed was either inadequate or ineffectual or
the extraneous circumstance that constituted it as a factual
impossibility (the fact that the check bounced)
Legal impossibility occurs where the intended acts, even if
completed, would not amount to a crime.(Impossibility of
killing a dead person)

Factual impossibility when extraneous circumstances


unknown to the actor or beyond his control prevent
consummation of the intended crime. (Like the example in
the case of Intod: a man puts hishand on the coat pocket of
another with intent to steal but gets nothing since the pocket
is empty)
From the time the petitioner took possession of the check
meant for Mega Foam, she had performed all the acts to
consummate the crime of theft, had it not been impossible of
accomplishment in this case. Replacement for the check was
no longer necessary for the consummation of the crime since
the crime of theft is not a continuing offense, petitioners act
of receiving the cash replacement should not be considered
as a continuation of the theft. The fact that the petitioner was
caught receiving the marked money was merely
corroborating evidence to strengthen proof of her intent to
gain.
Art 6. Stages Of Execution
Valenzuela v. People (G.R. No. 160188)
Facts:
Aristotel Valenzuela and Jovy Calderon were charged
with the crime of theft.
On 19 May1994, Valenzuela and Calderon were seen
outside the Super Sale Club inside the SM Complex along
North Edsa by Lorenzo Lago, SM Security Guard. Valenzuela
was hauling a push cart with cases of Tide detergent
and unloaded these cases of Tide in an open parking
space where Calderon was waiting. Valenzuela went inside
the supermarket again and came back with more cases of
detergent.
Thereafter, Valenzuela left the parking lot and haled
a taxi. He boarded the cab and told the driver to go to the
area where Calderon was waiting. Calderon loaded the cases
of Tide and boarded the taxi. Lago saw all of this and
proceeded to stop the taxi. When Lago asked for a
receipt of the merchandise, the two accused reacted by
fleeing on foot. Valenzuela and Calderon were apprehended
at the scene.
Valenzuela
and
Calderon
were
both
convicted by the trial court of consummated theft .
It was only Valenzuela who filed an appeal with the
Court of Appeals. Petitioner contends that he should only be
convicted of frustrated theft since at the time he was
apprehended, he was never placed in a position to freely
dispose of the articles stolen. The Court of Appeals rejected
this contention, hence, this Petition for Review.
Issue: Whether or not petitioner Valenzuela is guilty only of
frustrated theft?
Decision:
Petition dismissed. Under the statutory definition of
theft, free disposal of the stolen items is not a constitutive
element of theft.
Under Article 308 of the Revised Penal Code, the
crime of theft is defined as follows. Theft is committed by
any person who, with intent to gain but without force or

violence against or intimidation of persons nor force upon


things, shall take the property of another without the latters
consent xxx
On the face of the definition, there is only one
operative act of execution by the actor involved in theft the
taking of personal property of another. It is also clear from
the definition that in order such taking may be qualified as
theft, there must further be present the descriptive
circumstances that the taking was with intent to gain;
without force upon things or violence against or intimidation
of persons; and is was without the consent of the owner of
the property.
For the purpose of ascertaining whether theft is
susceptible of commissions in the frustrated stage, the
question is again, when is the crime of theft produced? There
would be all but certain unanimity in the position that theft is
produced when there is deprivation of personal property due
to to its taking by one with intent to gain. Viewed from that
perspective, it is immaterial to the product of the felony that
the offender, once having committed all the acts of execution
for theft, is able or unable to freely disposed of the property
stolen since the deprivation from the owner alone has
already ensued from such acts of execution.
Indeed the SC, after all, held that unlawful taking is
deemed complete from the moment of the offender gains
possession of the thing even if he has no opportunity to
dispose of the same.
People v Palaganas 501 scra 533
Facts:
Brothers Servillano, Melton and Michael Ferrer were having
their drinking spree at their house but later decided to
proceed to Tidbits Videoke Bar to continue their drinking
spree and to sing. Thereafter, Jaime Palaganas arrived
together with Ferdinand Palaganas(nephew) and Virgilio
Bautista. When Jaime Palaganas was singing, Melton Ferrer
sang with him. Jaime Palaganas got irritated and insulted. He
felt that he was being mocked by Melton Ferrer, that caused
him to went to the Ferrers table and uttered statements
which began the fight. Ferdinand sought help to Rujjeric
Palaganas. They went to the Bar and upon seeing the Ferrers
outside, Ferdinand pointing at the Ferrers instructed Rujjeric
to shoot them. Rujjeric Palaganas shot Servillano, Melton and
Michael with the use of unlicensed firearm. As a result,
Melton was killed, Servillano was fatally wounded and
Michael was shot in his right shoulder.
HELD:
Rivera v People 480 SCRA 188
Victim, Ruben went to a nearby store to buy food. Accused
Rivera was in the same vicinity. When he saw Ruben, Rivera
mocked Ruben for being jobless and dependent on his wife.
This caused an exchange of heated words between the two.
The next day when Ruben and his daughter were once again
buying food, Rivera and two other men attacked Ruben. The
two men punched and mauled Ruben while Rivera, on the

other hand, got a hollowblock and hit Rubenss head with it


three times. Rivera and his companions left only when the
policemen arrived.
Ruben was brought to the hospital and it was said that he
suffered only slight and superficial wounds but were it not for
the arrival of the policemen, Ruben would have died.
TC- the three are guilty of frustrated murder.
CA- affirmed the decision of the trial court, with
modifications.
ISSUE: Whether or not there was INTENT TO KILL.
HELD: Yes. There is intent to kill in the case at bar.
The pieces of evidence required to prove intent to kill are as
follows:
1.means used by the malefactors
2.nature,location and number of wounds sustained by the
victim
3. conduct of the malefactor before, during and after the
commission of the crime,
4.circumstances under which the crime was committed
5. motive of the accused.
Applying the elements to the case at bar, the means or
weapons used by RIVERA is a piece of hollow block.
As to nature, location and number of wounds, Rivera inflicted
injuries on the head of Ruben three times.
Conduct before during and after the commission of the crime,
Rivera was angry at Ruben because of the circumstances that
transpired between Ruben and him
People v Almazan 365scra 373
Henry Almazan unexpectedly arrived and brandished a .38
caliber revolver in front of the group. Almazan's fighting cocks
had just been stolen and he suspected Angel, one of the
spectators, to be the culprit. Thus he said, "manos-manosna
lang tayo,"2 aimed his gun at Angel and pulled the trigger. It
did not fire. He tried again, but again it failed. Henry shot Noli
at the left side of his stomach sending him immediately to the
ground. Henry then turned on Noel and shot him on the left
thigh. Noli died. Noel survived.
HELD: RTC: guilty of murder & frustrated murder.accusedappellant should be held liable for attempted murder , not
frustrated murder . For the charge of frustrated murder to
flourish, the victim should sustain a fatal wound that could
have caused his death were it not for timely medical
assistance. This is not the case before us. The court a quo
anchored its ruling on the statement of Dr. Ticman on crossexamination that the wound of Noel could catch infection or
lead to his death if not timely and properly treated. However,
in his direct testimony, Dr. Ticman declared that the wound
was a mere minor injury for which Noel, after undergoing
treatment, was immediately advised to go home. He even
referred to the wound as a slight physical injury that would
heal within a week and for which the victim was in no danger
of dying. According to jurisprudence,
if the victim was
wounded
with an injury that was not fatal, and could not
cause his death, the crime would only be attempted.

People v. Campuhan (G.R. No. 129433)


Facts:
On April 25, 1996 at around 4pm while Ma. Corazon
Pamintuan was downstairs busy preparing drinks for her two
daughters, she heard Crysthel, one of her daughters crying,
Ayoko, Ayoko prompting her to rush upstairs. Thereupon,
she saw Primo inside her childrens room kneeling in front of
her four-year old daughter, whose pajamas were already
removed, while his short pants were down to his knees.
Primo was apprehended and was charged with
statutory rape.
The trial court found him guilty and sentenced him
to the extreme penalty of death. In convicting the accused,
the trial court relied quite heavily on the testimony of
Corazon that she saw Primo with his short pants down to his
knees kneeling before Crysthel whose pajamas and panty
were supposedly already removed" and that Primo was
forcing his penis into Crysthels vagina.
Issue: Is Primo guilty of Consummated rape?
Decision:
Judgment modified into attempted rape.
In People vs. Dela Pena, the SC clarified that the
decisions finding a case for rape even if the attackers penis
merely touched the external portions of the female genitalia
were made in the context of the presence or existence of an
erect penis capable of full penetration. Where the accused
failed to achieve an erection, had a limp of flaccid penis, or an
oversized penis which could not fit into the victims vagina,
the Court nonetheless held that rape was consummated on
the basis of the victims testimony that the accused
repeatedly tried, but in vain, to insert his penis into her
vagina and in all likelihood reached the labia of her
pudendum as the victim felt his organ on the lips of her vulva,
or that the penis of the accused touched the middle part of
her vagina.
Thus, touching when applied to rape cases does not
simply mean mere epidermal contact, stroking or grazing of
organs, a slight brush or a scrape of the penis on the external
layer of the victim's vagina, or the mons pubis, as in this case.
There must be sufficient and convincing proof that
the penis indeed touched the labias or slid into the female
organ, and not merely stroked the external surface thereof,
for an accused to be convicted of consummated rape. As the
labias, which are required to be touched by the penis, are
by their natural situs or location beneath the mons pubis or
the vaginal surface, to touch them with the penis is to attain
some degree of penetration beneath the surface, hence, the
conclusion that touching the labia majora or the labia minora
of the pudendum constitutes consummated rape.
Judicial depiction of consummated rape has not
been confined to the oft-quoted touching of the female
organ, but has also progressed into being described as the
introduction of the male organ into the labia of the
pudendum, or the bombardment of the drawbridge. But
to the SC's mind, the case at bar merely constitutes a
"shelling of the castle of orgasmic potency," or a "strafing of
the citadel of passion."

Under Art. 6, in relation to Art. 335, of the Revised


Penal Code, rape is attempted when the offender
commences the commission of rape directly by overt acts,
and does not perform all the acts of execution which should
produce the crime of rape by reason of some cause or
accident other than his own spontaneous desistance. All the
elements of attempted rape and only of attempted rape
are present in the instant case; hence, the accused should be
punished only for it.

CONSPIRACY (ART 8)
People v Anticamara 651 scra 489
See full case
People v Baharan 639 scra 157
FACTS:
On 14 February 2005, an RRCG bus was plying its usual
southbound route, from its Navotas bus terminal towards its
Alabang bus terminal via Epifanio de los Santos Avenue
(EDSA). Around 6:30 to 7:30 in the evening, while they were
about to move out of the Guadalupe-EDSA southbound bus
stop, the bus conductor noticed two men running after the
bus. The two insisted on getting on the bus, so the conductor
obliged and let them in.
According to Elmer Andales, the bus conductor, he
immediately became wary of the two men, because, even if
they got on the bus together, the two sat away from each
otherone sat two seats behind the driver, while the other sat
at the back of the bus. At the time, there were only 15
passengers inside the bus. He also noticed that the eyes of
one of the men were reddish. When he approached the
person near the driver and asked him whether he was paying
for two passengers, the latter looked dumb struck by the
question. He then stuttered and said he was paying for two
and gave PhP20. Andales grew more concerned when the
other man seated at the back also paid for both passengers.
At this point, Andales said he became more certain that the
two were up to no good, and that there might be a holdup.
As soon as the bus reached the stoplight at the corner of
Ayala Avenue and EDSA, the two men insisted on getting off
the bus. According to Andales, the bus driver initially did not
want to let them off the bus, because a Makati ordinance
prohibited unloading anywhere except at designated bus
stops. Eventually, the bus driver gave in and allowed the two
passengers to alight. The two immediately got off the bus and
ran towards Ayala Avenue. Moments after, Andales felt an
explosion. He then saw fire quickly engulfing the bus. He ran
out of the bus towards a nearby mall. After a while, he went
back to where the bus was. He saw their bus passengers
either lying on the ground or looking traumatized. A few
hours after, he made a statement before the Makati Police
Station narrating the whole incident. The prosecution
presented documents furnished by the Department of

Justice, confirming that shortly before the explosion, the


spokesperson of the Abu Sayyaf Group Abu Solaiman
announced over radio station DZBB that the group had a
Valentines Day "gift" for former President Gloria MacapagalArroyo. After the bombing, he again went on radio and
warned of more bomb attacks. As stipulated during pretrial,
accused Trinidad gave ABS-CBN News Network an exclusive
interview some time after the incident, confessing his
participation in theValentines Day bombing incident. In
another exclusive interview on the network, accused Baharan
likewise admitted his role in the bombing incident. Finally,
accused Asali gave a television interview, confessing that he
had supplied the explosive devices for the 14 February 2005
bombing. The bus conductor identified the accused Baharan
and Trinidad, and confirmed that they were the two men who
had entered the RRCG bus on the evening of 14 February. As
found by the trial court, Asali, after his training, was required
by the Abu Sayyaf leadership, specifically Abu Solaiman and
Rohmat, to secure eight kilos of TNT, a soldering gun,
aluminum powder, a tester, and Christmas lights, all of which
he knew would be used to make a bomb. He then recalled
that sometime in November to December 2004, Trinidad
asked him for a total of 4 kilos of TNT that is, 2 kilos on two
separate occasions. Rohmat allegedly called Asali to confirm
that Trinidad would get TNT from Asali and use it for their
first mission. The TNT was allegedly placed in two buses
sometime in December 2004, but neither one of them
exploded. Asali then testified that the night before the
Valentines Day bombing, Trinidad and Baharan got another
two kilos of TNT from him. Late in the evening of 14 February,
he received a call from Abu Solaiman. The latter told Asali not
to leave home or go to crowded areas, since the TNT taken by
Baharan and Trinidad had already been exploded in Makati.
Thirty minutes later, Trinidad called Asali, repeating the
warning of Abu Solaiman. The next day, Asali allegedly
received a call from accused Rohmat, congratulating the
former on the success of the mission. According to Asali, Abu
Zaky specifically said, "Sa wakas nag success din yung tinuro
ko sayo."
ISSUE: Whether or not conspiracy is established.
HELD:Yes. The Court also affirms the finding of the existence
of conspiracy involving accused Baharan, Trinidad, and
Rohmat. Conspiracy was clearly established from the
"collective acts of the accused-appellants before, during and
after the commission of the crime." As correctly declared by
the trial court in its Omnibus Decision: Asalis clear and
categorical testimony, which remains unrebutted on its major
points, coupled with the judicial admissions freely and
voluntarily given by the two other accused, are sufficient to
prove the existence of a conspiracy hatched between and
among the four accused, all members of the terrorist group
Abu Sayyaf, to wreak chaos and mayhem in the metropolis by
indiscriminately killing and injuring civilian victims by utilizing
bombs and other similar destructive explosive devices. While
said conspiracy involving the four malefactors has not been
expressly admitted by accused Baharan, Angelo Trinidad, and
Rohmat, more specifically with respect to the latters
participation in the commission of the crimes, nonetheless it

has been established by virtue of the aforementioned


evidence, which established the existence of the conspiracy
itself and the indispensable participation of accused Rohmat
in seeing to it that the conspirators criminal design would be
realized. It is well-established that conspiracy may be inferred
from the acts of the accused, which clearly manifests a
concurrence of wills, a common intent or design to commit a
crime (People v. Lenantud, 352 SCRA 544). Hence, where acts
of the accused collectively and individually demonstrate the
existence of a common design towards the accomplishment
of the same unlawful purpose, conspiracy is evident and all
the perpetrators will be held liable as principals (People v.
Ellado, 353 SCRA 643).
In People v. Geronimo, the Court pronounced that it would
be justified in concluding that the defendants therein were
engaged in a conspiracy "when the defendants by their acts
aimed at the same object, one performing one part and the
other performing another part so as to complete it, with a
view to the attainment of the same object; and their acts,
though apparently independent, were in fact concerted and
cooperative, indicating closeness of personal association,
concerted action and concurrence of sentiments." Accused
contend that the testimony of Asali is inadmissible pursuant
to Sec. 30, Rule 130 of the Rules of Court. It is true that under
the rule, statements made by a conspirator against a coconspirator are admissible only when made during the
existence of the conspiracy. However, as the Court ruled in
People v. Buntag, if the declarant repeats the statement in
court, his extrajudicial confession becomes a judicial
admission, making the testimony admissible as to both
conspirators.
People v Montanir 647 scra 170

See full case


HELD: People of the Philippines v. Dima Montanir,
Ronald Norva and Eduardo Chua, G.R. No. 187534,
April
4, 2011. Liability of conspirators.
When
conspiracy is established, the responsibility of the
conspirators is collective, not individual. This renders all
of them equally liable regardless of the extent of their
respective participations, the act of one being deemed to
be the act of the other or the others, in the commission
of the felony.
People v Quirol 473 scra 509
On December 4, 1993, in celebration of a fiesta in Apas,
Lahug, Cebu City, a benefit disco dance was held at the
local UCMA Village. Appellants, Juanito and Mario Quirol, and
the two victims, Benjamin Silva and Roel Ngujo, attended.
At the dance, Juanito, Mario and Jed were together and
drank all through the night with some friends. The dance
ended just prior to 4 a.m. and prosecution principal witness
Wilson Cruz testified that it was about that time when he was

asked by Benjamin and Roel to accompany them in escorting


some ladies home.

People v Bulan 459 scra 550


PEOPLE vs. BULAN (GR 143404, June 8, 2005, 459 SCRA 550)

Wilson told them to go ahead and that he would just


follow. Wilson was behind them at a distance of 7 to 10
fathoms when the group passed by the house of Jed. From his
vantage point, Wilson saw Jed stop the two victims in front of
his house and frisk them.
Thereafter, Wilson saw Jed bind Benjamin and Roel
together with a pair of handcuffs and lead them towards the
control tower of the old airport of Lahug, Cebu City. There,
the three were met by Juanito and Mario and together they
proceeded to the airport runway.
Wilson, hidden behind a bush, said he could hear
Benjamin plead for his life. A few seconds later, Jed took out
his .38 caliber service revolver and shot Benjamin at pointblank range on the head. As Benjamin fell, Roel was dragged
down to his knees since he was handcuffed to Benjamin.
Mario then held Roel while Juanito started stabbing him using
a Batangas knife. Jed finished it by shooting Roel.
The lower court and Court of Appeals find that there was
conspiracy and convicting them despite their defense of alibi.
Issue: Can there be a conspiracy based on the foregoing
facts?
Decision:
Conspiracy need not be proven by direct evidence of
prior agreement to commit the crime. Neither it is necessary
to show that all the conspirators actually hit and killed the
victim. What has to be shown is that all the participants
performed specific acts with such closeness and coordination
as to unmistakably indicate a common purpose and design.
The conspiracy in the instant case was sufficiently proven by
Jed meeting with appellants at the old airport tower and
walking together with them towards the runway where
appellants and Jed performed acts in unison with each other
as to unmistakably reveal a common purpose and design.
Anent Marios defense of alibi, despite corroboration
from Exequiel Aranas, it is still an inherently weak defense
and cannot prevail over a positive identification from a
witness found credible by the trial court. Absent arbitrariness
or oversight of some fact or circumstance of significance and
influence, we will not interfere with the credence given to the
testimony of Wilson over that of Mario and that of Exequiel,
as assessments of credibility are generally left to the trial
court whose proximate contact with those who take the
witness stand places it in a more competent position to
discriminate between true and false testimony. Moreover, as
correctly discussed by the Court of Appeals, the distance
between the scene of the crimes and where Mario claims he
passed out is not so far away as to prevent him from being
physically present at the place of the crimes or its immediate
vicinity at the time the crimes were committed.

This case was certified by the Court of Appeals (CA) to this


Court for review, in view of its finding in its Decision that
appellants Jose Bulan and his son, Allan Bulan, are guilty of
murder as principals by indispensable cooperation punishable
by reclusion perpetua to death, and not merely as
accomplices as found by the Regional Trial Court (RTC) of
Virac,
Catanduanes,
Branch
42.
On the night of June 6, 1994, a dance was taking place at Brgy
Datag. Alberto Mariano, a barangay tanod in said barangay
was assigned the task of seeing to it that anybody who
entered the gate to the dance hall at the plaza must have a
ribbon. Appellant Allan Bulan came to the dance and entered
the gate without the required ribbon. Alberto Mariano
followed appellant Allan Bulan into the dance hall and asked
him why he entered the gate without a ribbon. Instead of
answering Albertos question, Allan boxed him on the head.
Accused Estemson Bulan, Allans brother, who had entered
the dance hall, likewise, boxed Alberto. Estemson then held
Alberto, while Allan boxed the latter on the chest. Perlita
Mariano, Albertos sister, who was present at the dance,
embraced her brother as Allan and Estemson unceasingly
pummeled him. The other barangay tanods Ceferino Ceballo
and Juan Boribor, and a barangay kagawad, Dante Ereso,
stopped Allan and Estemson from further beating Alberto.
After being pacified by the barangay officials, Allan and
Estemson left the dance hall. Alberto, on the other hand,
went back to where he originally stood to resume his duty.
His sister Perlita stood beside him. One Edwin Solo, a
policeman, suddenly came into the dance hall and dragged
Alberto into the street just outside the entrance. Perlita
embraced Alberto as he was dragged outside the barangay
plaza. Appellants Jose Bulan and Allan Bulan were waiting for
Alberto and immediately held the latter by his shoulders. Jose
held Albertos right shoulder while Allan held his left
shoulder. Perlita was still embracing her brother but she was
pulled away from. Accused Estemson Bulan suddenly
appeared behind Alberto and stabbed him twice in the back
with a small bolo. Perlita screamed for help. However,
despite the fact that there were people at the entrance gate,
nobody came to help Alberto and Perlita. After stabbing
Alberto, Estemson immediately escaped, while Jose and Allan
dragged the fatally wounded Alberto away from the barangay
plaza to the store of Valentin Talion, which was forty meters
away from the scene of the stabbing. They dropped Alberto
face down on the ground in front of Valentins store and then
left, running towards the direction of [the] barangay plaza.
Perlita, who followed Jose and Allan as they dragged her
brother, kept on shouting for help but nobody came to help
them. After Jose and Allan left, Perlita returned to the
barangay plaza and sought help from her relatives who were
at the dance. She, likewise, sought the help of the barangay
officials present and informed them that Alberto was already
dead. Minutes later, Nelson Rubio, a policeman, went to the
place where Alberto lay. When the policeman tried to lift

Alberto, the small bolo which was used in stabbing Alberto


and which remained embedded in his back, fell to the
ground.
ISSUE: Whether or not the appellants are guilty of the crime
charged as principals by direct participation as ruled by the
CA, or, as ruled by the RTC, mere accomplices to the crime of
murder.
RULING: Article 8 of the Revised Penal Code provides that
there is conspiracy when two or more persons agree to
commit a crime and decide to commit it. Direct proof is not
essential to prove conspiracy; it may be established by acts of
the accused before, during and after the commission of the
crime charged, from which it may be logically inferred the
existence of a common purpose to commit the same. The
prosecution must prove conspiracy by the same quantum of
evidence as the felony charged itself. Indeed, proof of
previous agreement among the malefactors to commit the
crime is not essential to prove conspiracy. It is not necessary
to show that all the conspirators actually hit and killed the
victim; what is primordial is that all the participants
performed specific acts with such closeness and coordination
as to indicate a common purpose or design to bring out the
victims death. Once conspiracy is established, it is
unnecessary to prove who among the conspirators inflicted
the fatal injury. If conspiracy is proved, all the conspirators
are criminally liable for the crime charged and proved. The
act
of
one
is
the
act
of
all.
In this case, the appellants were waiting outside the dance
hall near the gate when Edwin Solo brought the victim
towards them, onto the street. Jose held the victim by the
right shoulder, while Allan held him by the left. Estemson
suddenly appeared from behind the victim and stabbed the
latter at the back with a small bolo. The appellants continued
holding the victim as Estemson stabbed him yet again. Even
as Estemson fled, the appellants dragged the victim from the
gate, towards the store, where they dropped the victims
body and fled from the scene. Allan then left Catanduanes
and hid in Pasay City where he was arrested by the NBI on
August
7,
1994.
Considering the foregoing, the Court affirms the finding of the
CA that the appellants are guilty as principals by direct
participation in the killing of Alberto Mariano.

Danilo Lozano walking. The three stopped in front of the


house. While his companions looked on, Antonio suddenly
lobbed an object which fell on the roof of the terrace.
Appellants immediately fled by scaling the fence of a nearby
school.
The object, which turned out to be a hand grenade,
exploded ripping a hole in the roof of the house. Robert
Agbanlog, Jimmy Wabe, Gerry Bullanday, Rey Camat and
Lorenzo Eugenio were hit by shrapnel and slumped
unconscious on the floor.
The undisputed facts show that when Antonio Comadre
was in the act of throwing the hand grenade, George
Comadre and Danilo Lozano merely looked on without
uttering a single word of encouragement or performed any
act to assist him. The trial court held that the mere presence
of George Comadre and Danilo Lozano provided
encouragement and a sense of security to Antonio Comadre,
thus proving the existence of conspiracy.
Issue: Can there be a conspiracy based on the foregoing
facts?
Decision:
Similar to the physical act constituting the crime itself,
the elements of conspiracy must be proven beyond
reasonable doubt. Settled is the rule that to establish
conspiracy, evidence of actual cooperation rather than mere
cognizance or approval of an illegal act is required.
A conspiracy must be established by positive and
conclusive evidence. It must be shown to exist as clearly and
convincingly as the commission of the crime itself. Mere
presence of a person at the scene of the crime does not make
him a conspirator for conspiracy transcends companionship.
The evidence shows that George Comadre and Danilo
Lozano did not have any participation in the commission of
the crime and must therefore be set free. Their mere
presence at the scene of the crime as well as their close
relationship with Antonio are insufficient to establish
conspiracy considering that they performed no positive act in
furtherance of the crime.
People v Ramos 427 scra 299
People v Pacificado 376 scra 180

People v Comadre 431 scra 366


At around 7:00 in the evening of August 6, 1995, Robert
Agbanlog, Jimmy Wabe, Gerry Bullanday, Rey Camat and
Lorenzo Eugenio were having a drinking spree on the terrace
of the house of Roberts father, Barangay Councilman Jaime
Agbanlog, situated in Barangay San Pedro, Lupao, Nueva
Ecija. Jaime Agbanlog was seated on the banister of the
terrace listening to the conversation of the companions of his
son.
As the drinking session went on, Robert and the others
noticed appellants Antonio Comadre, George Comadre and

Art. 11: Justifying Circumstances


Self-defense
Urbano v. People (G.R. No. 182750)
Facts: Petitioner Urbano testified being, in the afternoon of
September 28, 1993, in the nearby town of Bugallon for a
picnic. He was with Tomelden and several others, including
Dominador Navarro, Chairperson of Lingayen Water District.
At a restaurant in Bugallon, the group ordered goats meat
and drank beer. When it was time to depart, Navarro asked

petitioner to inform Tomelden, then seated in another table,


to prepare to leave.
When so informed, Tomelden insulted petitioner,
telling the latter he had no business stopping him from
further drinking as he was paying for his share of the bill.
Chastised, petitioner returned to his table to report to
Navarro. At that time, petitioner saw that Tomelden had
already consumed 17 bottles of beer. In all, the group stayed
at the picnic place for three and a half hours before returning
to the LIWAD. Upon reaching the LIWAD compound,
Tomelden allegedly slapped and hurled insults at him, calling
him "sipsip" just to maintain his employment as Navarros
tricycle driver. Tomelden allegedly then delivered several fist
and kick blows at petitioner, a couple of which hit him despite
his evasive actions. Petitioner maintained that he only boxed
the victim in retaliation, landing that lucky punch in the
course of parrying the latters blows.
Thereafter, Tomelden went to the hospital several
times complaining of dizziness, headache, and other pains.
The last time he went to the hospital, things turned for the
worst. Tomelden died due, per Dr. Arellano, to "cardiorespiratory arrest secondary to cerebral concussion with
resultant cerebral hemorrhage due to mauling incident."
Orje Salazar, their co-worker, attests to the
provocative acts of Tomelden and to his being the aggressor.
Issue: Whether or not the victims insulting remarks directed
at the accused, and uttered immediately before the fist fight
constitute sufficient provocation?
Decision: When the law speaks of provocation either as a
mitigating circumstance or as an essential element of selfdefense, the reference is to an unjust or improper conduct of
the offended party capable of exciting, inciting, or irritating
anyone; it is not enough that the provocative act be
unreasonable or annoying; the provocation must be sufficient
to excite one to commit the wrongful act and should
immediately precede the act. This third requisite of selfdefense is present: (1) when no provocation at all was given
to the aggressor; (2) when, even if provocation was given, it
was not sufficient; (3) when even if the provocation was
sufficient, it was not given by the person defending himself;
or (4) when even if a provocation was given by the person
defending himself, it was not proximate and immediate to
the act of aggression.
In the instant case, Tomeldens insulting remarks
directed at petitioner and uttered immediately before the fist
fight constituted sufficient provocation. This is not to mention
other irritating statements made by the deceased while they
were having beer in Bugallon. Petitioner was the one
provoked and challenged to a fist fight.
People v. Sanchez (G.R. No. 161007)
Facts:
Sanchez's account of the facts shows that he and
Jamero were tenants of adjacent lots located in San Jose,
Mahayag, Zamboanga del Sur. At about 7:00 o'clock in the
morning of September 4, 1993, Sanchez saw Jamero
destroying the dike which served as the boundary between

the two lots. Sanchez confronted Jamero and told the latter
that he was encroaching on his land. Jamero struck him with
a shovel. The shovel got stuck in the mud so Jamero resorted
to throwing mud at Sanchez. Fighting back, Sanchez hacked
Jamero with a bolo, resulting in the latter's death. Sanchez
then proceeded to the municipal building to surrender upon
the advice of his son-in-law.
According to the OSG, Jamero's attack on Sanchez
was unsuccessful because the latter was able to evade it and
Jamero's shovel got stuck in the mud. Jamero fled toward the
ricefield when Sanchez unsheathed his bolo. Sanchez pursued
him and struck his head with a bolo. Jamero fell down but
was able to stand up again. He ran away but after a short
distance, fell down again. Sanchez approached him and
stabbed him several times. Not satisfied, Sanchez pushed
Jamero's face down into the knee-deep mud. After Jamero's
aggression ceased when he fled and left his shovel stuck in
the mud, there was no longer any justification for Sanchez to
go after him and hack him to death.
Issue: Whether or not unlawful aggression, if not continuous,
does not constitute aggression warranting self-defense?
Decision: There can be no self-defense, complete or
incomplete, unless the accused proves the first essential
requisiteunlawful aggression on the part of the victim.
Unlawful aggression presupposes an actual, sudden and
unexpected or imminent danger on the life and limb of a
person a mere threatening or intimidating attitude is not
sufficient. There must be actual physical force or a threat to
inflict physical injury. In case of a threat, it must be offensive
and positively strong so as to display a real, not imagined,
intent to cause injury. Aggression, if not continuous, does not
constitute aggression warranting self-defense.
In this case, the twin circumstances of Jamero's
shovel getting stuck in the mud and his running away from
Sanchez convincingly indicate that there was no longer any
danger to the latter's life and limb which could have justified
his pursuit of Jamero and subsequent hacking and killing of
the latter.
Sanchez's failure to prove unlawful aggression by
Jamero and the prosecution's evidence conclusively showing
that it was Sanchez who was the unlawful aggressor
completely discounts Sanchez's claim of self-defense. Even
incomplete self-defense by its very nature and essence would
always require the attendance of unlawful aggression
initiated by the victim which must clearly be shown.

Battered Woman Syndrome (sec 26. RA 9262)


People V Genosa 419 scra 537
In 1998, Marivic Genosa was sentenced to death by the RTCOrmoc City, Branch 35, for parricide for killing her husband.
She claimed that she had been physically and verbally abused
by her husband. In 2000, the Supreme Court remanded the
case to the RTC for her to prove her psychological and mental
state when the crime was committed considering her defense
of BWS. At the trial court, she was examined by 2 clinical

psychologists whose testimonies and documentary evidence


were submitted to the court.
HELD: A battered woman is defined as one who is repeatedly
subjected to any forceful physical or psychological behavior
by man, in order to coerce her to do something he wants her
to do without concern for her rights. Battered women include
wives or women who are in any form of intimate relationship
with men. Furthermore, to be classified as a battered woman,
the couple must have been through the battering cycle at
least twice. Any woman may find herself in an abusive
relationship with a man once. If it occurs a second time, and
she remains in that same situation, she is deemed a battered
woman.
The existence of BWS in a relationship does not in itself
establish the legal right of a woman to kill her abusive
partner. Evidence must still be considered in the context of
self-defense. The 3 main points of self-defense arising from
BWS are summed up as follows:
(1) Each of the phases of the cycle of violence must be proven
to have characterized at least 2 battering episodes between
the woman and her intimate partner;
(2) The final acute battering episode preceding the killing of
the batterer must have produced in the battered womans
mind an actual fear of an imminent harm from her batterer
and an honest belief that she needed to use force in order to
save her life; and
(3) At the time of the killing, the batterer must have posed
probable grave harm to the battered woman, based on the
history of violence perpetrated by the former against the
latter.
In the abovecited case, the defense failed to elicit from her
factual experience and thoughts that would clearly and fully
demonstrate the essential characteristics of BWS. There was
no unlawful aggression, nor an immediate and unexpected
attack on her by her batterer husband at the time she killed
him.
Nonetheless, the cyclical nature and the severity of the
violence inflicted upon the wife, Marivic Genosa, resulted in
cumulative provocation which broke down her psychological
resistance and natural self-control, psychological paralysis,
and difficulty in concentrating or impairment of memory.
Such manifestations were analogous to an illness that
diminished the exercise by the battered wife of her will
power without, however, depriving her of her consciousness
of her act, which is a mitigating circumstance under the
Revised Penal Code. Also found in her favor is the extenuating
circumstance of having acted upon an impulse so powerful as
to have naturally produced passion and obfuscation. The
killing was preceded by abusive and violent acts directed
against the battered wife, who was then 8 moths pregnant,
by her husband.
Marivic Genosas death sentence was reduced to an
indeterminate prison term of 6 years and 1 day of prision

mayor as minimum, to 14 years, 8 months, and 1 day of


reclusion temporal, as maximum, with the recognition of 2
mitigating circumstances and no aggravating circumstance.
The Court further held that she may be released immediately
from custody since she had been detained for more than the
minimum penalty imposed upon her, unless she is being held
for some other lawful cause.
Defense of Relatives
Balunueco v. CA and People (G.R. No. 126968)
Facts:
On May 2, 1982 at around 6:00 oclock in the
evening Amelia Iguico saw accused Reynaldo, his father
Juanito and brothers Ricardo and Ramon, all surnamed
Balunueco, and one Armando Flores chasing her brother-inlaw Servando Iguico. With the five (5) individuals in hot
pursuit, Servando scampered into the safety of Amelias
house. Meanwhile Senando went out of the house fully
unaware of the commotion going on outside. Upon seeing
Senando, Reynaldo turned his attention on him and gave
chase. Senando instinctively fled towards the fields but
Reynaldo, Ricardo, and Armando cornered him and ganged
up on him. To shield him from further violence, Amelia put
her arms around her husband but it was not enough to
detract Ricardo from his murderous frenzy. Amelia was also
hit on the leg.
In his defense, accused Ricardo invoke defense of
relatives. He testified that at that time he was fetching water
he heard somebody shout. When he hurried to the place, he
saw his brother Ramon embracing Senando who was
continuously hacking Reynaldo. Thereafter, Senando shoved
Ramon to the ground and as if further enraged by the
intrusion, he turned his bolo on the fallen Ramon. Ricardo
screamed, "tama na yan, mga kapatid ko yan." But the
assailant would not be pacified as he hacked Ramon on the
chest. At this point, Servando, the brother of Senando, threw
an axe at him but Reynaldo picked it up and smashed
Senando with it.
The trial court found the accused guilty of homicide
and frustrated homicide. According to the trial court, the
denial of Ricardo was self-serving and calculated to extricate
himself from the predicament he was in. Further, the trial
court added that the wounds allegedly received by Ricardo in
the hands of the victim, Senando Iguico, if at all there were
any, did not prove that Senando was the aggressor for the
wounds were inflicted while Senando was in the act of
defending himself from the aggression of Ricardo and his coconspirators.
The Court of Appeals sustained the conviction of
accused Ricardo with modification that his conviction for the
wounding of Amelia Iguico, should be for attempted homicide
only.
Issue: Whether or not there was a valid defense of relatives?
Decision:
Petitioner invokes the justifying circumstance of
defense of relatives under Art. 11, par. (2), of The Revised

Penal Code. The essential elements of this justifying


circumstance are the following: (a) unlawful aggression; (b)
reasonable necessity of the means employed to prevent or
repel it; and, (c) in case the provocation was given by the
person attacked, the one making the defense had no part
therein.
Of the three (3) requisites of defense of relatives,
unlawful aggression is a condition sine qua non, for without it
any defense is not possible or justified. In order to consider
that an unlawful aggression was actually committed, it is
necessary that an attack or material aggression, an offensive
act positively determining the intent of the aggressor to
cause an injury shall have been made; a mere threatening or
intimidating attitude is not sufficient to justify the
commission of an act which is punishable per se, and allow a
claim of exemption from liability on the ground that it was
committed in self-defense or defense of a relative. It has
always been so recognized in the decisions of the courts, in
accordance with the provisions of the Penal Code.
Having admitted the killing of the victim, petitioner has the
burden of proving these elements by clear and convincing
evidence. He must rely on the strength of his own evidence
and not on the weakness of that of the prosecution, for even
if the prosecution evidence is weak it cannot be disbelieved if
the accused has admitted the killing.
In the case at bar, petitioner Ricardo utterly failed to
adduce sufficient proof of the existence of a positively strong
act of real aggression on the part of the deceased Senando.
With the exception of his self-serving allegations, there is
nothing on record that would justify his killing of Senando.

Intestate Estate of Gonzales Vda. De Carungcong V People


GR. 181409 Feb 11, 2011
SEE FULL CASE..
People V Eduarte 187 scra 291
FACTS:

The evidence of prosecution establishes that


Fredeswindo Eduarte, suspecting that his wife was having an
illicit affair with another man, got a scissor. Upon seeing his
intention, Roberto (Fredeswindos brother-in-law) tried to
pacify him. However, instead of being appeased, Fredeswindo
thrusted the scissor to Roberto. Thereafter, Roberto drove his
jeep to seek the help of policemen. On the way back, he saw
Fredeswindo lying flat on the road so he stopped and alighted
to help. Just when Roberto was in the act of extending
assistance, Florentino Eduarte, Fredeswindos brother, shot
him, which caused his death. Not long after, the police
authorities arrived and the place and looked for Florentino,
but failed to locate him.

On the other hand, the defense, in its counterstatement of facts, relates that when Fredeswindo

confronted his wife, Roberto butted in and berated him


which resulted to an exchange of words leading to
altercation. Fredeswindo defended himself by grasping a
scissor from his back and thrusting it against his assailant.
When Robertos wife saw this, she sought the help of
Robertos three brothers and they ganged up on
Fredeswindo. Julie Eduarte, Fredewindos brother, saw this
and thereafter sought the help of their other brother,
Florentino Eduarte. When Florentino arrived at the place of
incident, he saw Roberto clubbing his brother who was lying,
face downward and his shirt soaked with blood.

The accused-appellant maintains that he is innocent


of the crime as charged invoking the justifying circumstance
of defense of relatives. Instead of making an assignment of
errors, the accused-appellant states that the trial court was
confronted with two conflicting versions, one asserting that
Roberto Trinidad was shot while assisting Fredeswindo
Eduarte and the other stating that Roberto was shot while
clubbing Fredeswindo. In both cases, the appellant states that
defense of relative should be appreciated.
ISSUE: Whether the defense of relatives under Article II,
subparagraph (2) of the Revised Penal Code can be invoked
by the accused-appellant in his favour.
RULE: Article II, subparagraph 2 of the Revised Penal Code
provides that:
2. Anyone who acts in defense of the person or rights of his
spouse, ascendants, descendants, or legitimate, natural or
adopted brothers or sisters, or of his relatives by affinity in
the same degrees and those by consanguinity within the
fourth civil degree, provided that the first and second
requisites prescribed in the next preceding circumstances are
present, and the further requisite, in case the provocation
was given by the person attacked, that the one making
defense had no part therein.
The first and second requisites are unlawful aggression and
reasonable necessity of the means employed to prevent or
repel the unlawful aggression. Hence, it was incumbent upon
the accused-appellant to prove the existence of the three
essential requisites of the justifying circumstance of defense
of relatives namely: (1) unlawful aggression; (2) reasonable
necessity of the means employed to prevent or repel it; and
(3) in case the provocation was given by the person attacked,
that the one making the defense had no part therein.
APPLICATION:
The accused-appellant claims that there was unlawful
aggression on the part of Roberto Trinidad considering that
the scenario he saw after he was summoned from his place of
work by his other brother Julie was that of his allegedly
assaulted brother Fredeswindo sprawled on the ground and
bathed in his own blood with Roberto Trinidad clubbing

Fredeswindo. It was when Roberto was about to deliver the


final blow to Fredeswindo that the accused-appellant claimed
he shot Roberto in defense of his brother. Thus, according to
the accused-appellant, the second requisite is also present
considering that the use of a gun at that pressing moment
was reasonable and necessary to prevent or repel the
aforestated unlawful aggression. As regards the third
requisite, the accused-appellant contends that there is no
debate as to its presence since the accused-appellant was
clearly not a part of the melee in question.
CONCLUSION:
We find no merit in the claim that the shooting of Roberto
was done in defense of a relative. For this justifying
circumstance to prosper, the evidence adduced must be
persuasive. Although it is true that the accused-appellant
took no part in the provocation that led to the killing incident,
his testimony that there was unlawful aggression on the part
of Roberto was self-serving and uncorroborated. Hence, for
lack of a clear unlawful aggression on the part of the victim
Roberto and of the reasonable necessity of the means
employed by the accused-appellant, the justifying
circumstance of defense of relative cannot be availed of.
Accused-appellant Florentino Eduarte is found guilty beyond
reasonable doubt of the crime of HOMICIDE without any
aggravating or mitigating circumstance.

Defense Of Strangers
People V DiJan 383 scra 15
People V Toring 191 scra 38
Facts: A benefit dance was held in one sitio in Lapu lapu City
for the last canvassing of votes for the candidates for
princesses, attended by the entire family of one of the
candidates. Also present were members of the kwaknit gang,
headed by Toring, noted for their bird-like way of dancing and
their propensity for drunkenness and provoking trouble.
Samuel, the father of the declared winner, while answering
the call of nature, was approached by Toring and two others
and was stabbed from behind by Toring using a knife handed
to him by a companion. Samuel died of stab wound he
sustained on the right side of his abdomen.
An information for MURDER was filed against Toring. The
lower court rendered a decision discrediting Toring's claim
that the killing of Samuel was justified because it was done in
defense of a stranger. While Toring testified that Samuel was
aiming his shotgun at the chest of Ely Amyon (Amion),
prosecution witness Joel Escobia claimed that he was at the
receiving end of Samuel's thrusts with the butt of his shotgun.
To the court, such discrepancy is fatal to the defense because
in appreciating the justifying circumstance of defense of a

stranger, the court must know "with definiteness the identity


of the stranger defended by the accused.
Upon appeal, Toring seeks his exoneration by contending that
his assault on Samuel was justified because he acted in
defense of his first cousin, Joel Escobia is the first cousin of
Toring their fathers being brothers, although no explanation
appears on record why they have different surnames. At any
rate, this allegation on relationship was not rebutted by the
prosecution. Escobia attested that as he was about to dance
with a girl, Samuel stopped him, pointed his shotgun at him,
took a bullet from his jacket pocket, showed it to Escobia.
Samuel pointed the shotgun at his chin and told him to eat
the bullet.
Issue: Whether the act of Toring in stabbing Samuel was
justified for being done in defense of his relative, Joel
Escobia.
Held: NO. SC ruled that there was no reason to doubt Joel
Escobia's assertion of Samuel's unlawful aggression and that
prosecution failed to prove that Joel testified to favor Toring.
However, the presence of unlawful aggression on the part of
the victim and the lack of proof of provocation on the part of
Toring notwithstanding, full credence cannot be given, to
Toring's claim of defense of a relative.
Toring himself admitted in court that in 1979, he was shot
with a .22 caliber revolver by Edgar Augusto, Samuel's
brother. It cannot be said, therefore, that in attacking
Samuel, Toring was impelled by pure compassion or
beneficence or the lawful desire to avenge the immediate
wrong inflicted on his cousin. Rather, he was motivated by
revenge, resentment or evil motive because of a "running
feud" between the Augusto and the Toring brothers. Indeed,
vendetta appears to have driven both camps to commit
unlawful acts against each other
State of Necessity
Ty V People 439 scra 220
Ty vs People439 SCRA 220G.R. No. 149275 September 27,
2004Vicky C. Ty, petitioner People of the Philippines,
Respondent
Facts: This case originated from the filing of 7 Information for
violation of B.P. 22against the petitioner Vicky C. Ty who
drew and issued to Manila Doctors Hospital seven (7) postdated checks amounting to P30,000.00 each which was
subsequently dishonored by the drawee bank for "Account
Closed" and despite receipt of notice of such dishonor, she
failed to pay the Hospital within five (5) banking days after
receiving said notice. The trial court as affirmed by the CA
with modification convicted Ms. Ty guilty of seven (7)counts
of violation of B.P 22 . In her appeal, she invoked the defense
that she issued the checks "under the impulse of an

uncontrollable fear of a greater injury or in avoidance of a


greater evil or injury."
Issue: Whether or not the defense of uncontrollable fear is
acceptable to warrant an exemption from criminal liability?
Held: No. For Uncontrollable fear to be appreciated as
exempting circumstance, the following requisites must
concur: (1) existence of an uncontrollable fear; (2)the fear
must be real and imminent; and (3) the fear of an injury is
greater than or at least equal to that committed. It must
appear that the threat that caused the uncontrollable fear is
of such gravity and imminence that the ordinary man would
have succumbed to it. In this case, the fear harbored by Ms.
Ty was not real and imminent. She claimed that she was
compelled to issue the checks, a condition the hospital
allegedly demanded of her before her mother could be
discharged, for fear that her mother's health might
deteriorate further due to the inhumane treatment of the
hospital or worse, her mother might commit suicide. This is
speculative fear; it is not the uncontrollable fear
contemplated by law. Hence, the decision was affirmed.

Held:
No. Juan Padernals reliance on the justifying circumstance is
erroneous because his act in preventing Marianito from
shooting Ricohermoso and Severo Padernal, the aggressors in
this case, was designed to insure the killing of Geminiano de
Leon without any risk to the assailants and not an act to
prevent infliction of greater evil or injury. His intention was to
forestall any interference in the assault.
Treachery was also appreciated in the case. The trial court
convicted the appellants with lesiones leves, from an
attempted murder charge with respect to Marianito de Leon.
Judgment as to Juan Padernal affirmed.
(Note: Severo Padernal withdrew his appeal, thus, in effect,
accepted the prosecutions version of the case and trial
courts finding of guilt)
Fulfilment of a Duty
Ambril Jr. V Sandiganbayan 653 scra 576

People V Recohermoso 56 scra 431


Mamangun v. People (GR 149152)
Avoidance of Greater Evil or Injury
Facts:Geminiano de Leon, together with his common-law
wife, son Marianito de Leon and one Rizal Rosales, chanced
upon Pio Ricohermoso. Owning a parcel of land, which
Ricohermoso cultivated as kaingin, Geminiano asked about
his share of palay harvest and added that she should be
allowed to taste the palay harvested from his land.
Ricohermoso said Geminiano could collect the palay anytime.
Upon returning from his trip to Barrio Bagobasin, Geminiano
dropped by Ricohermosos house and asked him about the
palay, to which the latter answered defiantly that he will not
give him the palay, whatever happens. Geminiano
remonstrated and that point (as if by prearrangement),
Ricohermoso unsheathed his bolo, while his father-in-law
Severo Padernal got an axe, and attacked Geminiano. At the
same time and place, Ricohermosos brother-in-law Juan
Padernal suddenly embraced Marianito. They grappled and
rolled down the hill, at which point Marianito passed out.
When he regained consciousness, he discovered that the rifle
he carried beforehand was gone and that his father was
mortally wounded.
The defendants shifted the responsibility of killing in their
version of the case.
Issue: W/N appellant Juan Padernal can invoke the justifying
circumstance of avoidance of a greater evil or injury

Facts:
On July 31, 1992, at about 8:00 in the evening, a
certain
Liberty
Contreras
was
heard
shouting,
MagnanakawMagnanakaw. Several residents responded
and thereupon chased the suspect who entered the yard of
Antonio Abacan and proceeded to the rooftop of Abacans
house.
At about 9:00 oclock that same evening, the desk
officer of the Meycauayan PNP Police Station, upon receiving
a telephone call that a robbery-holdup was in progress in
Brgy. Calvario, immediately contacted and dispatched to the
scene the crew including herein petitioner PO2 Rufino S.
Mamangun. With the permission of Abacan, petitioner
Mamangun, and two others went to the rooftop of the house
whereat the suspect was allegedly taking refuge.
The three policemen, each armed with a drawn
handgun, searched the rooftop. There, they saw a man whom
they thought was the robbery suspect. At that instance,
petitioner Mamangun, who was walking ahead of the group,
fired his handgun once, hitting the man. The man turned out
to be Gener Contreras (Contreras) who was not the robbery
suspect.
Contreras died from the gunshot wound.
Issue: Whether or not the shooting in question was done in
the performance of a duty or in the lawful exercise of a right
or office?
Decision: No. The justifying circumstance of fulfillment of
duty under paragraph 5, Article II, of the Revised Penal Code
may be invoked only after the defense successfully proves
that: (1) the accused acted in the performance of a duty; and
(2) the injury inflicted or offense committed is the necessary
consequence of the due performance or lawful exercise of
such duty.

Concededly, the first requisite is present in this case.


Petitioner, a police officer, was responding to a robberyholdup incident. His presence at the situs of the crime was in
accordance with the performance of his duty. However, proof
that the shooting and ultimate death of Contreras was a
necessary consequence of the due performance of his duty as
a policeman is essential to exempt him from criminal liability.
To be sure, acts in the fulfillment of a duty, without
more, do not completely justify the petitioners firing the
fatal gunshot at the victim. True, petitioner, as one of the
policemen responding to a reported robbery then in progress,
was performing his duty as a police officer as well as when he
was trying to effect the arrest of the suspected robber and in
the process, fatally shoot said suspect, albeit the wrong man.
However, in the absence of the equally necessary justifying
circumstance that the injury or offense committed be the
necessary consequence of the due performance of such duty,
there can only be incomplete justification, a privileged
mitigating circumstance under Articles 13 and 69 of the
Revised Penal Code.
Baxinela v. People (G.R. No. 149652)
Facts: Petitioner SPO2 Eduardo L. Baxinela was in a pub
drinking with two other policemen in as early as 11:00 p.m. of
October 18, 1996. At around 12:00 a.m. to 12:30 a.m. there
was a minor altercation between the deceased Sgt. Lajo and
another customer at the pub but eventually the two were
able to patch things up. While on his way out, Lajo was
followed by Braxinela with a gun already drawn out. From
behind, Baxinela held Lajos left arm and asked why he was
carrying a gun. Thereafter an explosion coming from
Baxinelas gun was heard. Lajo, still standing, took two steps
and
then
fell
down.
Issue: Whether or not fulfilment of duty may validly be
invoked by the petitioner?
Decision: No. In order to avail of this justifying circumstance
it must be shown that: 1) the accused acted in the
performance of a duty or in the lawful exercise of a right or
office; and 2) the injury caused or the offense committed is
the necessary consequence of the due performance of duty
or the lawful exercise of a right or office. While the first
condition is present, the second is clearly lacking. Baxinelas
duty was to investigate the reason why Lajo had a gun tucked
behind his waist in a public place. This was what Baxinela was
doing when he confronted Lajo at the entrance, but perhaps
through anxiety, edginess or the desire to take no chances,
Baxinela exceeded his duty by firing upon Lajo who was not at
all resisting. The shooting of Lajo cannot be considered due
performance of a duty if at that time Lajo posed no serious
threat or harm to Baxinela or to the civilians in the pub. The
Court will, however, attribute to Baxinela the incomplete
defense of fulfillment of a duty as a privileged mitigating
circumstance. In Lacanilao v. Court of Appeals, it was held
that if the first condition is fulfilled but the second is wanting,
Article 69 of the Revised Penal Code is applicable so that the

penalty lower than one or two degrees than that prescribed


by law shall be imposed.

Obedience to an Order
Tabuena v. Sandiganbayan (G.R. Nos. 103501-03)
Facts:
Luis A. Tabuena and Adolfo M. Peralta were
convicted by the Sandiganbayan of malversation under
Article 217 of the Revised Penal Code in the total amount of
P55 Million of the Manila International Airport Authority
(MIAA) funds during their incumbency as General Manager
and Acting Finance Services Manager, respectively, of MIAA.
Then President Marcos instructed Tabuena over the
phone to pay directly to the president's office and in cash
what the MIAA owes the Philippine National Construction
Corporation (PNCC), to which Tabuena replied, "Yes, sir, I will
do it." About a week later, Tabuena received from Mrs. Fe
Roa-Gimenez, then private secretary of Marcos, a Presidential
Memorandum dated January 8, 1986 reiterating in black and
white such verbal instruction, directed to pay immediately
the Philippine National Construction Corporation, thru this
Office, the sum of FIFTY FIVE MILLION (P55,000,000.00)
PESOS in cash as partial payment of MIAA's account with said
Company signed by the then President Marcos.
In obedience to President Marcos' verbal instruction
and memorandum, Tabuena, with the help of Dabao and
Peralta, caused the release of P55 Million of MIAA funds
The disbursement of the P55 Million was, as
described by Tabuena and Peralta themselves, "out of the
ordinary" and "not based on the normal procedure". Not only
were there no vouchers prepared to support the
disbursement, the P55 Million was paid in cold cash. Also, no
PNCC receipt for the P55 Million was presented.
Issue: Whether or not the petitioners defense of good faith is
tenable?
Decision:
Yes. It is settled that this is a valid defense in a
prosecution for malversation for it would negate criminal
intent on the part of the accused. To constitute a crime, the
act must, except in certain crimes made such by statute, be
accompanied by a criminal intent, or by such negligence or
indifference to duty or to consequences as, in law, is
equivalent to criminal intent. The maxim is actus non facit
reum, nisi mens sit rea a crime is not committed if the
mind of the person performing the act complained of is
innocent. Ordinarily, evil intent must unite with an unlawful
act for there to be a crime. Actus non facit reum, nisi mens sit
rea. There can be no crime when the criminal mind is
wanting.
Tabuena had no other choice but to make the
withdrawals, for that was what the MARCOS Memorandum
required him to do. He could not be faulted if he had to obey
and strictly comply with the presidential directive, and to

argue otherwise is something easier said than done. Marcos


was undeniably Tabuena's superior the former being then
the President of the Republic who unquestionably exercised
control over government agencies such as the MIAA and
PNCC. In other words, Marcos had a say in matters involving
inter-government agency affairs and transactions, such as for
instance, directing payment of liability of one entity to
another and the manner in which it should be carried out.
And as a recipient of such kind of a directive coming from the
highest official of the land no less, good faith should be read
on Tabuena's compliance, without hesitation nor any
question, with the MARCOS Memorandum. Tabuena
therefore is entitled to the justifying circumstance of "Any
person who acts in obedience to an order issued by a
superior for some lawful purpose." The subordinate-superior
relationship between Tabuena and Marcos is clear. And so
too, is the lawfulness of the order contained in the MARCOS
Memorandum, as it has for its purpose partial payment of the
liability of one government agency (MIAA) to another (PNCC).
Good faith in the payment of public funds relieves a public
officer from the crime of malversation.
Art.12: Exempting Circumstances
Insanity/Imbecility

Whether or not the lower court erred in convicting


the accused despite the fact that when he allegedly
committed the offense charged he was mentally ill, out of his
mind or insane?

Facts:
On March 6, 1991, at around 1:45 in the afternoon,
Roger Cabiguen was in his house at Burgos Street, Barangay
Tagumpay, Puerto Princesa City. He was working on a
lettering job inside his bedroom together with his first cousin,
then 25-year old Elsa Rodriguez, and his friends, Simplicio
Yayen and Antonio Magbanua. Roger was working at his table
and seated on his bed while Elsa was across the table.
Antonio was on the left side, while Simplicio was seated near
the door, on the right side of Roger.
All of a sudden, accused-appellant entered the
room; uttered Roger's nickname ("Jer") and immediately
attacked him with a knife, but Roger was able to parry the
thrust and was stabbed instead on the right forearm.
Accused-appellant then stabbed Elsa Rodriguez on the chest
and said, "Ako akabales den, Elsa." (I had my revenge, Elsa).
Thereafter, accused-appellant fled, leaving the stunned
Simplicio and Antonio unharmed.
Prosecution witness Roger Cabiguen testified that
sometime in 1980, accused-appellant suspected him of killing
his pet dog. In 1989, accused-appellant courted Elsa but she
jilted him. On one occasion, Elsa spat on and slapped
accused-appellant.
of

insanity

The defense offered in evidence the April 27, 1992


medical findings on accused-appellant by Dr. Guia Melendres
of the National Center for Mental Health, pertinent portion of
which reads: In view of the foregoing history, observations,
physical mental and psychological examinations the patient
Enrico Valledor y Andusay is found suffering from Psychosis
or Insanity classified under Schizophrenia. This is a thought
disorder characterized by deterioration from previous level of
functioning, auditory hallucination, ideas of reference,
delusion of control, suspiciousness, poor judgment and
absence of insight. Likewise, he is found to be suffering from
Psychoactive Substance Use Disorder, Alcohol, abuse. This is
characterized by a maladaptive pattern of psychoactive
substance use indicated by continued use despite knowledge
of having a persistent or recurrent social, occupational,
psychological or physical problems.
Issue:

People v. Valledor (G.R. No. 129291)

Accused-appellant's defense
anchored on the following facts:

On March 11, 1991, Dr. Manuel Bilog, City Health


Officer I of Puerto Princesa City interviewed accusedappellant and thereafter made the following conclusions and
recommendation, that subject patient be committed to the
National Mental Hospital, Metro Manila for proper medical
care and evaluation soonest.

was

Decision:
The Supreme Court ruled that in considering a plea
of insanity as a defense, the starting premise is that the law
presumes all persons to be of sound mind. Otherwise stated,
the law presumes all acts to be voluntary, and it is improper
to presume that acts were done unconsciously.
Since the presumption is always in favor of sanity, he
who invokes insanity as an exempting circumstance must
prove it by clear and positive evidence. And the evidence on
this point must refer to the time preceding the act under
prosecution or to the very moment of its execution.
Insanity is evinced by a deranged and perverted
condition of the mental faculties which is manifested in
language and conduct. An insane person has no full and clear
understanding of the nature and consequences of his acts.
Hence, insanity may be shown by the surrounding
circumstances fairly throwing light on the subject, such as
evidence of the alleged deranged person's general conduct
and appearance, his acts and conduct consistent with his
previous character and habits, his irrational acts and beliefs,
as well as his improvident bargains. The vagaries of the mind
can only be known by outward acts, by means of which we
read thoughts, motives and emotions of a person, and

through which we determine whether the acts conform to


the practice of people of sound mind.
In the case at bar, accused-appellant failed to
discharge the burden of overcoming the presumption of
sanity at the time of the commission of the crime. The
following circumstances clearly and unmistakably show that
accused-appellant was not legally insane when he
perpetrated the acts for which he was charged: 1) Simplicio
Yayen was positioned nearest to accused-appellant but the
latter chose to stab Roger and Elsa; 2) Accused-appellant
called out the nickname of Roger before stabbing him; 3)
Simplicio Yayen and Antonio Magbanua who were likewise
inside the room were left unharmed; 4) Accused-appellant, a
spurned suitor of Elsa, uttered the words, "Ako akabales den,
Elsa." (I had my revenge, Elsa) after stabbing her; and 5)
Accused-appellant hurriedly left the room after stabbing the
victims.
Evidently, the foregoing acts could hardly be said to
be performed by one who was in a state of a complete
absence of the power to discern. Judging from his acts,
accused-appellant was clearly aware and in control of what
he was doing as he in fact purposely chose to stab only the
two victims. Two other people were also inside the room, one
of them was nearest to the door where accused-appellant
emerged, but the latter went for the victims. His obvious
motive of revenge against the victims was accentuated by
calling out their names and uttering the words, "I had my
revenge" after stabbing them. Finally, his act of immediately
fleeing from the scene after the incident indicates that he
was aware of the wrong he has done and the consequence
thereof.
Accused-appellant's acts prior to the stabbing
incident to wit: crying; swimming in the river with his clothes
on; and jumping off the jeepney; were not sufficient to prove
that he was indeed insane at the time of the commission of
the crime. As consistently held by this Court, "A man may act
crazy but it does not necessarily and conclusively prove that
he is legally so." Then, too, the medical findings showing that
accused-appellant was suffering from a mental disorder after
the commission of the crime, has no bearing on his liability.
What is decisive is his mental condition at the time of the
perpetration of the offense. Failing to discharge the burden
of proving that he was legally insane when he stabbed the
victims, he should be held liable for his felonious acts.

Minority

A barbecue vendor nearby heard her cries and came


to the scene; the accused fled, and the vendor told the victim
to tell her parents what happened.
Together with her parents, the victim went to the
police and reported the incident; the vendor also testified to
what he saw during that time.
The medical examiner found no injury on the hymen
and perineum, but found scanty yellowish discharge between
the labia minora; there was also fresh abrasion of the
perennial skin at 1 oclock position near the anal opening.
The trial court found the victim guilty, declaring that
he acted with discernment, but crediting him with the special
mitigating circumstance of minority.
Issue:
W/N accused had carnal knowledge of the victim,
and if yes, whether he acted with discernment, being a minor
of age more than 9 years old but less than 15?
Decision: YES
Penetration, no matter how slight, or the mere
introduction of the male organ into the labia of the
pudendum, constitutes carnal knowledge. Hence, even if the
penetration is only slight, the fact that the private
complainant felt pains, points to the conclusion that the rape
was consummated.
While it is true that medical examiner did not find any
abrasion or laceration in the private complainants genitalia,
such fact does not negate the latters testimony the
petitioner had carnal knowledge of her. The absence of
abrasions and lacerations does not disprove sexual abuses,
especially when the victim is a young girl as in this case.
The court have held that when the offended party is
young and immature, from the age of thirteen to sixteen,
courts are inclined to give credence to their account of what
transpired, considering not only their relative vulnerability
but also the shame and embarrassment to which they would
be exposed if the matter to which they testified is not true.
Discernment is the mental capacity to understand the
difference between right and wrong.
The accused, with methodical fashion, dragged the
resisting victim behind the pile of hollow blocks near the
vacant house to insure that passers-by would not be able to
discover his dastardly acts.

Llave v. People (G.R. No. 166040)


Facts:
On Sept. 24, 2002, on an errand from her mother,
the victim, who was only 7 years old at that time, proceeded
to their house, where the accused waited for her, and
accosted her; he proceeded to sexually abuse her, while the
victim cried for help.

Jose v. People (G.R. No. 162052)


Facts:
Accused was arrested in a drug buy-bust operation
conducted by the police; accused was a passenger in the car

of Zarraga, whom allegedly made the deal with the


undercover in the said operation. They claimed that they
were kidnapped by the police and asked ransom for their
release from one of the accuseds wife.
The trial court found them guilty, and credited in their
favour the preventive imprisonment they had undergone. CA
reduced the penalty on petitioner since he was 13 years old
at the time of the commission of the offense.
Issue:
W/N petitioner acted with discernment and that
prosecution failed to allege in the information that he acted
with discernment?

The accused Toledo was charged with homicide for


the killing of one Ricky Duarte. Toledo insisted that when he
killed the victim, the same was purely accidental. He claimed
that the victim was so drunk that the same charged at the
door of his house. This prompted the accused to get his bolo
and when he tried to prevent Ricky from entering, he
accidentally hit the latter whereby killing him.
But still the RTC and the Ca found him guilty.
And so, the accused goes to the SC wherein this
time, he claims that his actions were purely on self-defense. It
was done when the victim attacked him and in trying to
defend himself, he accidentally killed Duarte.
Issue:

Decision: NO
For a minor at such an age to be criminally liable, the
prosecution is burdened to prove beyond reasonable doubt,
by direct or circumstantial evidence, that he acted with
discernment, meaning that he knew what he was doing and
that it was wrong.
Such circumstantial evidence may include the utterances
of the minor; his overt acts before, during and after the
commission of the crime relative thereto; the nature of the
weapon used in the commission of the crime; his attempt to
silence a witness; his disposal of evidence or his hiding
the corpus delicti.
The only evidence of the prosecution against the
petitioner is that he was in a car with his cousin, co-accused,
when the latter inquired from the poseur-buyer, if he could
afford to buy shabu.
There is no evidence that the petitioner knew what was
inside the plastic and soft white paper before and at the time
he handed over the same to his cousin. Indeed, the poseurbuyer did not bother to ask the petitioner his age because he
knew that pushers used young boys in their transactions for
illegal drugs.
Conspiracy is defined as an agreement between two or
more persons to commit a crime and decide to commit
it. Conspiracy presupposes capacity of the parties to such
conspiracy to discern what is right from what is wrong. Since
the prosecution failed to prove that the petitioner acted with
discernment, it cannot thereby be concluded that he
conspired with his co-accused.
Accident
Toledo v. People (439 SCRA 94)
Facts:

Should the Court find his actions exempting and/or


justifying?
Decision:
The Sc ruled that there is no such thing as accidental
self-defense. The accused cannot claim the death purely
accidental and when the findings of the lower courts were
unfavorable, later on change his defense by alleging that
what happened was purely self-defense.
The two defenses perpetuated by the accused are
totally inconsistent with each other. Although in the justifying
circumstance of self-defense, an accused is excused because
of DELIBERATELY trying to repel an unlawful aggression which
could have killed or injure him. And so, such acts are not in
tune with ACCIDENT which presupposes an act which was not
even contemplated or planned but purely accidental.

People v. Concepcion (386 SCRA 74)


Facts:
The accused Concepcion is a police officer charged
with murder for the killing of one Lorenzo Galang. According
to testimonies of both parties witnesses, Lorenzo Galang was
brought to the barangay hall because he was so drunk and
unruly at the town plaza and was continually disturbing the
peace.
The accused then came to the barangay hall
apparently to question Galang. But herein lies the differences
in the testimonies. The prosecution witnesses (2 of them)
testified that while interrogating Galang, Concepcion
suddenly fired two shots past the ear of the victim without
injuring him. But later on, he hit the victim in the abdomen
and fired a shot which wounded Galang in the thigh and then
Concepcion fired three more shots which hit the victim in the
chest and killed him.

But according to the accused, he was merely


pacifying Galang when the victim became so unruly that the
accused fired two warning shots. But instead of scaring
Galang, the latter tried to grab the gun from the accused.
Two shots were accidentally fired which hit Galang thus
causing his death.
The accused claims that he should be exempted
because he was just performing his lawful duty as a police
officer and that the shooting was purely accidental.
The trial court found Concepcion guilty.
Issue:
Should the accused be exempted from criminal
liability due to accident?
Decision:
Well settled is the rule in criminal cases, that the
prosecution has the burden of proof to establish the guilt of
the accused. However, once the defendant admits the
commission of the offense charged, but raises an exempting
circumstance as a defense, the burden of proof is shifted to
him. By invoking mere accident as a defense, appellant now
has the burden of proving that he is entitled to that
exempting circumstance under Article 12 (4) of the Code.
Unfortunately for the accused, his testimony was too
full of inconsistencies which failed to discharge the burden .
For one, Concepcion claims that when the victim tried to grab
his gun, said rifle was hanging on his shoulder on a swivel. But
then he claimed that Galang tried to rest the rifle away by
grabbing the BARREL OF THE GUN. This was very
inconceivable. Furthermore, it was not believable that a
person so drunk would try to take away a rifle from a police
officer who also had a handgun tucked by his waist.
Lastly, the prosecution witness categorically testified
that he saw Concepcion shoot the victim with the M-16 rifle.
And so, the finding of guilt by the lower court was

checks in full upon its presentment, which check when


presented for payment within ninety (90) days from the date
hereof, was subsequently dishonored by the drawee bank for
Account Closed and despite receipt of notice of such
dishonor, said accused failed to pay said Manila Doctors
Hospital the amount of the checks or to make arrangement
for full payment of the same within five (5) banking days after
receiving said notice.
Ty claimed that she issued the checks because of an
uncontrollable fear of a greater injury. She claims that she
was forced to issue the checks to obtain release of her
mother whom the hospital inhumanely and harshly treated,
and would not discharge unless the hospital bills are paid.
The trial court rendered judgment against Ty. Ty
interposed an appeal with the CA and reiterated her defense
that she issued the checks under the impulse of an
uncontrollable fear of a greater injury or in avoidance of a
greater evil or injury. The appellate court affirmed the
judgment of the trial court with modification. It set aside the
penalty of imprisonment and instead sentenced Ty to pay a
fine of sixty thousand pesos P 60,000.00 equivalent to double
the amount of the check, in each case.
Issue:
Whether or not the defense of uncontrollable fear is tenable
to warrant her exemption from criminal liability?
Decision: No.
Uncontrollable fear - For this exempting
circumstance to be invoked successfully, the following
requisites must concur: (1) existence of an uncontrollable
fear; (2) the fear must be real and imminent; and (3) the fear
of an injury is greater than or at least equal to that
committed.
In the instant case, the evil sought to be avoided is
merely expected or anticipated. If the evil sought to be
avoided is merely expected or anticipated or may happen in
the future, this defense is not applicable

proper.
Irresistible Force/Uncontrollable Fear
Ty v. People (G.R. No. 149275)
Facts:
This case stemmed from the filing of
7 Informations for violation of B.P. 22 against Ty before the
RTC of Manila. The said accused drew and issue to Manila
Doctors Hospital to apply on account or for value to Editha L.
Vecino several post-dated checks. The said accused well
knowing that at the time of issue she did not have sufficient
funds in or credit with the drawee bank for payment of such

It must appear that the threat that caused the


uncontrollable fear is of such gravity and imminence that the
ordinary man would have succumbed to it. It should be based
on a real, imminent or reasonable fear for ones life or limb. A
mere threat of a future injury is not enough. It should not be
speculative, fanciful, or remote. A person invoking
uncontrollable fear must show therefore that the compulsion
was such that it reduced him to a mere instrument acting not
only without will but against his will as well. It must be of
such character as to leave no opportunity to the accused for
escape.
The fear harbored by Ty was not real and imminent. Ty
claims that she was compelled to issue the checks, a

condition the hospital allegedly demanded of her before her


mother could be discharged, for fear that her mothers health
might deteriorate further due to the inhumane treatment of
the hospital or worse, her mother might commit suicide. This
is speculative fear; it is not the uncontrollable fear
contemplated by law.

Appeals promulgated the assailed decision denying the


appeal.
Issue:
Whether or not instigation was the act which
preceded Sta. Marias arrest?
Decision:

Entrapment v. Instigation
People v. Sta. Maria (G.R. No. 171019)
Facts:
On November 27, 2002, at around 10:00 oclock in
the morning, P/Chief Insp. Noli Pacheco, Chief of the
Provincial Drug Enforcement Group of the Bulacan Provincial
Office based at Camp Alejo Santos, Malolos, Bulacan received
an intelligence report about the illegal drug activities in Sitio
Gulod, Barangay Pantubig, San Rafael, Bulacan of a certain
"Fael," who later turned out to be appellant Rafael Sta. Maria.
P/Chief Insp. Pacheco formed a surveillance team to look for
a police asset to negotiate a drug deal with appellant. In the
morning of November 29, 2002, the surveillance team
reported to P/Chief Insp. Pacheco that a confidential asset
found by the team had already negotiated a drug deal for the
purchase of P200 worth of shabu from appellant at the
latters house at No. 123 Sitio Gulod, Barangay Pantubig, San
Rafael, Bulacan between 7:00 and 7:30 in the evening of
November 29, 2002. The surveillance team then prepared for
a buy-bust operation, with PO3 Enrique Rullan as team
leader, and PO1 Rhoel Ventura, who was provided with two
(2) marked P100-bills, as poseur-buyer. At the appointed time
and place, PO1 Ventura and the confidential informant
proceeded to appellants house and knocked at the door.
Appellant opened the door and the confidential informant
introduced to him PO1 Ventura as a prospective buyer. PO1
Ventura later handed the two (2) marked P100-bills to
appellant who, in turn, gave him a plastic sachet of shabu.
Thereupon, PO1 Ventura sparked his cigarette lighter, which
was the pre-arranged signal to the other members of the buybust team that the sale was consummated. Appellant was
arrested and the two marked P100-bills recovered from him.
Also arrested on that occasion was one Zedric dela Cruz who
was allegedly sniffing shabu inside appellants house and
from whom drug paraphernalia were recovered. Upon
laboratory examination of the item bought from appellant,
the same yielded positive for methylampetamine
hydrochloride or shabu weighing 0.041 gram.
The accused was charged of violation of Section 5,
Article II of R.A. No. 9165, otherwise known as the
Comprehensive Dangerous Drugs Act of 2002.
The trial court found appellant guilty beyond
reasonable doubt of the offense charged. The Court of

In entrapment, the entrapper resorts to ways and


means to trap and capture a lawbreaker while executing his
criminal plan. In instigation, the instigator practically induces
the would-be-defendant into committing the offense, and
himself becomes a co-principal. In entrapment, the means
originates from the mind of the criminal. The idea and the
resolve to commit the crime come from him. In instigation,
the law enforcer conceives the commission of the crime and
suggests to the accused who adopts the idea and carries it
into execution. The legal effects of entrapment do not
exempt the criminal from liability. Instigation does.
It is no defense to the perpetrator of a crime that
facilities for its commission were purposely placed in his way,
or that the criminal act was done at the "decoy solicitation"
of persons seeking to expose the criminal, or that detectives
feigning complicity in the act were present and apparently
assisting its commission. Especially is this true in that class of
cases where the offense is one habitually committed, and the
solicitation merely furnishes evidence of a course of conduct.
The solicitation of drugs from appellant by the
informant utilized by the police merely furnishes evidence of
the course of conduct. The police received an intelligence
report that appellant has been habitually dealing in illegal
drugs. They duly acted on it by utilizing an informant to effect
a drug transaction with the appellant. There was no showing
that the informant induced appellant to sell illegal drugs to
him.
Chang v. People (G.R. No. 165111)
Facts:
Chang was the Municipal Treasurer of Makati who
was tasked to examine or investigate tax returns of private
corporations and companies operating within Makati and
determine the sufficiency and insufficiency of the income tax
assessed on them and collect payments, San Mateo was the
Chief Operations, Business revenue Examination, Audit
division, Makati Treasurers office.
The examiners found that Group Developers, Inc.
(GDI) incurred a tax deficiency inclusive of penalty in the
amount of P494,601.11. The assessment notice was received
by Mario Magat, Chief Operating Officer of GDI. Magat was
later able to talk to San Mateo via phone. On May 15, 1991,
Magat and San Mateo met for lunch at the Makati Sports

Club. Chang later joined the two, the three agreed that if GDI
could pay P125,000 by the end of May 1991, the assessment
would be resolved.
On June 6, 1991, Magat met again for lunch with San
Mateo and Chang at the Makati Sports Club. Magat tried to
convince the two that GDI wanted to pay the correct amount
of tax to the municipality. He was advised by San Mateo and
Chang, however, that GDI had only two options: Pay the
P494,601.11 to the municipality or P125,000 to them.

conceives the commission of the crime and suggests to the


accused who adopts the idea and carries it into execution.
From the evidence of the prosecution, it was clearly
established that the criminal intent originated from the mind
of the petitioner. Even before the June 19, 1991 meeting took
place, petitioners already made known to Magat that GDI
only had two options to prevent the closure of the company,
either to pay the assessed amount of P494,601.11 to the
Municipality or to pay the amount of P125,000 to them.

On June 12, 1991, Magat met with the NBI Deputy


Director Epimaco Velasco who advised him to file a complaint
with the NBI. Magat thus gave a sworn statement. After
several days, Magat contacted San Mateo and asked him if
their position was still the same to which the latter said yes.
Magat thereafter told San Mateo that he would deliver the
P125,000 on June 19,1991 at the Makati Sports Club.
On June 19, 1991, Magat informed the NBI that
payment was to be made that day around lunchtime. The NBI
formed a team to conduct an entrapment. The genuine
money as well as the boodle money and the envelope where
the money was placed were then laced with fluorescent
powder.
San Mateo arrived and joined Magat at his table.
Chang arrived and joined the two. Magat told Chang and San
Mateo that GDI was ready to pay and asked them if they
could give him the Certificate of Examination showing that
GDI has no more tax liability. Chang handed the Certificate
while Magat gave the brown envelope. At that instant, the
NBI agents announced that they were being arrested.
The Sandiganbayan convicted San Mateo and Chang
of violation of sec 3(b) of R.A. No. 3019, otherwise known as
Anti- Graft and Corrupt Practices Act. Hence, this appeal.

Art. 13: Mitigating Circumstances


Incomplete Justifying or
Exempting Circumstances
People v. CA and Tangan (G.R. No. 103613)
Facts:
On December 1, 1984, Navy Captain Eladio C. Tangan
was driving alone on Roxas Boulevard heading south and
Generoso Miranda was driving his car in the same direction
with his uncle, Manuel Miranda. Generoso was moving ahead
of Tangan. Suddenly, firecrackers were thrown in Generoso's
way, causing him to swerve to the right and cut Tangan's
path. Tangan blew his horn several times. Generoso, slowed
down to let Tangan pass. Tangan accelerated and overtook
Generoso, but when he got in front, Tangan reduced speed.
Generoso tried four or five times to overtake on the right lane
but Tangan kept blocking his lane. When Tangan slowed
down to make a U-turn, Generoso passed him, pulled over
and got out of the car with his uncle. Tangan also stopped his
car and got out. Generoso and Tangan then exchanged
expletives. Then Tangan went to his car and got his .38 caliber
handgun on the front seat.

Issue:
Whether or not there was a valid entrapment
operation?
Decision:
Petitioners were undisputedly public officers at the
time of the commission of the offense. The prosecution, not
only established creditably how the offense charged was
committed. It is established just as creditably how petitioners
conspired to commit the crime.
There is entrapment when law officers employ ruses
and schemes to ensure the apprehension of the criminal
while in the actual commission of the crime. There is
instigation when the accused is induced to commit the crime.
The difference in the nature of the two lies in the origin of the
criminal intent. In entrapment, the mens reoriginates from
the mind of the criminal. The idea and the resolve to commit
the crime comes from him. In instigation, the law officer

According to the prosecution witnesses, Mary Ann


Borromeo, Rosalia Cruz and Manuel Miranda, the accused
pointed his gun at Generoso Miranda and when Manuel
Miranda tried to intervene, the accused pointed his gun at
Manuel Miranda, and after that the accused pointed again
the gun to Generoso Miranda, the accused shot Generoso
Miranda at a distance of about a meter. The shot hit the
stomach of Generoso Miranda causing the latter to fall.
Manuel Miranda grappled for the possession of the gun and
during their grappling, Rosalia Cruz intervened and took hold
of the gun and after Rosalia Cruz has taken hold of the gun, a
man wearing a red T-shirt took the gun from her. The man in
T-shirt was chased by Manuel Miranda who was able to get
the gun where the man in red T-shirt placed it.
On the other hand, the defense, particularly the
accused and his witness by the name of Nelson Pante claimed
that after the gun was taken by the accused from inside his
car, the Mirandas started to grapple for possession of the gun
and during the grappling, and while the two Mirandas were

trying to wrest away the gun from the accused, they fell
down at the back of the car of the accused. The accused lost
the possession of the gun after falling at the back of his car
and as soon as they hit the ground, the gun fell, and it
exploded hitting Generoso Miranda.
Tangan ran away while Generoso lay on the ground bloodied.
Manuel looked for the gun and ran after Tangan. Tangan
found a policeman who allowed him to enter his patrol car.
Manuel arrived and told the policeman that Tangan had just
shot his nephew. Manuel went back to where Generoso lay
and there found two ladies, Mary Ann Borromeo and Rosalina
Cruz, helping his nephew board a taxi. Manuel suggested that
Generoso be brought to the hospital in his car. He was rushed
to the Philippine General Hospital but he expired on the way.
Tangan was charged with the crime of murder with the use
of an unlicensed firearm. However, the information was
amended to homicide with the use of a licensed firearm, and
he was separately charged with illegal possession of
unlicensed firearm. Tangan entered a plea of not guilty in the
homicide case, but moved to quash the information for illegal
possession of unlicensed firearm on various grounds. The
motion to quash was denied, whereupon he filed a petition
for certiorari with this Court. On November 5, 1987, said
petition was dismissed and the joint trial of the two cases was
ordered.
After trial, the lower court acquitted Tangan of illegal
possession of firearm, but convicted him of homicide. The
privileged mitigating circumstance of incomplete self-defense
and the ordinary mitigating circumstances of sufficient
provocation on the part of the offended party and of passion
and obfuscation were appreciated in his favor; Tangan was
released from detention after the promulgation of judgment
and was allowed bail in the homicide case.
Tangan appealed to the Court of Appeals, which affirmed the
judgment of the trial court but increased the award of civil
indemnity to P50,000.00. His subsequent motion for
reconsideration and a motion to cite the Solicitor General in
contempt were denied by the Court of Appeals.
The Solicitor General, on behalf of the prosecution, alleging
grave abuse of discretion, filed a petition for certiorari under
Rule 65, naming as respondents the Court of Appeals and
Tangan, where it prayed that the appellate court's judgment
be modified by convicting accused-appellant of homicide
without appreciating in his favor any mitigating
circumstance.
Issue:
Whether or not Tangan acted in incomplete selfdefense?
Decision:

Incomplete self-defense is not considered as a


justifying act, but merely a mitigating circumstance; hence,
the burden of proving the crime charged in the information is
not shifted to the accused. In order that it may be
successfully appreciated, however, it is necessary that a
majority of the requirements of self-defense be present,
particularly the requisite of unlawful aggression on the part
of the victim. Unlawful aggression by itself or in combination
with either of the other two requisite suffices to establish
incomplete self-defense. Absent the unlawful aggression,
there can never be self-defense, complete or
incomplete, because if there is nothing to prevent or repel,
the other two requisites of defense will have no basis.
The element of unlawful aggression in self-defense
must not come from the person defending himself but from
the victim.
A mere threatening or intimidating attitude is not
sufficient. The exchange of insulting words and invectives
between Tangan and Generoso Miranda, no matter how
objectionable, could not be considered as unlawful
aggression, except when coupled with physical assault. There
being no lawful aggression on the part of either antagonists,
the claim of incomplete self-defense falls.
No intention to commit so grave wrong
PEOPLE VS. CALLET 382 SCRA 42 / 43
APPELLEE: PEOPLE OF THE PHILIPPINES
APPELLANT: ELBERT CALLET y SABANAL
PON: JUSTICE REYNATO S. PUNO
FACTS: On September 15, 1996 at 3:00 p.m., the accused,
Elbert Callet, played volleyball near the flea market. After
two (2) games, he stopped playing. It was past 4:00 p.m. He
stayed at the flea market and watched as others played
volleyball. While watching the game, he was hit on the left
side of the body by Alfredos elbow. He asked Alfredo why he
hit him. Alfredo retorted, Are you angry? Next, Alfredo
grabbed his left arm and tried to twist it. He pleaded with
Alfredo to let go of his arm, but Alfredo warned that he
would be his third victim if he would get angry with him. As
Alfredo was pulling out Provocation or Threatsa hunting knife
from his waist, he (the accused) managed to stab him first.
Thereafter, he ran towards the municipal hall to surrender.
The accused also claims that his liability should be mitigated
by the fact that he had no intention to commit so grave a
wrong.
Issue: won the accused should be given the benefit of
mitigating circumstance of acting without intention to
commit so grave a wrong.
Held: no, The lack of intent to commit a wrong so grave is
an internal state. It is weighed based on the weapon used,
the part of the body injured, the injury inflicted and the

manner it is inflicted. The fact that the accused used a 9-inch


hunting knife in attacking the victim from behind, without
giving him an opportunity to defend himself, clearly shows
that he intended to do what he actually did, and he must
be held responsible therefor, without the benefit of this
mitigating circumstance.
When mitigating circumstance of sufficient provocation
appreciated
PEOPLE vs TORPIO 431 scra 9
The accused was invited by the deceased for a drinking spree.
They had some round of drinks of tuba and beer at a nearby
store and a cottage at the seashore. For one reason or
another, because the accused refused to drink anymore, the
deceased got angry and he then bathed the accused with gin,
and boxed or mauled him and tried to stab him with a
batangas knife but failed to hit the accused as the latter was
crawling under the table. The accused got up and ran towards
their home got a knife and went back and upon arrival, the
deceased seeing the accused, ran. The accused caught up and
stabbed the deceased. When he was hit, the deceased ran
but then he got entangled with a fishing net and fell on his
back, and the accused mounted on him and continued
stabbing him. The trial court rendered judgment convicting
him of murder.

was proven by the wrong done on Dennis by Anthony prior to


the stabbing incident. The injury he sustained, the mauling,
the humiliation he suffered, the near attempt at killing
Dennis, these constitute some grave offense and an interval
of time elapsed before the accused returned and did the
commission of a felon which is killing.
NOTES:
In par 4 of art 13 immediately preceded the act there should not be any interval of time. The Spanish text is
controlling la de haber precedido inmediatamente
provocacion o amenaza adecuada
In par 5 of art 13 in the immediate vindication Immediate means proximate and, hence, an interval of
time may lapse from the commission of
the grave offense to the crime in vindication thereof. la de
haber ejucatado en vindicacion proxima de una ofensa grave
Vindication of a Grave offence
PEOPLE vs TORPIO 431 scra 9

See case above


Passion or Obfuscation
People v. Malejana (G.R. No. 145002)

ISSUE: WON the accused could invoke the mitigating


circumstance of sufficient provocation on the part of the
offended party.
,
RULING: The accused Dennis Torpio y Estrera is found guilty
beyond reasonable doubt of Homicide.
The appellant was humiliated, mauled and almost stabbed by
the deceased. Although the unlawful aggression had ceased
when the appellant stabbed Anthony, it was nonetheless a
grave offense for which the appellant may be given the
benefit of a mitigating circumstance of having acted in the
immediate vindication of a grave offense. But the mitigating
circumstance of sufficient provocation cannot be considered
apart from the circumstance of vindication of a grave offense.
These two circumstances arose from one and the same
incident, i.e., the attack on the appellant by Anthony, so that
they should be considered as only one mitigating
circumstance.

Facts:

In appreciating the mitigating circumstances, the trial court


explained:...The Court considers for appreciation the
following (1) that sufficient provocation on the part of the
deceased preceded the act, this is shown by the mauling of
Dennis, his being bathed with liquor, and the deceaseds
having tried to stab Dennis at the cottage before Dennis went
home and got his knife. (2) The act of killing was committed
in the immediate vindication of a grave offense to the one
committing the felony. Immediate means proximate and,
hence, an interval of time may lapse from the commission
of the grave offense to the crime in vindication thereof. This

Issue:

Prosecutions witness Andres Madrid narrated that


on July 28, 1990 at around 7:15 p.m., while he was seated in
front of his jeep parked at the side of the road at Marisfoque,
Pilar, Sorsogon in the company of Janus Roces, Antonio Sy,
Samuel Andrade, Bernarda Sy, Jose Belmonte and Ernesto
Francisco, he saw appellant at about 30 meters away heading
towards their direction. Upon reaching their group, appellant
asked them where Roces was. When he noticed Roces who
was sitting at a distance of 1 meter beside Madrid, appellant
brandished an armalite rifle and fired a shot into the air.
Then he pointed the barrel of the gun at Roces and fired five
(5) times, hitting Roces thrice. After the victim fell to the
ground, appellant left the scene of the incident and went ot
his house about 150 meters away. Madrid and his other
companions tried to assist Roces but discovered that the
latter was already dead, presumably from the gunshot
wounds that were inflicted upon him by appellant.

Whether or not the defense was able to establish


that accused was entitled to the mitigating circumstance of
passion and obfuscation?
Decision:
Passion and obfuscation similarly cannot be
appreciated in favor of the appellant. To be entitled to this
mitigating circumstance, the following elements must be
present: 1) There should be an act both unlawful and
sufficient to produce such condition of mind, 2) The act that

produced the obfuscation was not far removed from the


commission of the crime by a considerable length of time,
during which the perpetrator might recover his normal
equanimity. The bare assertion that the victim and appellant
had an argument does not provide justifiable basis for
applying to him this mitigating circumstance. The cause that
produced the passion and obfuscation has not been
established nor proven by clear and convincing evidence. The
defense advance mere speculations and conjectures to gloss
over the fact that there is lack of proof of the cause. Courts
are not permitted to render judgments upon guesses or
surmises. Suspicion, it has been said, cannot give probative
force to testimony which in itself is insufficient to establish or
justify an inference of a particular fact
People V Genosa 319 scra 537
FACTS: Marivic Genosa was charged by the Regional Trial
Court of Ormoc City with parricide with treachery as an
aggravating circumstance for killing his own husband Ben
Genosa. Marivic and Ben were married since Nov. 19, 1983 at
first the couple lived with the parents of Ben in their house at
Isabel, Leyte. But in 1995, the couple rented a house at
Barangay Bilwang, Isabel, Leyte where they lived with their 2
childre: John Marben and Earl Pierre.
On Nov. 15, 1995 Ben and Arturo Basobas went to a cockfight
after receiving their salary, both had 2 bottles of beer before
heading home. When they arrived at Ben house, the
appellant had gone to Isabela, Leyte to look for her husband.
While Ben went inside his house, Arturo went to a store just
across it. Arturo did not see Marivic arrive but on his way
home he heard at the Genosas rented house that Marivic
said I wont hesitate to kill you and so Ben replied Why kill
me when I am innocent?
On Nov. 16, 1995 Marivic asked Erlinda Paderog her close
friend and neighbor to look after her pig because she was
going to Cebu for her pregnancy check-up. That same day,
Joseph Valida saw Marivic going out of their house with 2 of
her kids carrying a bag.
On Nov. 18, 1995 neighbors of Steban Matiga told him about
the foul odor emanating from his house being rented by Ben
and Marivic. Steben went to that house and inside the unlock
bedroom he saw the lifeless body of Ben lying on his side on
the bed covered with a blanket. Ben was only in his briefs
with injuries at the back of his head.
Marivic Genosa admitted that she killed Ben when she
testified. On Nov. 15, 1995 she said that she was going home
after work when she got worried that her husband who was
not home yet might have gone gambling since it was payday.
With her cousin Ecel Arano, they looked for him at the
marketplace and taverns at Isabel, Leyte but did not find him
there. They just found Ben who was drunk at their rented
house; Ecel went home even her cousin requested her to
sleep in their house.
Ben got angry to Marivic and nagged her for following him
and even challenged her to a fight. She just ignored him and
attended to her children who were doing their assignments.

Ben turned off the light with a chopping knife, cut the
television antenna to keep her from watching television.
According to Marivic, Ben was about to attack her so she ran
to the bedroom, but Ben got hold of her hands and whirled
her around. She fell on the side of the bed and screamed for
help. Ben left. At this point, appellant packed his clothes
because she wanted him to leave. Seeing his packed clothes
upon his return home, Ben allegedly flew into a rage, dragged
appellant outside of the bedroom towards a drawer holding
her by the neck, and told her You might as well be killed so
nobody would nag me. Appellant testified that she was
aware that there was a gun inside the drawer but since Ben
did not have the key to it, he got a three-inch long blade
cutter from his wallet. She however, smashed the arm of
Ben with a pipe, causing him to drop the blade and his
wallet. Appellant then smashed Ben at his nape with the
pipe as he was about to pick up the blade and his wallet. She
thereafter ran inside the bedroom.
Appellant, however, insisted that she ended the life of her
husband by shooting him. She supposedly distorted the
drawer where the gun was and shot Ben. He did not die on
the spot, though, but in the bedroom.

Issue: Whether Passion or Obfuscation is appreciated in the


case.
HELD: Due to suffering of Battered Woman Syndrome, 2
mitigating circumstances were found in this case:
PSYCHOLOGICAL PARALYSIS AND PASSION & OBFUCATION
which did not rise from the same set of facts; BUT has
AFFECTED the APPELLANT, Marivic.
the first circumstance arose from the cyclical nature
and the severity of the battery inflicted by the batterer-spouse
upon appellant. That is, the repeated beatings over a period
of time resulted in her psychological paralysis, which was
analogous to an illness diminishing the exercise of her will
power without depriving her of consciousness of her acts.
The second circumstance, on the other hand, resulted
from the violent aggression he had inflicted on her prior to the
killing. That the incident occurred when she was eight
months pregnant with their child was deemed by her as an
attempt not only on her life, but likewise on that of their
unborn child. Such perception naturally produced passion and
obfuscation on her part.
The Court, however, is not discounting the possibility of selfdefense arising from the battered woman syndrome. We
now sum up our main points. First, each of the phases of the
cycle of violence must be proven to have characterized at
least two battering episodes between the appellant and her
intimate partner. Second, the final acute battering episode
preceding the killing of the batterer must have produced in
the battered persons mind an actual fear of an imminent
harm from her batterer and an honest belief that she needed
to use force in order to save her life. Third, at the time of the

killing, the batterer must have posed probable -- not


necessarily immediate and actual -- grave harm to the
accused, based on the history of violence perpetrated by the
former against the latter.
Taken altogether, these
circumstances could satisfy the requisites of self-defense.
HOWEVER, FOR THIS CASE, NOT ALL OF THE SAID ELEMENTS
WERE ESTABLISHED.
For the issue of treachery, the testimonies are insufficient to
establish its presence in the crime. It is indeed an equally
axiomatic rule when the killing is preceded by an argument or
quarrel, treachery cannot be appreciated as a qualifying
circumstance because the deceased may be said to have
been forewarned and to have anticipated aggression from
the assailant.
WHEREFORE, the conviction of Appellant Marivic
Genosa for parricide is hereby AFFIRMED. However, there
being two (2) mitigating circumstances and no aggravating
circumstance attending her commission of the offense, her
penalty is REDUCED to six (6) years and one (1) day of prision
mayor as minimum; to 14 years, 8 months and 1 day
of reclusion temporal as maximum.

Decision: Passion and obfuscation may not be properly


appreciated in favor of appellant. To be considered as a
mitigating circumstance, passion or obfuscation must arise
from lawful sentiments and not from a spirit of lawlessness or
revenge or from anger and resentment. In the present case,
clearly, Marcelo was infuriated upon seeing is brother,
Carlito, shot by Jose. However, a distinction must be made
between the first time that Marcelo hacked Jose and the
second time that the former hacked the latter. When
Marcelo hacked Jose right after seeing the latter shoot at
Carlito, and if appellant refrained from doing anything else
after that, he could have validly invoked the mitigating
circumstance of passion and obfuscation. But when, upon
seeing his brother Carlito dead, Marcelo went back to Jose,
who by then was already prostrate on the ground and hardly
moving, hacking Jose again was a clear case of someone
acting out of anger in the spirit of revenge.

Voluntary Surrender
People V Nimuan 645 scra 739
People v Maglian 646 scra 770

People v. Bates (G.R. No. 139907)

People v Concillado 661 scra 363

Facts:
Around 2:00 in the afternoon of November 28, 1995,
Edgar Fuentes, Simon Fuentes and Jose Boholst left Barangay
Esperanza, Ormoc City to deliver copra to a certain Fely
Rodado at Barangay Green Valley, Ormoc City. After
delivering copra around 5:00 in the afternoon, the three men
headed back to Barangay Esperanza. While they were along a
trail leading to the house of Carlito Bates, the latter suddenly
emerged from the thick banana plantation surrounding the
trail, aiming his firearm at Jose Boholst who was then walking
ahead of his companions. Jose grabbed Carlitos right hand
and elbow and tried to wrest possession of the firearm.
While the two were grappling for possession, the gun fired,
hitting Carlito who immediately fell to the ground. At that
instant, Marcelo Bates and his son Marcelo Bates, Jr., brother
and nephew of Carlito, respectively, emerged from the
banana plantation each brandishing a bolo.
They
immediately attacked Jose hacking him several times. Jose
fell to the ground and rolled but Marcelo and his son kept on
hacking him. Marcelo, then, turned to Simon and Edgar and
shouted huwes de kutsilyo. Upon hearing the same, Simon
and Edgar ran.
Upholding the prosecution evidence, the trial court
rendered its Judgment, finding Marcelo Bates guilty beyond
reasonable doubt of the crime of Murder.
Issue: Whether or not Marcelo could validly invoke the
mitigating circumstance of passion and obfuscation?

Confession of Guilt
People v. Monttinola 360 scra 631

People v. Dawaton (G.R. No. 146247)


Facts:
Edgar Dawaton was found guilty by the trial court of
murder qualified by treachery and was sentenced to death.
On 20 September 1998, Leonidas Lavares and several
companions, including Dawaton were drinking in the house of
the accuseds uncle. Already drunk, Leonidas Lavares decided
to sleep while the accused and his companions continued
drinking. Dawaton awakened Lavares by stabbing him at the
base of the neck. Dawaton continued stabbing Lavares until
the victim died. Dawaton then ran away to the house of his
other relative, where he was later on arrested by the police.
Issue: Whether or not the penalty of death imposed by the
trial court upon the accused was correct?
Decision:
No. The Supreme Court held that the trial court
erred in not considering the alternative circumstance of
intoxication in favor of the accused. Under Art. 15 of The
Revised Penal Code, intoxication of the offender shall be
considered as a mitigating circumstance when the offender
commits a felony in a state of intoxication, if the same is not
habitual or subsequent to the plan to commit said felony.
Otherwise, when habitual or intentional, it shall be

considered as an aggravating circumstance. The allegation


that the accused was drunk when he committed the crime
was corroborated by the prosecution witnesses. The accused
and his drinking companions had consumed four (4) bottles
of gin at the house of Esmeraldo Cortez, each one drinking at
least a bottle. It was also attested that while the four (4)
shared another bottle of gin at the house of Amado Dawaton,
it was the accused who drank most of its contents. The
Court further stated that Under Art. 63, par. 3, of The
Revised Penal Code, in all cases in which the law prescribes a
penalty composed of two (2) indivisible penalties, such as in
this case, when the commission of the act is attended by a
mitigating circumstance and there is no aggravating
circumstance, the lesser penalty shall be applied. Since no
aggravating circumstance attended the killing but there
existed the mitigating circumstance of intoxication, the
accused should be sentenced only to the lesser penalty of
reclusion perpetua.
Similar and Analogous Circumstances
Canta v. People (G.R. No. 140937)
Facts:
Narciso Gabriel owns a cow that was passed on from
one person to another and each person was responsible for
the care and custody of the said cow. At the time the cow
got lost, it was under the care and custody of Gardenio
Agapay. Agapay took the cow in the mountain of Pilipogan,
40 meters away from his hut, at around 5:00 in the
afternoon. When he came back to get the cow at past 9 in
the evening, the cow was gone. However, Aagapay saw
footprints that led to the house of Filomeno Vallejos. Vallejos
told Agapay that Exuperancio Canta took the cow.
Agapay and Maria were instructed by Narciso to get
the cow and on their way to Florenitno Cantas house, they
saw Exuperancio. The latter told them that if it was really
Narciso who was the owner of the cow, he should get it
himself. Exuperancia accompanied the two to his fathers
house and both recognized the cow but Florentino was not
home. Exuperancio told Maria and Agapay that he would call
them the next day to talk about the matter with his father.
Exuperancio never called. The matter was reported to the
police and Narciso and Exuperancio were called for
investigation. Exuperancio admitted taking the cow but
claims that he was the real owner of the cow and that it was
lost on December 3, 1985. However, Narciso presented a
certificate of ownership issued on March 9, 1986, signed by
the municipal treasurer, in which the cow was described as
two years old and female. Then, the petitioner also
presented a Certificate of Ownership of Large Cattle dated
February 27, 1985 and a statement executed by Franklin
Telen, who was the janitor at the treasurer's office of the
municipality, that he executed the certificate of ownership in
favor of Exuperancio. The trial court rendered its decision
finding petitioner guilty of the offense charged. Exuperancio

filed a Motion for reconsideration but was denied by the


Court of Appeals and affirmed the trial court's decision.
Issue:
Whether or not the lower courts were correct in
sentencing Exuperancio to ten (10) years and one (1) day of
prision mayor, as minimum, to twelve (12) years, five (5)
months, and eleven (11) days of reclusion temporal medium,
as maximum, and to pay the costs?
Decision: No. The Supreme Court held that the trial court
correctly found petitioner guilty of violation of 2(c) of P. D.
No. 533, otherwise known as the Anti-Cattle Rustling Law of
1974. However, it erred in imposing the penalty of 10 years
and 1 day of prision mayor, as minimum, to 12 years, 5
months and 11 days of reclusion temporal medium, as
maximum. The trial court apparently considered P. D. No. 533
as a special law and applied 1 of the Indeterminate Sentence
Law, which provides that "if the offense is punished by any
other law, the court shall sentence the accused to an
indeterminate sentence, the maximum term of which shall
not exceed the maximum fixed by said law and the minimum
shall not be less than the minimum term prescribed by the
same." However, as held in People v. Macatanda, P. D. No.
533 is not a special law. The penalty for its violation is in
terms of the classification and duration of penalties
prescribed in the Revised Penal Code, thus indicating that the
intent of the lawmaker was to amend the Revised Penal Code
with respect to the offense of theft of large cattle. In fact, 10
of the law provides:
The provisions of Articles 309 and 310 of Act No. 3815,
otherwise known as the Revised Penal Code, as amended,
pertinent provisions of the Revised Administrative Code, as
amended, all laws, decrees, orders, instructions, rules and
regulations which are inconsistent with this Decree are
hereby repealed or modified accordingly.
There being one mitigating circumstance and no
aggravating circumstance in the commission of the crime, the
penalty to be imposed in this case should be fixed in its
minimum period. Applying the Indeterminate Sentence Law,
in relation to Art. 64 of the Revised Penal Code, petitioner
should be sentenced to an indeterminate penalty, the
minimum of which is within the range of the penalty next
lower in degree, i. e., prision correccional maximum to prision
mayor medium, and the maximum of which is prision mayor
in its maximum period.

Art. 14: Aggravating Circumstances


Classes of Aggravating Circumstances
People v. Evina (405 SCRA 152)
Facts:

Gerardo Gavina was serve sentence of Reclusion


Pertpetua for raping certain Ms. Maritess Catcharo. Based on
the given facts, Gerardo took advantage of the time when the
victims mother was not around. He would likely forced
Maritess to have carnal knowledged against her will and even
poked a knife at her while doing the deed in the victims
dwelling and threthened the victim to kill her family should
she tell her parents what happened. On November 13, 1991
when the appellant arrived at the Catcharro residence he
proceeded inside the bedroom of Maritess, the latter ran out
of the bedroom and told her mother not to leave her because
her Papa Gerry might raped her again. Surprised by what he
heard, the following day Maritess was brought to Tacloban
City Medical Center for a check-up and found to have
lacerations to the victims genitalia. Contrary to the facts
above, appellant claimed that the night of the incident he
was working as porter until 10 PM, thus it cannot be said that
he committed the crime accused of him. Based on the
information submitted, aggravating circumstances of use of
weapon and dwelling were not alleged.
Issue:

Frustrated homicide but not guilty of violation of COMELEC


RES. 2958.
Issue:
Whether or not violation of COMELEC RES. 2958 may
be considered as Special aggravating circumstances which will
negate consideration of mitigating circumstances of voluntary
surrender?
Decision:
With the passage of Republic Act. No. 8294 on 6
June 1997, the use of an unlicensed firearm in murder or
homicide is now considered as a SPECIAL aggravating
68
circumstance and not a generic aggravating circumstance.
Republic Act No. 8294 applies to the instant case since it took
effect before the commission of the crimes in 21 April 1998.
Therefore, the use of an unlicensed firearm by the petitioner
in the instant case should be designated and appreciated as a
SPECIAL aggravating circumstance and not merely a generic
aggravating circumstance.

Whether or not aggravating circumstances proved


during trial but was not alleged in the information may be
considered?
People v. Mendoza (327 SCRA 695)
Decision:
The supreme court held in the negative. Although
the special aggravating circumstance of the use of a weapon
and the aggravating circumstance of dwelling were proven,
these aggravating circumstances cannot be considered in
fixing the penalty because they were not alleged in the
information as mandated by Rule 110, Sections 8 and 9 of the
Revised Rules of Criminal Procedure. Although the crimes
charged were committed before the effectivity of the said
rule, nevertheless, the same should be applied retroactively
being favorable to the appellant.
Katrina Garcia
2006-0127
People v. Palaganas (501 SCRA 533)
Facts:
On January 16, 1998 brothers Servillano and Michael
Ferrer went to Tidbits Videoke bar singing and drinking beer.
On the same evening Jaime Palaganas and Ferdinand
Palaganas and Virgilio Bautista arrived. The two groups
occupied separate tables. After the Ferrers singing Jaime
Palaganas started singing and was joined by Tony Ferrer who
sang loudly and in mocking manner. This insulted Jaime and
soon a fight ensued between Ferrers and Palaganas.
Ferdinand ran towards his house and sought help from his
brother Fuijeric, the latter went outside however he was
stoned by the Ferrer brothers. As they were continuously
stoned the appellant Ferdinand suddenly pulled the trigger
with the gun in his hands. The trial court rendered a decision
finding the petitioner guilty of the crime of Homicide and

Facts:
Efren Mendoza was charged with the crime of
murder for killing Anchito Nano. In this case Efren alleged
that Anchito Nano arrived at their house and upon arrival it
started to destroy the house and that the her wife was
shouting for help. Efren immediately look for something to
protect his family but found a bolo. He approached Anchito
but the latter tried to hacked him but he was able to hacked
him first on the right side of his neck resulting to the death of
the victim. Thereafter Mendoza went to Municipal Hall of
Vinzon and voluntarily surrendered to the police. He claimed
that it was self defense. The autopsy revealed that location
of the wounds found on the body of the victim came from the
back of the victims body. The court ruled rejecting
appellants self defense. This court finds that the accused
was not in imminent danger of death or great bodily harm, an
attempt to defend himself by means which appeared
unreasonable by using a long bolo is unjustifiable. Hence this
appeal.
Issue:
Whether or not voluntary surrender was offset by
the aggravating circumstances of treachery?
Decision:
The Supreme Court held in the negative. A
qualifying circumstance changes the nature of the crime. A
generic aggravating circumstance, on the other hand, does

not affect the designation of the crime; it merely provides for


the imposition of the prescribed penalty in its maximum
period. Thus, while a generic aggravating circumstance may
be offset by a mitigating circumstance, a qualifying
32
circumstance may not.
Treachery in the present case is a qualifying, not a generic
aggravating circumstance. Its presence served to characterize
the killing as murder; it cannot at the same time be
considered as a generic aggravating circumstance to warrant
the imposition of the maximum penalty. Thus, it cannot offset
voluntary surrender.
In Contempt or With Insult to the Public Authorities
People v. De Mesa (G.R. No. 137036)
Facts:
Barangay Chairman Patricio Motas of Sta. Cruz
Putol, San Pablo City was pronounced dead on arrival on
October 15, 1996 at San Pablo City District Hospital. The
autopsy report showed that the cause of death was shock
and hemorrhage due to gunshot wounds at the back of the
victim.
Hernando De Mesa was found guilty beyond
reasonable doubt for the crime of murder by the Regional
Trial Court of San Pablo City. He was sentenced to suffer the
penalty of Reclusion Perpetua, pay the costs and to
indemnify the heirs of the victim. Treachery, nighttime, in
contempt of or with assault to public authorities, were
appreciated by the trial court as aggravating circumstances
attending the case thereby qualifying the crime committed
to murder.

Renato Tac-an and Francis Escano were close friends


being classmates in high school and members of the local
Bronx gang. Francis withdrew from the gang on the advice of
his mother who saw that Renato carried a handgun on his
visits to their home. Things started turning sour between the
two, and came to a head on Dec 14, 1984. After an earlier
altercation on that day, Renato went home and got his gun.
He entered the Mathematics class under Mr. Damaso Pasilbas
in Rm15 and shouted for Francis. After locating the victim he
fired at him but missed. He was later able to hit him in the
head as he was running to the door with his classmates to
escape. After this, Renato paced outside in the hallway. A
teacher unknowing that Renato was the culprit, asked him for
help unwittingly informing him that Francis was still alive.
Renato immediately re-entered the room and saying "So, he
is still alive. Where is his chest?" Standing over Francis
sprawled face down on the classroom floor, Renato aimed at
the chest of Francis and fired once more. The bullet entered
Francis' back below the right shoulder, and exited on his front
chest just above the right nipple.
Tac-an was charged with illegal possession of
firearms under P.D. No. 1866. An amended information for
murder was subsequently filed aggravated by the use of
illegal possession of firearms.
Issues:
Whether or not illegal possession of a firearm is a
special aggravating circumstance in crimes of homicide and
murder?
Decision:

Issue:
Whether or not the trial court erred in determining
the nature of the crime committed and the corresponding
penalty to be imposed?
Decision:
Yes. The prosecution failed to positively prove the
presence of any qualifying aggravating circumstance whereby
the crime committed is only homicide for which the
imposable penalty provided by the Revised Penal Code is
Reclusion Temporal.
Being the case, Indeterminate Sentence Law may
now be applied and absent any aggravating nor mitigating
circumstance, the penalty that may be imposed is prision
mayor in its medium period as minimum to reclusion
temporal in its medium period as maximum.

People v. Tac-an (G.R. No. 76338-39)


Facts:

No. Under an information charging homicide or


murder, the use of an unlicensed firearm is not an
aggravating circumstance nor can it be used to increase the
penalty for the second offense of homicide or murder to
death or reclusion perpetua. The character of the instrument
used in taking or destroying human existence is not one of
those included in the enumeration of aggravating
circumstances under Article 14 of the Revised Penal Code.
On the other hand, under an information for
unlawful possession of a firearm or ammunition, P.D. 1866
authorizes the increase of the imposable penalty for unlawful
possession if the unlicensed firearm was used to destroy
human existence. Though it is not one of the enumerated
aggravating circumstances in Article 14 of the Revised Penal
Code, it may still be considered to increase the penalty
imposed because of the explicit provision of the said special
law.

Abuse of Public Position


Fortuna v. People (G.R. No. 135784)

Facts:
Issue:
On July 21, 1992, siblings Diosdada Montecillo and
Mario Montecillo were standing at the corner of Mabini and
Harrison Streets. A mobile patrol car stopped in front of
them and a policeman alighted. The policeman frisked Mario
and took Marios belt. He motioned Mario to enter the car.
Mario obeyed and was followed by Diosdada. While inside
the car, the policemen told Mario that he would be brought
to the Bicutan police station where he would be interrogated,
mauled and heckled for carrying a deadly weapon. They told
the Montecillos that the bailbond for carrying a deadly
weapon was P12,000. The Montecillos were asked how much
they had and then Diosdada was asked to alight from the car.
The driver followed her, took P1,500 from her wallet and
instructed her to tell the others that she only had P3,500.
Inside the car, they were told to put all her money on the
box. The Montecillos were told to get off at Harrison Plaza.
From there, they went home. The 3 policemen, Fortuna,
Garcia, and Pablo, were charged with robbery and were
found guilty of having conspired in committing the crime with
intimidation of persons.
Issue:
Whether or not abuse of public position should be
taken as an aggravating circumstance by the mere fact that
the accused were police officers?

Whether or not the Trial Court properly applied the


aggravating circumstance of taking advantage of public
position?
Decision:
The Supreme Court ruled that the aggravating
circumstance of taking advantage of public position under
paragraph 1 of Article 14 of the Revised Penal Code was
improperly applied.
A public officer must use the influence that is vested
in his office as a means to realize the purpose of the crime to
be appreciated as an aggravating circumstance. The question
Did the accused abuse his office to commit the crime must
be asked in order to appreciate this circumstance as an
aggravating circumstance.
No proof was shown that Villamor
took advantage of his position of being a policeman when he
shot Jelord Velez. Neither was his influence, prestige or
ascendancy used in killing Velez. Even without occupying a
public position, the accused could have committed the crime

Night time, Uninhabited or Obvious Place or Band


Decision:
People v. Villanueva (G. R. No. 135330)
The Supreme Courted held that the lower courts
failed to appreciate the aggravating circumstance of abuse
of public position.
Being police officers, it placed them in a position terrify the
Montecillos to surrender their money as bail. It was on the
account of their authority that convinced the Montecillos that
they had committed a crime and that they would be taken to
the police station. Had they not been police officers, they
would have not convinced the Montecillos into giving them
their money.

Facts:
For automatic review by the Supreme Court is the
judgment of 12 May 1998, of the Regional Trial Court, Branch
15, Cebu City, in Criminal Case No. CBU-46026-A, finding
accused-appellant Camilo Villanueva (hereafter Camilo) guilty
of the crime of rape committed on the victim, Nia Gabuya
(hereafter Nia), and sentencing him to suffer the extreme
penalty of death and to pay Nia moral damages in the amount
of P50,000.

People v. Villamor (G.R. Nos. 140407-08)


Facts:
On November 25, 1995, brothers Jerry Velez and
Jelord Velez were on their way home on board a motorcycle.
A motorcycle was speeding behind them and as they were
about to cross the bridge, they heard gun shots firing behind
them. As they turned around, Jerry identified PO3 Renato
Villamor and Jessie Maghilom riding the motorcycle behind
them. Shots were fired at them and Jerry sustained wounds
on the abdomen and elbow while Jelord died on the spot. The
trial proceeded against PO3 Villamor while Maghilom was still
at large. During trial, the Trial Court found the PO3 Renato
Villamor guilty of having commited Murder aggravated by the
circumstance of taking advantage of his public position.

In an amended Information dated 16 January 1998,


Camilo was charged with rape as defined and penalized in
R.A. No. 8353. The accusatory portion of the indictment
states:
That at midnight of 4 December 1997, in Cebu City,
Philippines, and within the jurisdiction of this Honorable
Court, the accused, with deliberate intent, with force and
intimidation upon person, did then and there willfully,
feloniously and unlawfully have carnal knowledge with Nia
Gabuya, a minor of only 11 years of age and step-daughter of
the said accused, against the latters will. CONTRARY TO LAW.
Issues:

Whether the private complainant Nia Gabuya was


really raped by the accused Camilo Villanueva on 4 December
1997 at around midnight?
Whether the testimony of the private complainant
Nia Gabuya is not tainted with material inconsistencies and
grievous falsity?
Whether the testimony of the private complainant is
enough to convict the accused for a crime punishable by
death?

Decision:
The issue of credibility raised in the three assigned
errors should be resolved against Camilo.
Nia clearly testified that Camilo raped her. She
recounted the details of her harrowing experience in a
credible, convincing and straightforward manner.
The prosecution was able to establish with moral
certainty the fact of penetration, although incomplete. In
order that the crime of rape may be said to be consummated,
the successful penetration by the rapist of the females
genital organ is not indispensable. Penile invasion, it has
often been held, necessarily entails contact with the labia and
even the briefest of contacts under circumstances of force,
intimidation or unconsciousness, even without laceration of
the hymen, is deemed to be rape in our jurisprudence. It
would, in fine, be enough in a conviction for rape that there is
an entrance of the male organ within the labia of the
pudendum of the female organ. Neither the penetration of
the penis beyond the lips of the vagina nor the rupture of the
hymen is indispensable to justify conviction.
Consequently, the finding that Nias hymen is intact
does not disprove that rape was committed. Even the fact
that there was no reddening or hematoma in the external
genitalia does not render the occurrence of rape improbable.
The doctrine is well settled that the absence of external
injuries does not negate rape. Even Camilos claim that the
sperm found in the vagina of NIA was not his because he has
undergone vasectomy, is inconsequential. The absence of
spermatozoa is not an essential element of rape. This is
because in rape, the important consideration is not the
emission of semen but the penetration of the female genitalia
by the male organ.
On the issue of inconsistencies and discrepancies,
these things on minor matters neither impair the essential
integrity of the prosecutions evidence as a whole nor reflect
on the witness honesty. Such inconsistencies, which may be
caused by the natural fickleness of the memory, even tend to
strengthen rather than weaken the credibility of the witness
because they erase any suspicion of rehearsed testimony.

Camilo has moral ascendancy over Nia, being the


common-law spouse of her mother and the man who acted
as her father since she reached the age of reason. Nias
tender age and Camilos custodial control and domination
over her had rendered her so meek and subservient to his
needs and desires, thus, becoming an easy prey to Camilos
lecherous advances. Moreover, Camilo threatened her with a
knife.
For rape to exist it is not necessary that the force or
intimidation employed be so great or of such character as
could not be resisted. It is only necessary that the force or
intimidation be sufficient to consummate the purpose which
the accused had in mind. Intimidation must be viewed in the
light of the victims perception and judgment at the time of
the rape and not by any hard and fast rule. It is enough that it
produces fear that if the victim does not yield to the bestial
demands of the accused, something would happen to her at
the moment or thereafter, as when she is threatened with
death if she reports the incident. It is this form of intimidation
which explains why there are no traces of struggle which
would indicate that the victim fought off her attacker.
Under the Anti-Rape Law of 1997, any physical overt
act manifesting resistance against the act of rape in any
degree from the offended party, or where the offended party
is so situated as to render her/him incapable of giving valid
consent, may be accepted as evidence in the prosecution of
the acts punished in the new Article 266-A of the Revised
Penal Code.

People v. Ancheta (G.R. No. 70222)


Facts:
Juan Ancheta was charged in the Regional Trial Court
of Aparri, Cagayan, with the crime of robbery with arson,
committed in conspiracy with two other persons who could
not be tried with him because they were then at large. He
asks for a reversal of the decision convicting him of the crime
of arson and sentencing him to the maximum penalty of
reclusion perpetua plus civil indemnity in the sum of
P40,000.00 for the properties burned.
On 25 August 1980, at about 11 o'clock in the
evening, Ancheta and his two companions awakened Teresa
Gorospe, forced their entry into her house, demanded the
amount of P1,000.00, and burned her house when the money
was not delivered. Later, while the house was in flames, the
Ancheta, brandishing a bolo, prevented the people from
approaching and putting out the fire by warning them that he
had thirty companions.
Issues:
Whether or not there was conspiracy between and
among the accused?

Recidivism
Whether or not the penalty imposed on Ancheta was
proper?

People v. Dacillo (G.R. No. 149368)

Decision:

Facts:

On whether or not there was conspiracy between


and among the accused. The Supreme Court agreed that
there was a conspiracy among the accused-appellant and his
two companions when they forcibly entered the house of
Teresa Gorospe and burned it after their demand for
P1,000.00.

Appellant Dacillo together with Joselito Pacot were


indicted for murder in an information and that the
commission of the foregoing offense was attended by the
aggravating circumstance of abuse of superior strength.

A conspiracy exists when two or more persons come


to an agreement concerning the commission of a felony and
decide to commit it, whether they act through the physical
volition of one or all, proceeding severally or collectively. It is
settled that conspiracies need not be established by direct
evidence of acts charged, but may and generally must be
proved by a number of indefinite acts, conditions, and
circumstances which vary according to the purpose to be
accomplished. The very existence of a conspiracy is generally
a matter of inference deduced from certain acts of the
persons accused, done in pursuance of an apparent criminal
or unlawful purpose in common between them.
The conspiracy having been established, it should
follow that the accused-appellant is as guilty as his
companions of the crime of arson, even if it be conceded that
he was not the one who actually poured the kerosene and
ignited it to burn Teresa Gorospe's house. When there is a
conspiracy, the act of one is the act of all and visits equal guilt
upon every conspirator.
On whether or not the penalty imposed on Ancheta
was proper. Under Article 321 of the Revised Penal Code, the
penalty of reclusion temporal to reclusion perpetua shall be
imposed "if the offender shall set fire to any building,
farmhouse, warehouse, hut, shelter, or vessel in port,
knowing it to be occupied at the time by one or more
persons. .... "
The aggravating circumstance of nighttime was
correctly appreciated because it was sought by the
defendants to facilitate the commission of the offense and
their subsequent escape. Evident premeditation should also
have been applied because the offenders had deliberately
plotted the crime, as early as 9 o'clock of the night in
question, or two hours before they actually burned the
house.
With these aggravating circumstances and no
mitigating circumstances to offset them, the proper penalty
as imposed by the trial court is reclusion perpetua. The civil
indemnity of P40,000.00 is allowed, but the costs of the suit
shall be adjudged in toto against the accused-appellant and
not to be shared, as ordered by the trial court.

The case against appellants co-accused, Joselito


Pacot, was provisionally dismissed for lack of sufficient
evidence to identify him with certainty. Appellant was
arraigned on February 21, 2001 and, assisted by counsel,
pleaded not guilty. Pre-trial was conducted on March 1, 2001
and trial ensued thereafter.
When the body was discovered in the evening of
February 11, 2000, appellant immediately left for Cebu City,
arriving there the next day, February 12, 2000. He stayed in
Cebu City until his arrest the following year.
On May 31, 2001, the trial court rendered judgment
finding appellant guilty of murder and imposed upon him the
supreme penalty of death. The Court finds the accused
Francisco Dacillo, guilty beyond reasonable doubt of the
crime of murder for the death of Rosemarie Tallada, as
defined and penalized under Art. 248 of the Revised Penal
Code, as amended. Considering the aggravating circumstance
of recidivism with no mitigating circumstance to offset the
same, he is hereby sentenced to the extreme penalty of
death.
Issue:
Whether or not it is necessary, in recidivism as an
aggravating circumstance, to be alleged in the information?
Decision:
The Court, however, finds that the trial court erred
in imposing the death penalty on the ground that appellant
admitted during re-cross examination that he had a prior
conviction for the death of his former live-in partner. The fact
that appellant was a recidivist was appreciated by the trial
court as a generic aggravating circumstance which increased
the imposable penalty from reclusion perpetua to death.
In order to appreciate recidivism as an aggravating
circumstance, it is necessary to allege it in the information
and to attach certified true copies of the sentences previously
meted out to the accused. This is in accord with Rule 110,
Section 8 of the Revised Rules of Criminal Procedure which
states:
SEC. 8. Designation of the offense. - The complaint or
information shall state the designation of the offense given

by the statute, aver the acts or omissions constituting the


offense, and specify its qualifying and aggravating
circumstances. If there is no designation of the offense,
reference shall be made to the section or subsection of the
statute punishing it.
The aggravating circumstance of recidivism was not
alleged in the information and therefore cannot be
appreciated against appellant. Hence the imposable penalty
should be reduced to reclusion perpetua.

the mandatory penalty of death shall be


imposed.
Presidential Decree No.
1613
Sec. 5. Where
Death Results from Arson if by reason of
or on the occasion of the arson death
results, the penalty of reclusion perpetua to
death shall be imposed.

By Means of Inundation, fire, etc.


People v. Malngan (G.R. No. 170470)

Both laws provide only one penalty for the


commission of arson, whether considered destructive or
otherwise, where death results therefrom. The reason is that
arson is itself the end and death is simply the consequence.

On January 2, 2001, Edna, one hired as a housemaid


by Roberto Separa Sr. was accused of setting fire the house of
his employer resulted in the destruction of his employers
house and the death of six persons including his employer
Roberto Separa Sr., some seven adjoining residential houses,
were also razed by fire.

The case falls under simple arson since from a


reading of the body of the information it can be seen that it
states that the accused, with intent to cause damage, xxx
deliberately set fire upon the two-storey residential house, xxx
that by reason and on the occasion of the said fire, xxx which
were the direct cause of their death xxx. It is clear that her
intent was merely to destroy her employers house through
the use of fire.

Facts:

She was apprehended by the Barangay Chairman


and was brought to the Barangay Hall. She was then
identified by a neighbor, whose house was also burned, as
the housemaid of the Separas and upon inspection, a
disposable lighter was found inside accused-appellants bag.
Thereafter, accused-appellant confessed to the Barangay
Chairman.
On January 9, 2001, an information was filed before
the RTC of Manila, charging the accused-appellant with the
crime of Arson with multiple homicide. The RTC as well as the
Court of Appeals finds the accused guilty beyond reasonable
doubt of the crime of Arson with multiple homicide.

When fire is used with the intent to kill a particular


person who may be in a house and that objective is attained
by burning the house, the crime is murder only. When the
Penal Code declares that killing committed by means of fire is
murder, it intends that fire should be purposely adopted as a
means to that end. There can be no murder without a design
to take life. In other words, if the main object of the offender
is to kill by means of fire, the offense is murder. But if the
main objective is the burning of the building, the resulting
homicide may be absorbed by the crime of arson. The latter
being the applicable one in this case.

Issue:
Whether or not Edna Malngan was guilty of the
crime of destructive arson or simple arson?
Decision:
The crime committed by the accused-appellant is
Simple Arson and not Arson with Multiple Homicide. The
Supreme Court ruled that there is no complex crime of Arson
with Multiple Homicide. There are two laws that govern the
crime of arson where death results therefrom Article 320 of
the Revised Penal Code and Section 5 of Presidential Decree
1613, quoted hereunder, to wit:
Revised Penal Code
Art.
320.
Destructive Arson xxxx If as a consequence
of the commission of any of the acts
penalized under this Article, death results,

People v. Comadre (G.R. No. 153559)


Facts:
At around 7:00 oclock in the evening of August 6,
1995, Robert Agbanlog, Jimmy Wabe, Gerry Bullanday, Rey
Camat and Lorenzo Eugenio were having a drinking spree on
the terrace of the house of Roberts father, Jaime Agbanlog.
Jaime was seated on the banister of the terrace listening to
the conversation of the companions of his son.
As the drinking session went on, Robert and the
others noticed appellants Antonio Comadre, George Comadre
and Danilo Lozano walking. The three stopped in front of the
house. While his companions looked on, Antonio suddenly
lobbed an object which fell on the roof of the terrace.
Appellants immediately fled by scaling the fence of a nearby
school.
The object, which turned out to be a hand grenade,
exploded ripping a hole in the roof of the house. Robber

Agbanlog and his companions were hit by shrapnel and


slumped unconscious on the floor. They were all rushed to
the hospital for medical treatment. However, Robert
Agbanlog died before reaching the hospital for wounds
sustained which the grenade explosion inflicted. Roberts
companions sustained shrapnel injuries.

relationship with Antonio are insufficient to establish


conspiracy considering that they performed no positive act in
furtherance of the crime. There being no conspiracy, only
Antonio Comadre must answer for the crime.

The appellants were arrested the following day but


denied any participation in the incident, claimed they were
elsewhere when the incident occurred and that they had no
animosity towards the victims whatsoever.

People v. Labuguen (G.R. No. 127849)

After trial, the court a quo convicted appellants of


the complex crime of Murder with Multiple Attempted
Murder for having conspiring, confederating and mutually
helping one another, with intent to kill and by means of
treachery and with the use of an explosive.
Issue:
Whether or not the use of explosive qualifies the crime
to murder?
Whether or not appellants conspired to kill the victims?
Decision:
Yes, the killing by means of explosives qualifies the
crime to murder. The information alleges that both treachery
and the use of explosive attended the crime.
Since both circumstances can qualify the killing to
murder under Article 248 of the Revised Penal Code, the
Supreme Court held that when the killing is perpetrated with
treachery and by means of explosives, the latter shall be
considered as a qualifying circumstance. Not only does
jurisprudence support this view but also, since the use of
explosives is the principal mode of attack, reason dictates
that this attendant circumstance should qualify the offense
instead of treachery which will then be relegated merely as a
generic aggravating circumstance.
No, there was no conspiracy. The undisputed facts
show that when Antonio Comadre was in the act of throwing
the hand grenade, George Comadre and Danilo Lozano
merely looked on without uttering a single word of
encouragement or performed any act to assist him.
A conspiracy must be established by positive and
conclusive evidence. It must be shown to exist as clearly and
convincingly as the commission of the crime itself. Mere
presence of a person at the scene of the crime does not make
him a conspirator for conspiracy transcends companionship.
The evidence shows that George Comadre and Danilo
Lozano did not have any participation in the commission of
the crime and must therefore be set free. Their mere
presence at the scene of the crime as well as their close

Craft, Fraud or Disguise

Facts:
The deceased Bonifacio Angeles was engaged in
buying cows and selling them to the public market. One day,
the accused Vivencio Labuguen went to him and told him that
he knows of three big cows for sale and that the place where
they are is near. Believing on such declaration, he took
money from his cabinet at his house amounting to P40,000
and then drove in his motorcycle with the accused to see the
cows. On their way to see the cows, they have been seen
together by several witnesses who later on identified them in
court as the victim and the accused respectively. The accused
according to the witness was wearing a jacket and with a
handkerchief tied on his forehead. One of the witnesses, a
driver of a minibus testified that while driving on his way to
his destination, he saw a man behind the talahibs and he
noticed that he was wiping something from his head and right
face. It was the same man whom his conductor identified as
the one who stopped their bus and rode on it. His conductor
testified further that he noticed that the mans jacket was
soaked with blood including his pants and that he did not talk
when asked where he was headed to and instead just gave
his fare. The conductor even noticed that there was a lot of
money on the breast side pocket of his jacket and that one
bill was even falling. The man then alighted from the minibus
after reaching his destination without saying any word. Later
that afternoon, a news broke out that a mans body was
found dead in the middle of the ricefield. He was later on
identified as Bonifacio Angeles. Based on the strength of the
testimony of the witnesses, complaint and information were
filed against Vivencio and the Regional Trial Court found him
guilty of the crime of Robbery with Homicide and sentenced
with the penalty of death. The case was brought to the
Supreme Court for automatic review.
Issue:
Whether or not the court has correctly appreciated
the employment of generic aggravating circumstance of fraud
and craft in the commission of the crime even if not alleged in
the information?
Decision:
Though not alleged in the Information, the generic
aggravating circumstances of fraud and craft were properly
appreciated by the trial court. Craft involves intellectual
trickery and cunning on the part of the offender. When there
is a direct inducement by insidious words or machinations,

fraud is present. By saying that he would accompany the


victim to see the cows which the latter intended to buy,
appellant was able to lure the victim to go with him.

The court denied the motion and affirmed


appellants conviction for murder.
Issue:

Under Article 294 of the Revised Penal Code, the


penalty for Robbery with Homicide is reclusion perpetua to
death Applying Article 63 of the same Code, the imposable
penalty under the premises is death in view of the presence
of the aggravating circumstances of craft and fraud and the
absence of any mitigating circumstance.
Four members of the Court are steadfast in their
adherence to the separate opinion expressed in People vs.
Echegaray that Republic Act No. 7659 is unconstitutional
insofar as it prescribes the death penalty. However, they bow
to the majority opinion that the aforesaid law is
constitutional and therefore, the penalty prescribe
thereunder has to be imposed.

Abuse of Superior Strength

People v. Calpito (416 SCRA 491)


Facts:

Whether or not the abuse of superior strength


qualified the killing to murder?
Decision:
Yes. A perusal of the facts of the case readily reveals
that abuse of superior strength attended the crime. In
several cases, this Court has ruled that this circumstance
depends on the age, size and strength of the parties. It is
considered whenever there is a notorious inequality of forces
between the victim and the aggressor, assessing a superiority
of strength notoriously advantageous for the aggressor which
the latter selected or took advantage of in the commission of
the crime. In a recent case, it was held that an attack made by
a man with a deadly weapon upon an unarmed and
defenseless woman constitutes an abuse of the aggressors
superior strength. The circumstance must apply with more
reason in the present case, where the abuse of superior
strength is evident from the notorious disparity between the
relative strength of the victim, a 74-year-old unarmed
woman, and the assailant, a young man armed with a knife.

st

That on or about the 21 day of November, 1990,


appellant Francisco Calpito armed with a deadly weapon,
with intent to gain did, then and there willfully, unlawfully
and feloniously by means of violence and intimidation on the
person of Florentina Villas rob, take and carry away a
shoulder bag containing cash in the amount of P15,000 and
jewelries
amounting
to
P30,000
belonging
to Florentina Villas.
The appellant attack and stab with the said
weapon Florentina Villas and Israel Montilla inflicting wounds
on Florentina Villas which caused her death and a wound on
Israel Montilla which necessitated medical attendance on him
for a period of 5-7 days and which incapacitated him from
performing his usual work for the same length of time.
However, appellant entered a plea of not guilty and
waived pre-trial. But on June 15, 1993, appellant was rearraigned and after being appraised of the consequences of
the nature of his offense, he changed his plea to one of guilty.
The court a quo finding the charge of Robbery with
Homicide unsubstantiated by evidence, convicted appellant
of the crime of murder.
Appellant, thereafter, filed a Motion for
Reconsideration arguing that the trial court erred in
convicting him of Murder instead of Homicide and in failing to
apply the mitigating circumstance of minority.

Treachery
People v. Piliin (515 SCRA 207)
Facts:
On 19 November 1997, Rodrigo arrived at the gate
of his house aboard on an owner-type jeep he was driving.
His wife, Norma Zayenis (Norma), who was inside the house
at the time, went out to open the gate. When Rodrigo was
about to park his jeep, a man, later identified as Piliin,
suddenly approached him, poked his gun, and fired at him,
hitting the left side of his neck. Rodrigo fell unconscious and
the man quickly ran away. By reason of the gunshot wound,
the victim thereafter died. Piliin confesses killing Rodrigo and
implicated Yu and Caballes as his co-perpetrators. However,
After trial, appellant was found guilty for murder. The two
other accused, Yu and Caballes were acquitted for
insufficiency of evidence.
During the stage of appeal, Piliin argues that the
prosecution failed to establish the existence of treachery.
According to him, the witness failed to see the inception of
the attack because she was in the act of opening the gate for
her husband when the latter was shot. She lacked knowledge
of the attending circumstances prior to the shooting incident.
Hence, the trial courts finding of treachery becomes
speculative.
Issue:

Whether or not treachery must be appreciated as an


aggravating circumstance?
Decision:
Yes. There is treachery when the offender commits
any of the crimes against persons, employing means,
methods or forms in their execution, without risk to himself
arising from the defenses which the offended party might
make. To establish treachery, two elements must concur: (1)
that at the time of the attack, the victim was not in a position
to defend himself, and (2) that the offender consciously
adopted the particular means of attack employed. The
essence of treachery is the unexpected and sudden attack on
the victim which renders the latter unable and unprepared to
defend himself by reason of the suddenness and severity of
the attack. Appellants wife witnessed the incident from its
inception up to its consummation.
In this case, the victim was about to park his car
when appellant suddenly appeared and shot him without any
warning. The attack was so sudden that the latter had no
opportunity to repel it or defend himself. It can readily be
inferred that the manner of the attack adopted by appellant
manifested treachery. Furthermore, as correctly observed by
the Solicitor General, the weapon used and the nature of the
injury inflicted, which pertained to the lone gunshot fatally
wounding appellant, established that appellant deliberately
and consciously adopted the particular mode of attack to
ensure the commission of the offense with impunity.

Ignominy

Aid of Minor or By Means of Motor Vehicles


People v. Mallari (404 SCRA 170)
Facts:
Based on the accusatory portion of the Information
filed against Rufino Mallari, he was accused of hitting and
bumping one Joseph Galang with an Isuzu Canter Elf truck on
or about July 7, 1996. The evidence for the prosecution
showed that the said incident was preceded by an altercation
between Rufino Mallari and Joseph Galang when the latter
admonished the former not to drive fast while passing by the
latters house. To end the situation Joseph, together with his
brothers, who were also present at that time, asked for
apology from Rufino. However, the conflict did not end there
because when dusk came and while Joseph was watching
basketball game with his wife, Rufino arrived with some
companions and attacked Joseph with bladed weapons. They
chased him and when Joseph was able to run away, Rufino
pursued him with the use of the Isuzu Canter Elf truck. When
he caught up with him, he bumped him which resulted in his

instant death. The doctor who conducted the medico-legal


inspection of the cadaver testified that Josephs cause of
death was crushing injury on the head secondary to
vehicular accident. The trial court found Rufino liable with
murder and sentenced with the penalty of death after
considering the qualifying circumstance of use of motor
vehicle in committing the crime. The case was brought to the
Supreme Court pursuant to the requirement of automatic
review of cases penalized with death penalty based on Article
47 of the Revised Penal Code. Rufino argued that the use of a
motor vehicle was only incidental, considering that he
resorted to it only to enable him to go after Joseph after he
failed to catch up with the latter.
Issue:
Whether or not the qualifying circumstance of use of motor
vehicle was correctly appreciated by the trial court in
imposing the death penalty?

Decision:
The evidence shows that Rufino deliberately used his
truck in pursuing Joseph. Upon catching up with him, Rufino
hit him with the truck, as a result of which Joseph died
instantly. It is therefore clear that the truck was the means
used by Rufino to perpetrate the killing of Joseph.
The case of People v. Muoz cited by Rufino finds no
application to the present case. In the said case, the police
patrol jeep was merely used by the accused therein in looking
for the victim and in carrying the body of the victim to the
place where it was dumped. The accused therein shot the
victim, which caused the latters death. In the present case,
the truck itself was used to kill the victim by running over
him.
Under Article 248 of the Revised Penal Code, a
person who kills another by means of a motor vehicle is
guilty of murder. Thus, the use of motor vehicle qualifies the
killing to murder. The penalty for murder is reclusion
perpetua to death. Since the penalty is composed of two
indivisible penalties, we shall apply Article 63(3) of the
Revised Penal Code, which reads:
3. When the commission of the act is attended by some
mitigating circumstances and there is no aggravating
circumstance, the lesser penalty shall be applied.
In the present case, the aggravating circumstances of
evident premeditation and treachery, which were alleged in
the information, were not proved. What was proved was the
mitigating circumstance of voluntary surrender through the
testimonies of Rufino and Myrna, which were not rebutted by
the prosecution.
In view of the absence of an aggravating
circumstance and the presence of one mitigating

circumstance, reclusion perpetua, not death, should be the


penalty to be imposed on Rufino.

People v. Enguito (326 SCRA 508)


Facts:
On or about September 22, 1991, Felipe Requerme
was driving a motorela, together with his wife Rosita and
another passenger, Engr. Wilfredo Achumbre, who is the
deceased in this case. The deceased was picked up by them
on their way home and requested them to bring him to his
house. While on their way, a white vehicle, which was later
on identified as a Ceres Kia automobile bearing Plate No. 722,
intentionally hit and pushed the motorela that they were
riding and violently kept pushing it causing it to turn around
facing the direction from where it came from and fell on its
right side. Rosita testified that while she was struggling out
of the motorela she noticed that the white vehicle went up
the elevated catwalk or pathway pursuing Achumbre who
was hit when he was already at the railing (barandilla). Then
she observed that the white vehicle drove away without even
caring to see what happened to them. The spouses/victims
were brought to the police station while the Achumbre was
brought to the hospital who was declared dead on arrival. It
was later on found out upon investigation that said incident
was predicated on the earlier fight which transpired between
Achumbre and the driver of the motor vehicle, Thadeos
Enguito, the accused in this case. As a result of the death of
Achumbre, his wife filed a criminal complaint against the
accused. The Regional Trial Court found him guilty with the
crime of Homicide with Less Serious Physical Injuries, taking
into consideration the aggravating circumstance of use of
motor vehicle which was alleged in the information. On
appeal to the Court of Appeals, the latter modified the crime
to Murder due to the aggravating circumstance. The accused
went to the Supreme Court imputing error on the decision of
the Court of Appeals with respect to the declaration of the
crime of Murder against him on the ground that he did not
intentionally choose the motor vehicle he was driving as a
means of committing the offense, and that at most, the
vehicle was the only available means to stop the deceased
from escaping. He argued that it was his intention to
apprehend and surrender the deceased to the police for his
previous act of mauling him but in the process, he killed the
deceased.

Issue:
Whether or not the aggravating circumstance of use
of motor vehicle should be considered in this case?
Decision:
The indictment against accused-appellant is murder
attended by the use of motor vehicle. The use of a motor

vehicle qualifies the killing to murder if the same was


perpetrated by means thereof. Appellant's claim that he
merely used the motor vehicle, Kia Ceres van, to stop the
victim from escaping is belied by his actuations. By his own
admission, he testified that there was a police mobile patrol
near the crossing. Accused-appellant could have easily sought
the assistance of the police instead of taking the law into his
own hands. Moreover, accused-appellant already noticed the
deceased trying to jump out of the motorela but he still
continued his pursuit. He did not stop the vehicle after hitting
the deceased[16] who was hit when he (Achumbre) was at
the railing of the Marcos bridge. Accused-appellant further
used the vehicle in his attempt to escape. He was already
more than one (1) kilometer away from the place of the
incident that he stopped his vehicle upon seeing the police
mobile patrol which was following him.
Appellant contends that he should have been
convicted of the crime of homicide with two (2) mitigating
circumstances of acting in passion and voluntary surrender;
and had the charge been homicide he could have pleaded
guilty. We find that these mitigating circumstances cannot be
appreciated in his favor. Accused-appellant was allegedly
"still very angry" while he was following, bumping and
pushing the motorela which was in front of him. He was
previously mauled by the deceased and he was allegedly
rendered unconscious by the blows inflicted on him. When he
regained consciousness, he claims that he wanted to look for
a policeman to report that he was mauled. Clearly, accusedappellant's state of mind after he was mauled and before he
crushed Achumbre to death was such that he was still able to
act reasonably. In fact, he admitted having seen a police
mobile patrol nearby but instead, he chose to resort to the
dastardly act which resulted in the death of Achumbre and in
the injuries of the spouses Requerme. For passion to be
considered as a mitigating circumstance, facts must be
proved to show causes sufficient to produce loss of selfcontrol and to overcome reason. The turmoil and unreason
which naturally result from a quarrel or fight should not be
confused with the sentiment or excitement in the mind of a
person injured or offended to such a degree as to deprive him
of his sanity and self-control.
The mitigating circumstance of voluntary surrender
cannot be appreciated. Evidence shows that accusedappellant was further pursued by the police. Appellant
himself testified that he stopped his vehicle just after the
police mobile stopped but admitted having "stopped farther
than the police mobile". SPO3 Catiil further testified that
appellant did not surrender but only stopped his vehicle
when its right tire was already flat. His testimony was
corroborated by PO3 Makiling who was patrolling the portion
of Marcos Bridge. He testified that he saw the vehicle being
driven by accused-appellant already destroyed and the right
portion of the vehicle a little bit lower as it was running flat.
Clearly, accused-appellant could have eluded arrest but his
situation became futile when his vehicle suffered a flat tire.

The foregoing notwithstanding, the existence or


non-existence of a mitigating circumstance in the case at bar
will not affect the penalty to be imposed pursuant to Article
63 of the Revised Penal Code. The crime committed by
accused-appellant is the complex crime of murder with less
serious physical injuries. Under Article 48 of the Revised
Penal Code, the penalty for a complex crime shall be the
maximum period of the penalty for the most serious crime.
The crime was committed in 1992 where the penalty for the
crime of murder, which is the most serious crime, was
reclusion temporal in its maximum period to death under
Article 248 of the Revised Penal Code. The death penalty
being the maximum period of the penalty for murder should
be imposed for the complex crime of murder with less serious
physical injuries considering that under Article 63, an
indivisible penalty cannot be affected by the presence of any
mitigating or aggravating circumstance. And, consonant with
the ruling in People vs. Muoz that Article III, Section 19 (1) of
the 1987 Constitution did not change the period of the
penalty for murder except only insofar as it prohibits the
imposition of the death penalty and reduces it to reclusion
perpetua, the Court of Appeals was correct in imposing the
penalty of reclusion perpetua.

victim, according to Jacalne, by clubbing the victim first with


the wooden stick, and then cutting his head and his penis
with a knife.
Another withness, Ireneo Acierto, appellants
brother-in-law, testified that while he was resting in his house
at past 11:30 in the morning of July 7, 1997, he heard
someone screaming. When he looked out from his window,
he saw that the person screaming was his sister-in-law, Ana.
He went out of the house and went near the porch of the
Guerreros, where he saw Ernesto Ocampos head about to be
severed by appellant. When the head was cut off, appellant
placed the same on the right side of the victims trunk. After
that, appellant cut off Ernestos penis. Ireneo noticed that
while the head was being severed, the victim was lying down
on the floor, but not moving. Ireneo then told appellant, That
is enough, bayaw. Stop it. According to the witness, his wife
Ana was also saying, That is enough, Manong. Appellant
angrily turned to Ireneo, telling him not to interfere or else he
might also be implicated. Ireneo hurriedly went away after
that. Ireneo did not see his father-in-law, Dino, at the time of
the incident and did not know where Dino was.

Cruelty

The trial court convicted Orlando Guerrero, Jr. of


murder while his father Dino was acquitted.

People v. Guerrero (389 SCRA 389)

Issue:

Facts:

Whether or not the court a quo gravely erred in appreciating


the qualifying circumstance of cruelty and/or outraging and
scoffing the corpse in order to classify the killing as murder
despite failure of the prosecution to allege the same in the
information

Orlando Guerrero, Jr., also known as Pablo, together


with his father Orlando Guerrero, Sr., nicknamed Dino, was
accused of murder. The accuseds, conspired, confederated
and mutually helped one another, with deliberate intent to
kill and with evident premeditation and treachery, did then
and there willfully, unlawfully and feloniously and without
justifiable cause, attack, assault, club, beheaded and cut off
the penis of the victim Ernesto Ocampo, which caused his
death thereafter, to the damage and prejudice of his lawful
heirs.
Upon arraignment, both pleaded not guilty. Orlando
interposed self-defense while his father, Dino, denied any
complicity in the killing.
According to the the witness, Jacalne, he was
informed that one Dino Guerrero was inside the house
nearby. Dino Guerrero came out with his hands extended
forward. SPO1 Emilio Taracatac immediately frisked and
handcuffed him. Before Dino was handcuffed, according to
the witness, he said that it was his son who had killed the
victim. Thereafter, Dino was brought to the police station for
custodial investigation.
Further, Jacalne testified that appellant Orlando
Guerrero, Jr., was not at the scene of the crime during their
investigation. But upon their return to the police station,
appellant was already there.[11 Appellant admitted killing the

Decision:
The information alleges the qualifying circumstances
of (1) treachery and (2) evident premeditation. It also states
that there was cruelty in the perpetration of the crime, where
there was deliberate and inhuman suffering of the victim and
the offender had scoffed at the victims corpse.
On treachery and evident premeditation, the trial
court found that the evidence adduced by the prosecution
fell short of the requirements of the law.[ we hold that in the
present case, the trial court did not err when it found neither
treachery nor evident premeditation. However, the trial court
found there was cruelty as well as outraging or scoffing at the
corpse, thus, qualifying the crime to murder.
Simangan v. People (434 SCRA 38)
Facts:
On February 10, 1980 at about 8
pm, Simangan and four other men wearing
fatigues knocked on the door of the store
owned by Ernesto and Sofronia. The couple

was having dinner with their daughter


Lorna. Simangan asked Ernesto to guide
them on the road as they were not familiar.
Ernesto agreed, he then ordered his
houseboy Romeo to accompany him in
guiding the group of Simangan. The next
morning, Romeo reported to Sofronia that
Ernesto is dead. Ernesto was found near a
creek, he sustained 10 stab wounds.

the supposed aggravating circumstance of dwelling cannot be


appreciated as there was no trespass to the sanctity of the
house of the victim on the part of Calonqui, while the
aggravating circumstance of relationship is likewise cannot go
against Calongui, even as an alternative circumstance, as
being first cousins is not within the concept contemplated in
Article 15 of the Revised Penal Code. However, his conviction
is nonetheless affirmed.

Issue:

People v. Marcos (G.R. No. 132392)

Whether or not Simangan is guilty


beyond reasonable doubt.

Facts:

Decision:
Yes. The testimonies of Romeo and
Sofronia are credible. Thus, Simangans
conviction is affirmed. It is found that
Simangan stabbed Ernesto 10 times, three
of which were fatal. But the number of stab
wounds does not qualify as an aggravating
circumstance against Simangan for it must
be proven that Simangan intended to
exacerbate the suffering of Ernesto. Nigh
time is also not appreciated as it was
included in the original information.

Prosecution, with the testimony of a lone eyewitness, who happened to be the son of the victim, along
with the admission of guilt, found Cesar Marcos guilty beyond
reasonable doubt of Murder for the killing of his elder
brother, Virgilio, as aggravated by the qualifying circumstance
of evident premeditation.
During Appeal, the Solicitor General insisted that since the
accused is a brother of the victim, the alternative
circumstance of relationship must be considered in
determining the imposable penalty.
Issue:
Whether or not the alternative circumstance of
relationship shall be considered in the imposition of the
proper penalty?

Art. 15: Alternative Circumstances

Decision:

Relationship

In order that the Alternative Circumstance of


relationship may be taken into consideration in the
imposition of the proper penalty, Paragraph 2 of Article 15 of
the Revised Penal Code provides that the offended party
must either be the (a) spouse, (b) ascendant, (c) descendant,
(d) legitimate, natural or adopted brother or sister, or (e)
relative by affinity in the same degree, of the offender.
Relationship is Mitigating in Crimes against Property, while it
must be considered as Aggravating in Crimes against Persons
where the offended party is a relative of a higher degree than
the offender or when in the same degree or level, as in
brothers.

People v. Calongui (G.R. No. 170566)


Facts:
Calonqui was found guilty for two counts of rape. On
January 1, 1998 about 2 am in Tagbong, Camarines Sur,
Calonqui was able to rape the 13 year old girl Maricel in the
latters house. On September 26, 1998 at about three in the
morning, the accused again raped the victim. Both rape
incidents were witnessed by the brother of Maricel.
Issue:
Whether or not the aggravating circumstances of
dwelling and relationship be appreciated against Calonqui
and the latter circumstance as an alternative circumstance?

Decision:
Calonqui and Maricel live under the same shelter as
they are first cousins. At the time of the incident, both are
living in the same house and in the same room. Therefore,

Therefore, the Alternative Circumstance


relationship shall be considered as Aggravating.

Intoxication

Arts. 16-20: Persons Criminally Liable for Felonies


Principals

of

People v. Vasquez (G.R. No. 123939)


Facts:
The appellant drove the passenger jeepney with his
cohorts on board looking for Luable and Geronimo. When the
appellant saw the two going in the opposite direction, the
appellant drove the vehicle and sideswiped Geronimo. And
when Geronimo fled, the appellant, armed with a bolo,
pursued him. When the appellant failed to overtake the
victim, he returned to the passenger jeepney and drove it to
where his cohorts ganged up on the victim. The appellant
urged them on to kill Geronimo. Thereafter, he left the scene
along with his cohorts, leaving the hapless Geronimo mortally
wounded.
After trial, the court rendered judgment acquitting
Ramon, but convicting the appellant of murder for the killing
of Geronimo, and attempted homicide for attempting to kill
Luis.
The appellant avers that he and his brother Ramon
had no motive to kill Geronimo. The appellant contends that
the witnesses for the prosecution were not in agreement as
to who killed Geronimo. The appellant noted that according
to the testimony of the witness, the appellant stayed in the
jeepney and merely yelled to his companions who ganged up
on Geronimo, "Sige patayin ninyo, patayin ninyo na, at huwag
ninyong iwanang buhay!"
The appellant further posits that the prosecution
witnesses were not even in accord as to where Geronimo was
stabbed to death. The appellant argues that because of the
inconsistencies in the testimonies of the witnesses of the
prosecution, it failed to prove his guilt beyond reasonable
doubt of the crimes charged. Hence, he should be acquitted
of the said charges.

conspirators by being present at the scene of the crime, or by


exerting moral ascendancy over the rest of the conspirators
as to move them to executing the conspiracy."
The Supreme Court, likewise, stressed that where
there are several accused and conspiracy has been
established, the prosecution need not pinpoint who among
the accused inflicted the fatal wound. Where conspiracy has
been established, evidence as to who among the accused
rendered the fatal blow is not necessary. All the conspirators
are liable as co-principals regardless of the intent and
character of their participation because the act of one is the
act of all.
Article 8 of the Revised Penal Code provides that
there is conspiracy when two or more person agree to
commit a felony and decide to commit it. Conspiracy need
not be proven by direct evidence. It may be inferred from the
conduct of the accused before, during and after the
commission of the crime, showing that they had acted with a
common purpose and design. Conspiracy may be implied if it
is proved that two or more persons aimed by their acts
towards the accomplishment of the same unlawful object,
each doing a part so that their combined acts, though
apparently independent of each other were, in fact,
connected and cooperative, indicting a closeness of personal
association and a concurrence of sentiment. Conspiracy once
found, continues until the object of it has been accomplished
and unless abandoned or broken up. To hold an accused
guilty as a co-principal by reason of conspiracy, he must be
shown to have performed an overt act in pursuance or
furtherance of the complicity. There must be intentional
participation in the transaction with a view to the furtherance
of the common design and purpose. Each conspirator is
responsible for everything done by his confederates which
follows incidentally in the execution of a common design as
one of its probable and natural consequences even though it
was not intended as part of the original design.

Issue:
Whether the trial court erred in convicting the
appellant when the witnesses testimony didnt confirm who
chased and stabbed the victims?
Decision:
Whether Domingo Vasquez chased the deceased
with a bolo was averred by Luis Luable or whether the
accused merely incited his companions in the jeepney to kill
the deceased as averred by Luisa Abellanosa, is immaterial in
the determination of his liability because a conspiracy among
the occupants of the jeepney has been established.
In order to hold an accused guilty as co-principal by
reason of conspiracy, it must be established that he
performed an overt act in furtherance of the conspiracy,
either by actively participating in the actual commission of
the crime, or by lending moral assistance to his co-

Responsibility of a conspirator is not confined to the


accomplishment of a particular purpose of conspiracy but
extends to collateral acts and offenses incident to and
growing out of the purpose intended. Conspirators are held
to have intended the consequences of their acts and by
purposely engaging in conspiracy which necessarily and
directly produces a prohibited result, they are, in
contemplation of law, chargeable with intending that result.
Conspirators are necessarily liable for the acts of another
conspirator unless such act differs radically and substantively
from that which they intended to commit. When a
conspirator embarks upon a criminal venture of indefinite
outline, he takes his chances as to its content and
membership, so be it that they fall within the common
purposes as he understands them."
All the foregoing constitutes evidence beyond cavil
of conspiracy between the appellant and the principals by
direct participation. The appellant is, thus, criminally liable for

the death of the victim, although there is no evidence that he


did not actually stab the latter.

palpable mistake or no such admission was in fact made.


There was never any such disclaimer by appellant.

People v. Dacillo (G.R. No. 149368)

Moreover,
despite
appellants
self-serving,
exculpatory statement limiting his involvement in the crime,
all circumstances pointed to his guilt. Assuming for the sake
of argument that Pacot was the mastermind, appellants
admission that he participated in its commission by holding
Rosemaries legs made him a principal by direct participation.

Facts:
Appellant was convicted by the trial court of the
crime of murder for the death of Rosemarie Tallada, with
aggravating circumstance of recidivism with no mitigating
circumstance to offset the same, and sentenced to the
extreme penalty of death.
In his defense, appellant admitted complicity in the
crime but minimized his participation. Appellant alleged that
he only held down Rosemaries legs to prevent her from
struggling and, after the latter was killed by another man he
identified as Joselito Pacot, he encased the corpse in cement.
He claimed that Pacot, a co-worker at Davao Union
Cement Corporation (DUCC), was looking for a house where
he and his girlfriend Rosemarie could spend the night. He
offered his brothers house which was under his care. In the
evening of February 6, 2000, he and Joselito Pacot brought
Rosemarie to the house at Purok No. 3, New Society Village,
Ilang, Davao City.
After accompanying the couple there, he went home
to take supper. Later that evening, he returned to the house
with the bottle of Sprite Pacot had ordered. When he arrived,
Pacot and Rosemarie were already grappling with each other
and Pacot was strangling the girl. He told Pacot to stop but
instead of heeding him, the latter ordered him to close the
door. Pacot told appellant that he was going to be implicated
just the same so he closed the door as ordered and helped
Pacot "(hold) the feet of the woman" as "her feet kept hitting
the walls."

Two or more persons taking part in the commission


of a crime are considered principals by direct participation if
the following requisites are present: 1.) they participated in
the criminal resolution and 2.) they carried out their plan and
personally took part in its execution by acts which directly
tended to the same end.
Both requisites were met in this case. Two or more
persons are said to have participated in the criminal
resolution when they were in conspiracy at the time of the
commission of the crime. To establish conspiracy, it is not
essential that there be proof of the previous agreement and
decision to commit the crime, it being sufficient that the
malefactors acted in concert pursuant to the same objective.
It is well-settled that a person may be convicted for
the criminal act of another where, between them, there is
conspiracy or unity of purpose and intention in the
commission of the crime charged. Conspiracy need not be
proved by direct evidence of prior agreement on the
commission of the crime as the same can be inferred from
the conduct of the accused before, during, and after the
commission of the crime showing that they acted in unison
with each other pursuant to a common purpose or design.
Accomplices
Abarquez v. People (G.R. No. 150762)

The two men stopped only when Rosemarie was


already motionless. Pacot wanted to dump the body into the
sea but appellant told him it was low tide. Appellant then
suggested that they entomb the body in cement for which
Pacot gave appellant P500. Pacot left the house at dawn the
following day, February 7, 2000. At past 10:00 a.m., appellant
brought the concrete mixture and cast the dead body in
cement.

Issue:

Facts:
The prosecution charged Abarquez with the crimes
of homicide and attempted homicide alleging in the two
informations filed that said accused was conspiring and
confederating with one Alberto Almojuela in the killing of
Ricardo Quejong Bello, by stabbing him twice with a bladed
weapon and hitting him with a gun at the back.
The trial court found Abarquez guilty beyond
reasonable doubt as an accomplice in the crime of homicide.

Whether or not appellant is liable as a principal?


Decision:
The rule is that any admission made by a party in the
course of the proceedings in the same case does not require
proof to hold him liable therefor. Such admission may be
contradicted only by showing that it was made through

Abarquez filed an appeal to the Court of Appeals.


However the Court of Appeals rejected Abarquezs allegation
that he was merely at the crime scene to pacify the
quarreling parties.
Abarquez alleges that the prosecutions evidence
does not satisfy the test of moral certainty and is not

sufficient to support his conviction as an accomplice. He


further alleges that there was a misapprehension of facts and
that the trial court and the Court of Appeals reached their
conclusion based entirely on speculation, surmises and
conjectures. Abarquez also assails the credibility of the
witnesses against him.

Abarquezs act of trying to stop Paz does not translate to


assistance to Almojuela.

Issue:
Whether or not there is sufficient evidence to prove
that fact that Abarquez was an accomplice in the killing of
Ricardo Bello?

Accessories

Decision: No.

People v. Tolentino (G.R. No. 139179)

Two elements must concur before a person becomes liable as


an accomplice:

Facts:

(1) community of design, which means that the accomplice


knows of, and concurs with, the criminal design of the
principal by direct participation; and
(2) the performance by the accomplice of previous or
simultaneous acts that are not indispensable to the
commission of the crime. Mere commission of an act, which
aids the perpetrator, is not enough.
The cooperation that the law punishes is the
assistance knowingly rendered, which cannot exist without
the previous cognizance of the criminal act intended to be
executed. It is therefore required in order to be liable as an
accomplice; that the accused must unite with the criminal
design of the principal by direct participation.
The court held in one case that the mere presence of
the accused at the crime scene cannot be interpreted to
mean that he committed the crime charged.
In convicting Abarquez in this case, the trial court
and the Court of Appeals relied mainly on the testimony of
Paz. Paz testified that he was held by Abarquez on the
shoulders, thus preventing him from helping Quejong who
was grappling with Almojuela.
Pazs testimony does not show that Abarquez
concurred with Almojuelas criminal design. "Tumigil" literally
means "stop." Clearly, Abarquez was trying to stop Paz from
joining the fray, not from helping Quejong. Paz claims that he
was only trying to talk to Almojuela. However, Paz could not
have been merely talking to Almojuela, as he tried to portray,
because Almojuela was already grappling with Quejong at
that time. Paz interpreted Abarquezs action as an attempt to
prevent him from helping Quejong. His interpretation was
adopted by the trial court and sustained by the Court of
Appeals. Yet, in his testimony, Paz admitted that while
restraining him, Abarquez was scolding or reprimanding him
and telling him to stop. It was not shown that Abarquez was
stopping Paz from helping Almojuela. It is more likely that
Abarquez was trying to stop Paz from joining the fight.

On February 28, 1996 appellant Jonathan Fabros and


his cousins, Sheila Guilayan and Merwin Ledesma, were at
their house in Luyahan, Pasonanca, Zamboanga City when
their neighbor Wilfredo Tolentino called them. When asked
what it was all about, Wilfredo simply motioned to them to
come to his house located just across the road. Once they
were inside the house, Wilfredo immediately revealed his
plan to kill Hernan Sagario, Sheila's stepfather. Wilfredo
explained that it was the only way to free Sheila's mother appellant's aunt - of the sufferings being caused by Hernan.
Wilfredo then instructed Merwin to go back to the house and
get the bolo of Hernan. Merwin obliged, got the bolo, and
gave it to Wilfredo. Thereafter, they were told by Wilfredo to
go home and wait for Hernan.
Around 8:30 in the evening, Hernan arrived. He went
directly to the kitchen and fixed the bag of rice he was
carrying. Jonathan together with Sheila and Merwin, just
stayed quiet in the living room.Later, Wilfredo with a 2"x2"
piece of wood in his hand entered the house. He then
followed Hernan towards the kitchen. When about an
armslength away from Hernan, Wilfredo, immediately
walloped Hernan on the right side of the neck sending the
latter unconscious and falling face down to the ground.
Wilfredo immediately instructed appellant and Merwin to
help him bring Hernan out of the house. Lifting Hernan out of
the house, Wilfredo held him by the neck while both
appellant and Merwin grasped his feet. They then carried
Hernan towards the creek, upon reaching the creekside, the
three stopped, then Wilfredo successively stabbed Hernan on
different parts of the body causing the latter's instant death.
After throwing the victim's lifeless body in the creek, the
three immediately left. Tolentino called Jonathan, Sheila and
Merwin and warned them that if they will tell other people,
he will kill them. Out of fear, they just followed whatever
Tolentino told them.
On 01 March 1996, however, Jonathan was arrested
for the death of Hernan Sagario. Accused Jonathan Fabros
and Wilfredo Tolentino both denied killing the victim. Instead,
they pointed to each other as the one who killed Hernan

Sagario. Fabros pointed to Tolentino as the assailant and the


latter also fingered the former as the killer of Sagario.
However, on 14 July 2000, long after the trial court's
decision had become final and executory on his part,
Wilfredo Tolentino, apparently conscience-stricken, executed
an affidavit admitting sole responsibility for the death of
Hernan Sagario and retracted his testimony implicating
accused-appellant Jonathan Fabros.
The trial court held that the prosecution's evidence
positively identified Wilfredo Tolentino as the person who
had hit the victim with a piece of wood and later stabbed him
with a bolo. It also ruled that the killing was qualified by
treachery and attended by the aggravating circumstance of
dwelling.
The court a quo observed that overt and positive
acts of appellant (Jonathan Fabros) manifested his approval
of the killing and the concurrence of his acts with those of the
8
other accused. Thus, the RTC concluded that Fabros was a
co-conspirator and should be held equally responsible for the
murder.
Hence, this appeal.
Issue:
Whether or not appellant (Jonathan Fabros) should
be convicted as an accessory?
Decision:
Appellant cannot be convicted as an accessory.
Article 19 of the Revised Penal Code defines an accessory as
one who had knowledge of the commission of the crime and
did not participate in its commission as principal or
accomplice, yet took part subsequent to its commission by
any of three modes: (1) profiting oneself or assisting the
offender to profit by the effects of the crime; (2) concealing
or destroying the body of the crime, or the effects or
instruments thereof, in order to prevent its discovery; and (3)
harboring, concealing, or assisting in the escape of the
principals of the crime, provided the accessory acts with
abuse of his public functions or when the offender is guilty of
treason, parricide, murder, or an attempt to take the life of
the Chief Executive, or is known to be habitually guilty of
some other crime. To convict an accused as an accessory, the
following elements must be proven: (1) knowledge of the
commission of the crime and (2) subsequent participation in
it by any of the three above-cited modes.
Under paragraph 2 of said codal provision, the
concealment or the destruction of the body of the crime or of
the effects or the instruments thereof must have been done
in order to prevent the discovery of the crime. That, precisely,
is wanting in the present case.

In his testimony, appellant stated that because he


was afraid his co-accused would hurt him if he refused, he
agreed to assist the latter in carrying the victim towards the
river. The fact that appellant left thereafter likewise indicated
his innocence of the charge. Verily, he adequately explained
his conduct prior to the stabbing incident as one born of fear
for his own life. It is not incredible for an eyewitness to a
crime, especially if unarmed, to desist from assisting the
victim if to do so would put the former's life in peril.
The presumption of innocence in favor of appellant
has not been overcome by proof beyond reasonable doubt.
Thus, he must be acquitted.

People v. Cui (G.R. No. 121982)


Facts:
In the evening of December 5, 1990, ten (10) armed
robbers raided the compound of Johnny and Rose Lim on
Edison Street, Lahug, Cebu City. The Lims, their three (3)
children, and the employees of the family-owned business,
were able to see the faces of the leader Wilfredo alias "Toto"
Garcia and two of his men, Mawe Garcia and a certain Edgar.
The other robbers could not be identified as they had flour
sacks over their heads. The robbers carted away cash and
jewelries worth twenty thousand pesos (P20,000.00). They
also blindfolded and forcibly abducted seventeen (17) year
old Stephanie, the youngest daughter of the Lims. They
demanded a ransom of one million pesos (P1,000,000.00) for
her release.Johnny Lim turned over to Toto Garcia the
ransom amount in the afternoon of the next day at an
arranged meeting place. Stephanie, in turn, was released to
her father.
Initially, the Lims kept the crime a secret. But on the
third day, they reported the kidnapping to the Philippine
National Police Cebu Metropolitan District Command (Cebu
Metrodiscom). The Metrodiscom Intelligence Security Team
(MIST) conducted an investigation and Johnny Lim identified
one of the suspects as Toto Garcia.
Toto Garcia was known as the leader of a group of
armed robbers called the Baong Gang. The gang's base of
operation was pinpointed at Quiot, Pardo, Cebu. When the
police learned that Eduardo Basingan, hailed from Quiot,
Pardo, Cebu City, they decided to interrogate him.
Upon Basingan's interrogation, he identified Toto
Garcia, Mawe Garcia and Edgar as the three (3) who did not
wear masks, Sadam and Rey as the two (2) who held him and

the Lims at gunpoint, and Tata Garcia, Yul Alvarez, a certain


Benjie, a certain Leos and a certain Laring as the look-outs
who stayed outside the Lim compound. He named Toto
Garcia as the chief plotter of the crime at bar, and revealed
that his neighbor and close family friends, the spouses
Leonilo and Beverly Cui, participated in the plan. Basingan
said he was asked to join the plot and was assured that he
would not be under suspicion because he would be placed at
gun point together with the other members of the Lim
household when the crime is committed. However, he
refused to join the plot during the December 2, 1990 meeting
of the group at the residence of the Cuis in Quiot, Pardo,
Cebu City. Leonilo Cui even invoked their close ties as
godfathers of each other's children but he was unmoved. At
the meeting were Toto Garcia, Mawi Garcia, Edgar, Rey,
Sadam and the Cuis.
On December 18, 1990, Basingan executed a sworn
statement reiterating these revelations in writing. Johnny and
Rose Lim then formalized their complaint by executing a Joint
Affidavit. Assistant Prosecutor Bienvenido N. Mabanto, Jr.
filed an information for Kidnapping with Ransom against
Basingan, the Cuis, and the members of the group of Toto
Garcia as identified by Basingan in his sworn statement.On
the same day, Basingan and Leonilo Cui were arrested.
On March 14, 1991, Joselito "Tata" Garcia, Hilaria
Sarte and her live-in partner, Luis Obeso, referred to by
Basingan as "Laring" and "Leos", respectively, were arrested
in the neighboring Negros Island. The next day, however, Tata
Garcia died due to "hemorrhage, severe, secondary to
gunshot wounds." Upon presentation of his death certificate,
the trial court ordered his name deleted from the
information.
After preliminary investigation, Prosecutor Manuel J.
Adlawan found that the participation of the Cuis was only
that of accomplices amended the Information downgrading
the charge against the Cuis as mere accomplices in the
kidnapping with ransom of Stephanie Lim.
On May 15, 1991, Basingan, the Cuis, Obeso and
Sarte were arraigned and they all pleaded not guilty. On June
27, 1991, Basinga escaped from the prison. Trial on the merit
ensued against the Cuis, Obeso and Sarte. Basingan was tried
in absentia.
On
February
13,
1992,
Beinvenido
Nacario, alias "Rey Nacario", was arrested. On arraignment
on April 13, 1992, he pleaded not guilty. However, on May 5,
1991, he, too, escaped from detention and remains at large
to this date.
On August 18, 1992, the prosecuting fiscal manifested before
the trial court that, per newspaper report, Toto Garcia had
been killed in Davao. Thus, on December 6, 1993, the trial
court convicted the Cuis, Obeso, Sarte, Basingan and Nacario.
Obeso and Sarte filed their Notice of Appeal on May 19, 1994.
The Cuis filed theirs on May 31, 1994.

Issue:
Whether or not the Cuis are liable as accessories?
Decision:
Art. 19 of the Revised Penal Code, as amended,
penalizes as accessories to the crime those who, subsequent
to its commission, take part therein by profiting themselves
or assisting the offenders to profit by the effects of the crime,
without having participated therein, either as principals or
accomplices. Conviction of an accused as an accessory
requires the following elements: (1) that he has knowledge of
the commission of the crime; and (2) that he took part in it
subsequent to its commission by any of the three modes
enumerated in Article 19 of the Revised Penal Code, as
amended. These twin elements are present in the case of the
Cuis, and indubitable proof thereof is extant in the records of
the case.
The Court held that the Cuis profited from the
kidnapping of Stephanie Lim and are liable as accessories.

People v. Verzola (G.R. No. L-35022)


Facts:
On September 28, 1969, Bernardo Molina was
clubbed to death by Ricardo Verzola in the presence of
appellant Josefina Molina inside Molina's house at Barrio
Lipcan, Bangued, Abra. The body of the victim was
subsequently carried by the two appellee to the ground and
left at the foot of the stairs. Appellant Verzola then went to
his house, changed his clothes and threw his bloodstained
sweater undershirt and underwear, including the piece of
wood be used in clubbing the deceased, inside their toilet.
Afterwards, he went to the municipal building and reported
to the police authorities that Bernardo had died in an
accident. The police authorities together with the Municipal
Health Officer, the Municipal Judge and a photographer went
to Lipcan to conduct the investigation. They found the body
of the deceased Bernardo Molina sprawled at the foot of the
bamboo ladder. Blood had oozed from the mouth, nose and
ears. There were bloodstains on the floor of the bedroom of
the house, on the mat, as well as on the beddings of the
deceased. The bloodstains led to the bamboo ladder where
some of the stains could be found on the steps of the ladder.
When questioned by the police, Josefina revealed that the
assailant of her husband was Ricardo Verzola.
Upon her request, she was brought to the Office of
the Chief of Police of Bangued, where at about 2:00 o'clock in
the morning of September 29, 1969 she gave a written
statement narrating the circumstances surrounding the
incident in question and pointing to appellant Verzola as the
assailant of her husband. In that extra-judicial statement, she
stated that immediately after 10:00 o'clock in the evening of

September 28. 1969, appellant Ricardo Verzola went to their


house in Barrio Lipcan, Bangued Abra entered the room
where she was sleeping with her husband, Bernardo Molina,
woke her up and had carnal knowledge of her; that when
Bernardo Molina woke up and attempted to rise from the
floor, that was the moment when Verzola clubbed Bernards,
hitting him on the head several times that afterwards, she
heard the sound of a body being dragged downstairs and the
voice of Verzola saying that he was leaving and warning her
not to say anything about the incident. She looked out of the
door and saw her husband already lying prostrate at the foot
of the stairs. This statement was sworn to by her before
Municipal Judge Francisco T. Valera.
On that same morning, appellant Verzola was picked
up by the police and brought to the municipal building, and
there he also executed a written statement admitting that he
clubbed the victim several times. Both appellants admit that
it was appellant Verzola who inflicted the fatal blows on the
victim.
The trial court convicted Verzola as principal and
Josefina Molina as an accessory to the crime of murder.
Issue:
Whether or not assisting the principal in bringing the
body of the deceased to the ground will make one an
accessory to the crime?
Decision:
An accessory does not participate in the criminal
design, nor cooperate in the commission of the felony, but
with knowledge of the commission of the crime, he
subsequently takes part in three (3) ways: (a) by profiting
from the effects of the crime; (b) by concealing the body,
effects or instruments of the crime in order to prevent its
discovery; and (c) by assisting in the escape or concealment
of the principal of the crime, provided he acts with abuse of
his public functions or the principal is guilty of treason,
parricide, murder, or an attempt to take the life of the Chief
Executive or is known to be habitually guilty of some other
crime.
Even if she assisted her co-appellant without duress,
simply assisting Verzola in bringing the body down the house
to the foot of the stairs and leaving said body for anyone to
see, cannot be classified as an attempt to conceal or destroy
the body of the crime. The concealing or destroying of the
body of the crime, the effects or instruments thereof, must
be done to prevent the discovery of the crime. In the case at
bar, the body was left at the foot of the stairs at a place
where it was easily visible to the public. Under such
circumstances there could not have been any attempt on the
part of Josefina to conceal or destroy the body of the crime.
Thus, Josefina Molina is acquitted.

Accessories Exempt from Criminal Liability


People v. Mariano (G.R. No. L-40527)
Facts:
That on or about and during the period from May 11
and June 8, 1971, in San Jose del Monte, Bulacan, the said
accused Hermogenes Mariano, being then appointed as
Liaison Officer by the then incumbent Municipal Mayor,
Constantino Nolasco, acting for and in behalf of the
municipality and authorized to receive and be receipted for
US excess property of USAID/NEC for the use and benefit of
said municipality, received from the said USAID/NEC the
following items with a total value of $717.50 or P4,797.35,
involving the duty of making delivery of said items to the said
Municipal Mayor, but the said accused Hermogenes Mariano
once in possession of the said items and far from complying
with his aforesaid obligation and in spite of repeated
demands, did then and there wilfully, unlawfully and
feloniously, with grave abuse of confidence and with deceit,
misappropriate, misapply and convert to his own personal use
and benefit the said items valued at $717.50 or P4,797.35,
belonging to the said USAID/NEC, to the damage and
prejudice of the said owner in the said sum of $717,50 or
P4,797.35. On February 19, 1975, Hermogenes Mariano thru
his counsel Filed a motion to quash the Information on the
following grounds:
1. That the court trying the cause has no jurisdiction
of the offense charged or of the person of the defendant;
2. That the criminal action or liability has been
extinguished;
3. That it contains averments which , if true, would
constitute a legal excuse or justification.
In his motion to quash, Mariano claimed that the
items which were the subject matter of the Information
against him were the same items for which Mayor
Constantino A. Nolasco was indicted before a Military
Commission under a charge of malversation of public
property, and for which Mayor Nolasco had been found guilty
and that inasmuch as the case against Mayor Nolasco had
already been decided by the Military Tribunal, the Court of
First Instance of Bulacan had lost jurisdiction over the case
against him.
On March 14, 1975 respondent Judge issued an
Order granting the motion to quash on the ground of lack of
jurisdiction reasoning as follows:

Considering that the Military Commission had


already taken cognizance of the malversation case against
Mayor Nolasco involving the same subject matter in its
concurrent jurisdiction with this Court, the case involving the
subject properties had already been heard and decided by a
competent tribunal, the Military Commission, and as such this
Court is without jurisdiction to pass upon anew the same
subject matter. (pp. 30-31, rollo, emphasis supplied)
Respondent Judge issued an order granting the motion to
quash on the ground of lack of jurisdiction but did not rule on
the other grounds invoked in the motion to quash.

action. In the case at bar, it is rightly contended by the


Solicitor General that at the time Criminal Case No. SM649 was filed with the Court of First Instance of Bulacan, that
was December 18, 1974, the law in force vesting jurisdiction
upon said court was the Judiciary Act of 1948, the particular
provision of which was not affected one way or the other by
any Presidential issuances under Martial Law. The Military
Commission is not vested with jurisdiction over the crime
of estafa.

Issue:
Whether or not Mariano can be held liable for

Arts. 21-24: Penalties in General


Retroactive Effect of Penal Laws

estafa?
Decision:
The Supreme Court ruled that Respondent court
gravely erred when it ruled that it lost jurisdiction over
the estafa case against respondent Mariano with the filing of
the malversation charge against Mayor Nolasco before the
Military Commission. Estafa and malversation are two
separate and distinct offenses and in the case now before the
SC the accused in one is different from the accused in the
other.
The conferment of jurisdiction upon courts or judicial
tribunals is derived exclusively from the constitution and
statutes of the forum. Thus, the question of jurisdiction of
respondent Court of First Instance over the case filed before it
is to be resolved on the basis of the law or statute providing
for or defining its jurisdiction. That, We find in the Judiciary
Act of 1948 where in its Section 44 (f) it is provided that
Courts of First Instance shall have original jurisdiction In all
criminal cases in which the penalty provided by law is
imprisonment for more than six months,or a fine of more than
two hundred pesos.The offense of estafa charged against
respondent Mariano is penalized with arresto mayor in its
maximum period to prision correccional in its minimum
period, or imprisonment from four (4) months and one (1)
day to two (2) years and four (4) months. By reason of the
penalty imposed which exceeds six (6) months imprisonment,
the offense alleged to have been committed by the accused,
now respondent, Mariano, falls under the original jurisdiction
of courts of first instance.
The above of course is not disputed by respondent
Judge; what he claims in his Order is that his court exercises
concurrent jurisdiction with the military commission and
because the latter tribunal was the first to take cognizance of
the subject matter, respondent court lost jurisdiction over
it .That statement of respondent court is incorrect. In People
vs. Fontanilla, this Court speaking through then Justice now
Chief Justice Fred Ruiz Castro, categorically reiterated the
settled rule that the jurisdiction of a court is determined by
the statute in force at the time of the commencement of the

People v. Evina (G.R. No. 124830-310)


Facts:
Gerardo Evina was found guilty by the Regional Trial
Court of Tacloban City (Branch 9) of two counts of simple
rape and sentenced to suffer the penalty of reclusion
perpetua for each count. The crime of rape was committed
on November 3, 1991 and on November 7, 1991, in the City
of Tacloban against Marites Cacharo while she was sleeping
in her bedroom.
Evina tied Maritess hands with a big
handkerchief and poked a knife at her. This special
aggravating circumstance of the use of a weapon and the
aggravating circumstance of dwelling were both proven
during the trial. However, these were not alleged in the
information.
Issue:
Whether or not the aggravating circumstances be
considered in fixing the penalty?
Decision:
The aggravating circumstances cannot be considered
in fixing the penalty because they were not alleged in the
information as mandated by Rule 110, Sections 8 and 9 of the
Revised Rules of Criminal Procedure. Although the crimes
charged were committed before the effectivity of the said
rule, nevertheless, the same should be applied retroactively
being favorable to the appellant. Although the aggravating
circumstances in question cannot be appreciated for the
purpose of fixing a heavier penalty in this case, they should,
however, be considered as bases for the award of exemplary
damages, conformably to current jurisprudence.
People v. Lazaro (G.R. No. 112090)
Facts:
Lazaro was charged, tried and convicted for two
separate crimes of illegal possession of firearms/ammunition

and homicide under Section 1 of P.D. No. 1866 which was the
governing law at the time the crime was committed in 1991.
The two separate cases, Criminal Case No. 91-3487 (for
homicide) and Criminal Case No. 91-3483 (for illegal
possession of firearm) were not tried jointly, although filed in
the same trial court. Republic Act No. 8294 has since
amended P.D. No. 1866 by reducing the penalties for simple
and aggravated forms of illegal possession and considering
the use of an unlicensed firearm simply as an aggravating
circumstance in murder or homicide. R.A. 8294 took effect
on July 6, 1997. The crime involved in the case at bench was
committed on May 5, 1991. In view of the amendments
introduced by Republic Act 8294 to Presidential Decree 1866,
separate prosecutions for homicide and illegal possession are
no longer in order. Instead, illegal possession of firearms is
merely to be taken as an aggravating circumstance in the
homicide case.
Issue:
Whether or not the accused can be rightfully
convicted of the crime of illegal possession of firearms
separately from the crime of homicide under RA 8294
(amending PD 1866).
Decision:
No. As a general rule, penal laws will generally have
prospective application except where the new law will be
advantageous to the accused. In this case R.A. 8294 will spare
accused-appellant Lazaro from a separate conviction for the
crime of illegal possession of firearm. Accordingly, said law
should be given retroactive application.
Accordingly, accused-appellant Lazaro should be
spared from a separate conviction for the crime of Illegal
Possession of Firearms, which is the subject of the present
review. Accused-appellant Lazaro was hereby acquitted of
the said crime and the case was dismissed.

Thereafter, Ballecer and Sta. Catalina proceeded to


Citytrust Bank to open a letter of credit. They were required
to post a marginal deposit amounting to P100, 000. The two
went to United Coconut Planters Bank to encash a check.
After the encashment, they went back to Citytrust but arrived
after banking hours. Sta. Catalina suggested that the money
be deposited in his account which Ballecer agreed.
Few days after, while preparing the supporting
documents for the letter of credit, Ballacer found that there
was an overpricing on the cost of the jute sacks. Realizing that
his business venture was losing proposition he asked Sta.
Catalina to return the P100, 000, however, the latter failed to
return the money despite repeated verbal and formal
demands made by the former.
Sta. Catalina as defense alleged that there was no
misappropriation of the money. He further claimed that the
said money was spent and used for the office expenses,
salaries and other expenses of the office which both of the
occupy.
The trial court convicted Sta. Catalina for the crime
of Estafa. Aggrieved, he appealed the decision of the trial
court before the Court of Appeals. However, the public
prosecutor filed a manifestation stating that Ballecer is no
longer interested in pursuing his complaint and the case
should be decided based on Ballecers Affidavit of Desistance.
The Court of Appeals rendered a Decision affirming the
judgment of conviction by the trial court. Hence, this instant
petition.
Issue:
Whether or not the Affidavit of Desistance executed
by Ballacer will justify the dismissal of the action?
Decision:

Pardon by Offended Party


Sta. Catalina v. People (G.R. No. 167805)
Facts:
Lorenzo Ballecer entered into a joint business
venture with Arnold Sta. Catalina involving importation of
Jute sacks from China. Petitioner told that he had a ready
buyer in the Philippines which was willing to buy the jute
sacks at P12.25 per piece. Convinced, Ballecer ordered one
container to Sta. Catalina.

The Supreme Court held that an Affidavit of


Desistance is not a ground for the dismissal of an action, once
the action has been instituted in court. In the case at bar,
Ballecer made the so-called pardon of Sta. Catalina after the
institution of the action, almost two years after the trial court
had rendered its decision.
The court attaches no persuasive value to a
desistance especially when executed as an afterthought. It
would be a dangerous rule to reject the testimony taken
before the court of justice simply because the witness who
had given it later on changed his mind for one reason or
another. Such a rule will make a solemn trial a mockery and
place the investigation at the mercy of unscrupulous
witnesses.
Balderama v. People (G.R. No. 147578-85)
Facts:

People v. Dimaano (G.R. No. 168168)


Rolando Balderama and Rolando Nagal are
employees of the Field Enforcement Division of LTO. On the
other hand, Juan Armamento is the operator of SJ Taxi. On
July 14, 1992, the team of Flying Squad flagged down one taxi
owned by Armamento. They impounded the taxi on the
ground that its meter was defective, however, upon
inspection and testing by the LTO the results showed that the
meter was functioning normally.
Feeling aggrieved, Armamento filed a complaint for
Bribery and violation of Anti-Graft and Corrupt Practices Act
before the Ombudsman. He alleged that prior to the
impounding of his taxi, the four LTO officers had been
collecting protection money from him in exchange of nonapprehension and non-impounding of his vehicles.
Eventually, the Office of the Ombudsman filed with
the Sandiganbayan nine (9) Information for violation of Direct
Bribery and Anti-Graft and Corrupt Practices Act. During the
pendency of the action, accused De Jesus died. The cases
against him were dismissed but the hearing proceeded
against Balderama, Nagal and Lubrica.
The Sandiganbayan rendered its Decision, convicting
Balderama, Nagal and Lubrica for the above violation. They
filed a motion for reconsideration but were denied by the
former. Hence, this instant petition.
Issue:
Whether or not Armamentos affidavit
recantation will result to the dismissal of the complaint?

Facts:
Maricar Dimaano is the daughter of the accused
Edgardo Dimaano. A complaint was filed by Maricar charging
Edgardo with two counts of Rape and one count of attempted
rape. Maricar alleged that she was only 10 years old when her
father Edgardo started sexually abusing her. It was only on
November of 1995 that she confided the sexual abuses to her
mother. The last sexual assault happened in the afternoon of
January 1, 1996.
Maricar and her mother went to Camp Crame upon
the advised of a relative. The Medico-Legal Officer at the PNP
Crime Laboratory examined the complainant and found her
to have suffered deep healed hymenal lacerations and was in
a non-virgin state.
On the other hand, the accused denied such
accusations. He contended that he could not have raped
Maricar because he was always in the office. He claimed that
it was impossible for him to rape his daughter because there
were other people in the house. He further argued that had
he raped Maricar, then she would have not accompanied him
to the Paranaque Police Station to apply for police clearance.
The trial court rendered its Decision, convicting
Edgardo of the crime of Rape. Aggrieved, he appealed his
case before the Court of Appeals but the latter affirmed the
decision of the trial court.

of

Decision:
The Supreme Court held that the complaint shall not
be dismissed. A recantation or an affidavit of desistance is
viewed with suspicion and reservation. The court looks with
disfavor upon retractions of testimonies previously given in
court. It is settled that an affidavit of desistance made by a
witness after conviction of the accused is not reliable, and
deserves only scant attention.
The rationale for the rule is obvious: affidavits of
retraction can easily be secured from witnesses, usually
through intimidation or for a monetary consideration. Only
when there exist special circumstances in the case which
when coupled with the retraction raise doubts as to the truth
of the testimony or statement given, can retractions be
considered and upheld.
In this case, there is indubitably nothing in the affidavit which
creates doubts on the guilt of the accused Balderama and
Nagal.

Issue:
Whether or not the voluntary and due execution of
the Affidavit of Desistance by Maricar is a ground for the
dismissal of the complaint against Edgardo?

Decision:
The Supreme Court held that by itself, an Affidavit of
Desistance is not a ground for the dismissal of an action, once
the action has been instituted in court. A private complainant
loses the right or absolute privilege to decide whether the
rape charge should proceed, because the case was already
filed and must therefore continue to be heard by the trial
court.
The court attaches no persuasive value to a
desistance, especially when executed afterthought. The
unreliable character of this document is shown by the fact
that it is quite incredible that a victim, after going through the
trouble of having the accused arrested by the police,
enduring the humiliation of a physical examination of her
private parts and recounting her anguish in detail, will
suddenly turn around and declare that she is no longer
interested in pursuing the case.

In the case at bar, Maricar repudiated the affidavit of


desistance in open court by stating that no lawyer assisted
her when she affixed her signature and had shown her
resolve to continue with the prosecution of the cases.
Arts. 25-45: Penalties
Reclusion Perpetua
People v. Novio (G.R. No. 139332)
Facts:
On September 23, 1994, 13-year-old Maricel B.
Talisay, together with her minor brothers Jun and Joey slept
side-by-side at their store. Their parents were caretakers of a
beach house and needed to sleep there at that time. At 3:00
in the morning, a ticklish sensation and stabs of pain in her
vagina awakened Maricel. When she woke up, she saw the
accused, Noli Novio, naked on top of her. Her duster was
rolled up to her neck and her panty has already been
removed by the accused. Noel Novio was able to penetrate
his penis inside Maricels Vagina.
Meanwhile, Maricels parents were awakened by
their neighbor and reported to them that a man was inside
their store. Nenita, Maricels mother immediately got hold of
her bolo and flashlight and proceeded to their store. Nenita
saw a mans sandals at the doorstep. Nenita knocked and
called out to Maricel and ordered her to open the door.
Despite repeated demands to open the door, Maricel was not
able to do so. Nenita was able to forcibly open the door and
beamed the flashlight to Maricel and saw Noel Novio on top
of her. The accused was holding the hands of Maricel with
his left hand and covered her mouth with his right hand.
Nenita mounted to hack Novio with her bolo but the accused
immediately took his jogging pants and ran away leaving his
shirt, wallet, underwear and sandals. Nenita immediately
reported the incident to the barangay and went to the police
for investigation. Maricel submitted herself to medical
examination right after the incident.
Noli Novio denied the allegations and argued that
Maricel and him are sweethearts. The trial court found the
accused guilty beyond reasonable doubt for the crime of rape
and sentenced him to 30 years of reclusion perpetua and to
indemnify the victim the sum of Fifty Thousand (P50,000.00)
pesos without subsidiary imprisonment in case of insolvency
and to pay the cost of these proceedings.
Issue:
Whether or not the trial court was correct in
imposing the proper penalty for the crime of rape?
Decision:
No, the Supreme Court held that the penalty
imposed by the trial court is void. Under Article 335 of the

Revised Penal Code, as amended by Republic Act 7659, the


prescribed penalty for simple rape is reclusion perpetua.
However, the trial court sentenced the appellant to thirty
years of reclusion perpetua. The penalty imposed by the trial
court is void. Although under Article 27 of the Revised Penal
Code as amended by Republic 7659, reclusion perpetua has a
range of twenty years and one day to forty years, by nature,
the penalty remains a single and indivisible penalty. It cannot
be divided into periods or equal portions. If the law
prescribes reclusion perpetua as a single and indivisible
penalty for a felony, the trial court is mandated to impose
said penalty, absent any privileged mitigating circumstances
conformably with Article 63 of the Revised Penal Code. The
trial court is not authorized to vary the penalty provided for
by law either in the character or the extent of punishment
inflicted.
There was no need for the trial court to specify the
duration of thirty years of reclusion perpetua whenever it is
imposed as a penalty in any proper case. The Court is not
impervious to Article 70 of the Revised Penal Code which
pertinently provides that, in applying the so-called three-fold
rule, i.e., that (w)hen the culprit has to serve two or more
penalties, . . . the maximum duration of the convicts
sentence shall not be more than three-fold the length of time
corresponding to the most severe of the penalties imposed
upon him the duration of perpetual penalties (penal
perpetua) shall be computed at thirty years. The imputation
of a thirty-year duration to reclusion perpetua in Article 70 is,
as this Court recently held, only to serve as the basis for
determining the convicts eligibility for pardon or for the
application of the three-fold rule in the service of multiple
penalties.

People v. Zacarias (G.R. No. 138990)


Facts:
Sergio Pelicano, Sr., on direct examination, testified
that on June 23, 1993 at about 12:30 in the morning while
waiting for his son, he heard a commotion outside his house.
When he looked outside, he saw Christopher Sacay, son of his
long-time friend, being chased by Sammy Zacarias, Rodel
Zacarias, Wally Ticalo and Rene Matugas. The boy ran
towards the Seventh Day Adventist Church.
Pelicano
followed the group and when he was only about 10 meters
away from the four men, he saw Rodel Zacarias hold the
victim while the rest took turns in stabbing and hacking the
boy. However, Ticalo claimed that on the day of the said
incident, he was working in the farm and had a drinking spree
with the owner of the farm until 10:00 in the evening. The
owner of the farm even contended that the town where his
farm was was far distant from the town where the stabbing
incident took place. The trial court sentenced Ticalo to serve
the penalty of reclusion perpetua for the death of Christopher
Sacay
Issue:

Whether or not the court a quo gravely erred in


finding Ticalo guilty of the crime charged?
Decision:
No, the Supreme Court is not convinced with the
contention of Ticalo, however, a word, in passing, about the
manner the trial court imposed the penalty. In the scales of
penalties under the Revised Penal Code, reclusion perpetua is
the penalty immediately higher than reclusion temporal
which has a duration of twelve years and one day to twenty
years. The minimum range of reclusion perpetua should then,
by necessary implication, start at 20 years and 1 day while the
maximum thereunder could be co-extensive with the rest of
the natural life of the offender. Article 70, however, provides
that the maximum period in regard to the service of sentence
shall not exceed 40 years. Reclusion perpetua remains to be
an indivisible penalty and, when it is the prescribed penalty,
should be imposed in its entirety, i.e., reclusion perpetua sans
a fixed period for its duration, regardless of any mitigating or
aggravating circumstance that may have attended the
commission of the crime. In prescribing the penalty of
reclusion perpetua, its duration in years, in fine, need not be
specified.
People v. Ramirez (G.R. No. 138261)
Facts:
On May 23, 1993, at 7:30 in the evening in Bgy. San
Jose, Ormoc City, Montano Banez, while strolling in the plaza,
saw the victim Jonathan Jojo Alkuino. Since Jojo was a
former resident of the barangay, Banez invited him to have a
drinking spree in the nearby store. The two sat side-by-side
and were exchanging stories when Pedro Ramirez suddenly
came up to them. Ramirez hit Jojo on the right side of his
body just below his ribs. Jojo was immediately brought to the
hospital and was still alive on arrival but died the next day
due to hypovolemic shock or massive blood loss. The trial
court found Ramirez guilty of murder and sentencing him to
"suffer imprisonment of forty (40) years reclusion perpetua.
Issue:
Whether or not the trial court was correct in
specifying the length of imprisonment in the penalty of
Reclusion Perpetua?
Decision:
No, the Supreme Court held that in sentencing
appellant "to suffer imprisonment of forty (40) years
reclusion perpetua." There was no justification or need for
the trial court to specify the length of imprisonment, because
reclusion perpetua is an indivisible penalty. The significance of
this fundamental principle was laid down by the Court in
People v. Diquit, "Since reclusion perpetua is an indivisible
penalty, it has no minimum, medium or maximum periods. It
is imposed in its entirety regardless of any mitigating or

aggravating circumstances that may have attended the


commission of the crime. (Art. 63, Revised Penal Code)
Reclusion perpetua is imprisonment for life but the person
sentenced to suffer it shall be pardoned after undergoing the
penalty for thirty (30) years, unless by reason of his conduct
or some other serious cause, he shall be considered by the
Chief Executive as unworthy of pardon (Art. 27, Revised Penal
Code)."
Arts. 46-77: Application of Penalties
Complex Crime
People v. Pineda (G.R. No. L-26222)
Facts:
On July 29, 1965, the occupants of the home of the
spouses Teofilo Mendoza and Valeriana Bontilao de Mendoza
in Pugaan City of Iligan, were asleep. It was then that guns
(rifle, caliber 22) and paliuntod (homemade gun) were fired in
rapid succession from outside the house. Teofilo Mendoza fell
dead. Thereafter, defendants below destroyed the door of
the house, entered therein, and let loose several shots killing
Neceforo Mendoza, all minor children of the couple and
wounding Valeriana Bontilao de Mendoza.
Two of the three defendants in the five criminal
cases: Tomas Narbasa and Tambak Alindo, moved for a
consolidation thereof "into one (1) criminal case." Their plea
is that "said cases arose out of the same incident and
motivated by one impulse."
Respondent Judge directed the City Fiscal to unify all
the five criminal cases, and to file one single information in
Case 1246. He also ordered that the other four cases, Nos.
1247, 1248, 1249 and 1250 "be dropped from the docket."
The City Fiscal moved for reconsideration upon the
ground that "more than one gun was used, more than one
shot was fired and more than one victim was killed."
Thereafter, the defense opposed.
On May 31, 1966, respondent Judge denied the
motion to reconsider. He took the position that the acts
complained of "stemmed out of a series of continuing acts on
the part of the accused, not by different and separate sets of
shots, moved by one impulse and should therefore be treated
as one crime though the series of shots killed more than one
victim;" and that only one information for multiple murder
should be filed, to obviate the necessity of trying five cases
instead of one."

Issue:
Whether or not there should be one information,
either for the complex crime of murder and frustrated
murder or for the complex crime of robbery with multiple

homicide and frustrated homicide or should the five


indictments remain as they are?
Decision:
Four separate crimes of murder and a frustrated
murder result from the firing of several shots at five victims.
The crimes are not complex. Five information should be filed.
There is a complex crime where one shot from a gun results
in the death of two or more persons, or where one stabbed
another and the weapon pierced the latters body and
wounded another, or where a person plants a bomb in an
airplane and the bomb explodes, with the result that a
number of persons are killed. When various victicms expire
from separate shots, such acts constitute separate and
distinct crimes.

People v. Sanidad (G.R. No. 146099)


Facts:
On 16 January 1999, Marlon Tugadi, Jun Quipay,
Raymund Fontanilla, Rolando Tugadi, Pepito Tugadi, Delfin
Tadeo, Ricardo Tadeo, Edwin Tumalip, Bobby Velasquez and
Dennis Balueg left Budac, Tagum, Abra, on board a passenger
jeepney driven by Delfin Tadeo to attend a barangay fiesta in
the town of Lagangilang, Abra. When they arrived they joined
the residents in a drinking spree that lasted the following
morning.
Accused-appellants Jimmel Sanidad, Ponce Manuel
alias Pambong and several other residents of Lagangilang
joined them in drinking. Marlon Tugadi and accused Jimmel
Sanidad were drinking buddies and members of the CAFGU
before then.
On 17 January 1999, Jimmel Sanidad and his
companions finished drinking and left. Shortly after, the
group of Marlon Tugadi also stopped drinking and headed
home for Budac, Tagum, Abra, boarding the same jeepney
driven by Delfin Tadeo.
As the jeepney moved closer, the accused in a classic
case of ambuscade suddenly and without warning unleashed
a volley of shots at the jeepney. Miraculously, almost all of its
passengers, with the exception of Rolando Tugadi, survived
the ambush and suffered only minor injuries.

Sanidad, Ponce Manuel alias Pambong, John Doe and Peter


Doe. The defense of the accused rested on bare denial and
alibi.
The trial court disregarded the defense interposed
by the accused and convicted them of the complex crime of
murder and multiple attempted murder, and sentenced them
to death.
Issue:
Whether or not accused-appellants are guilty of
complex crime of murder and multiple attempted murder
and imposing upon then the supreme penalty of death?
Decision:
The Supreme Court fully agreed with the lower court
that the instant case comes within the purview of Art. 48
of The Revised Penal Code which, speaking of complex
crimes, provides that when "a single act constitutes two or
more grave or less grave felonies, or when an offense is a
necessary means for committing the other, the penalty for
the most serious crime shall be imposed in its maximum
period." In a complex crime, although two or more crimes are
actually committed, they constitute only one crime in the
eyes of the law as well as in the conscience of the offender.
Although several independent acts were performed
by the accused in firing separate shots from their individual
firearms, it was not possible to determine who among them
actually killed victim Rolando Tugadi. Moreover, there is no
evidence that accused-appellants intended to fire at each and
every one of the victims separately and distinctly from each
other. On the contrary, the evidence clearly shows a single
criminal impulse to kill Marlon Tugadi's group as a whole.
Thus, one of accused-appellants exclaimed in frustration after
the ambush: "My gosh, we were not able to kill all of
them." Where a conspiracy animates several persons with a
single purpose, their individual acts done in pursuance of that
purpose are looked upon as a single act, the act of execution,
giving rise to a single complex offense.

Delito Continuado
Ramiscal v. Sandiganbayan (G.R. Nos. 169727-28)
Facts:

Apparently shaken and dazed by their terrifying


ordeal, the victims hid in a culvert on the side of the road and
did not come out until the police arrived at the scene.
Apparently shaken and dazed by their terrifying ordeal, the
victims hid on the side of the road and did not come out until
the police arrived at the scene.
An Information for murder with multiple attempted
murder and malicious mischief was filed against Jimmel

Pursuant to the recommendation of the Senate Blue


Ribbon Committee to prosecute and/or cause the
prosecution of Gen. Jose Ramiscal Jr. (Ret), past AFP-RSBS
President, who had signed the unregistered deeds of sale
covering the acquisition of certain parcels of land,
Ombudsman
Investigators
Ricardo
Sullano,
Rodil
Buenaventura and Anatolio Alejandrino of the Office of the
Deputy Ombudsman for the Military conducted a fact-finding

investigation. They executed a Joint Affidavit-Complaint,


stating that based on their findings, the following may be
charged with falsification of public documents and violation
of Section 3(e) and (g) of Republic Act (R.A.) No. 3019:
petitioner B/Gen. Jose Ramiscal, Jr., former AFP-RSBS
president; Atty. Meinrado Enrique Bello, Head of the AFPRSBS Legal Department in charge of Land Acquisition; Capt.
Perfecto Enrique Quilicot, AFP-RSBS Project Officer, Tanauan,
Batangas, Land Acquisition; and Notaries Public Alfredo
Nasser and Manuel Satuito.
The matter was further looked into by a panel of
Ombudsman Investigators, which issued on March 30, 2001 a
Joint Resolution finding probable cause to file the
corresponding Informations for 148 counts of violation of
Article 315, in relation to Article 171, paragraph 4 of the
Revised Penal Code, and Section 3 (e) of R.A. No. 3019 against
Meinrado Bello and Atty. Manuel Satuito. However, it was
likewise recommended that the complaint against petitioner
be dismissed, without prejudice to a thorough fact-finding
investigation on his liability.
After conducting clarificatory hearings, the
investigating panel issued a Memorandum, recommending to
the Ombudsman that petitioner be charged with 148 counts
of estafa through falsification of public documents, and one
count violation of Section 3(e) of R.A. No. 3019. The
Ombudsman approved the recommendation of the Panel of
Prosecutors. Petitioner and his co-accused filed their
respective Motions for Reconsideration of the investigating
panels June 15, 2004 Memorandum.The Sandiganbayan
denied the motion. It likewise denied the motion for the
consolidation of the cases, considering that the other cases
filed were pending in its other divisions.
Petitioner filed a motion for reconsideration of the
resolution which was denied again by the Sandiganbayan.
Motion to Quash was likewise denied.
Issue:
Whether or not only one information for estafa
should be filed for all these cases?
Held:
The petition has no merit. The Sandiganbayan, for its
part, sustained the contention of respondents and ruled that
the determination of (a) the charge/s and the person/s
against whom the charge is filed are addressed to the sound
discretion of the Prosecutors based on the facts before them;
and (b) the crimes committed by petitioner are separate, and
not a single crime consisting of series of acts arising from a
single criminal resolution.

Special Prosecutor objected thereto, insisting that there were


as many crimes committed by the accused as there were
sales contracts forged by them.
Indeed, the determination of what charges to file
and who are to be charged are matters addressed to the
discretion of the Ombudsman, including the matter of
whether the crime perpetrated by petitioner and his coaccused under the Informations pending in the Divisions of
the Sandiganbayan constitute delito continuado or classified
as concurso de delitos; or involve separate crimes under the
category of concurso real delito involve factual issues. Such
factual issues should be resolved after trial on the merits, and
not in this case. The Court is being tasked to determine
whether the several sales contracts executed by petitioner
and his co-accused were set afoot or triggered by a single
impulse and operated by an uninterrupted force however
long a time it may occupy, which, however, is a matter best
left to the determination of the trial court, in this case, the
Santiago v. Garchitorena (G.R. No. 109266)
Facts:
On May 1, 1991, petitioner was charged in Criminal
Case No. 16698 of the Sandiganbayan with violation of
Section 3(e) of R.A. No. 3019, as amended, otherwise known
as the Anti-Graft and Corrupt Practices Act, allegedly
committed by her favoring "unqualified" aliens with the
benefits of the Alien Legalization Program.
On May 24, 1991, petitioner filed with us a petition
for certiorari and prohibition, docketed as G.R. No. 9928999290 (Santiago v. Vasquez, 205 SCRA 162 [1992]), to enjoin
the Sandiganbayan from proceeding with Criminal Case No.
16698 on the ground that said case was intended solely to
harass her as she was then a presidential candidate. The
petition was dismissed on January 13, 1992.
On October 27, 1992, the Sandiganbayan (First
Division), of which Presiding Justice Garchitorena is a
member, set the criminal case for arraignment on November
13, 1992. The Sandiganbayan (First Division) denied the
motion to defer the arraignment. Petitioner filed a motion for
a bill of particulars. According to petitioner, unless she was
furnished with the names and identities of the aliens, she
could not properly plead and prepare for trial.
On March 14, 1993, the Sandiganbayan (First
Division) promulgated a resolution, admitting the 32
Amended Informations and ordering petitioner to post the
corresponding bail bonds. Hence, the filing of the instant
petition.
Issue:

When required to comment on the motion of


petitioner and his co- accused for a consolidation of the
charges filed against them before the Sandiganbayan, the

Whether or not the 32 Amended Informations may be


admitted?

guilty to the crime of theft. He is also a habitual delinquent,


this being his third conviction.

Held:
The petition is denied.
The Court find that, technically, there was only one
crime that was committed in petitioner's case, and hence,
there should only be one information to be file against
her.The 32 Amended Informations charge what is known as
delito continuado or "continued crime" and sometimes
referred to as "continuous crime."
The original information charged petitioner with
performing a single criminal act - that of her approving the
application for legalization of aliens not qualified under the
law to enjoy such privilege. The original information also
averred that the criminal act : (i) committed by petitioner was
in violation of a law - Executive Order No. 324 dated
April 13, 1988, (ii) caused an undue injury to one offended
party, the Government, and (iii) was done on a single day, i.e.,
on or about October 17, 1988. The 32 Amended Informations
reproduced verbatim the allegation of the original
information, except that instead of the word "aliens" in the
original information each amended information states the
name of the individual whose stay was legalized.
The 32 Amended Informations aver that the offenses
were committed on the same period of time, i.e., on or about
October 17, 1988. The strong probability even exists that the
approval of the application or the legalization of the stay of
the 32 aliens was done by a single stroke of the pen, as when
the approval was embodied in the same document. Likewise,
the public prosecutors manifested at the hearing the motion
for a bill of particulars that the Government suffered a single
harm or injury.

Issue:
Whether or not recidivism, as inherent in habitual
delinquency, should still be taken into consideration in fixing
the principal penalty?
Decision:
Yes, recidivism should still be taken into
consideration in fixing the principal penalty even though it is
inherent in habitual delinquency. The appellant in this case is
a habitual delinquent, this being his third conviction.
Recidivism, although inherent in habitual delinquency, should
still be considered in fixing the principal penalty. There is no
doubt that the purpose of the law in imposing additional
penalty on a habitual delinquent is to punish him more
severely. However, the result would be otherwise if, for
imposing the additional penalty, recidivism could not be
considered as an aggravating circumstance in fixing the
principal penalty. In the instant case, the mitigating
circumstance of voluntary plea of guilty is present. If the
aggravating circumstance of recidivism is not to be taken into
consideration for imposing the additional penalty for habitual
delinquency, the mitigating circumstance would require that
the penalty prescribed by law be imposed in it minimum
period. The imposition of the additional penalty would make
the penalty lighter, instead of more severe, contrary to the
purpose of the law.
People v. De Jesus (G.R. No. 45198)
Facts:

The Resolution dated March 3, 1993 in Criminal Case


No. 16698 of the Sandiganbayan (First Division) is affirmed
and its Resolution dated March 11, 1993 in Criminal Case No.
16698 is modified in the sense that the Office of the Special
Prosecutor of the Office of the Ombudsman is directed to
consolidate the 32 Amended Informations (Criminal Cases
Nos. 18371 to 18402) into one information charging only one
offense under the original case number, i.e., No. 16698.

Basilio de Jesus y Javier was convicted by the Court


of First Instance of Manila of the crime of theft of an umbrella
and a buri hat valued at P 2.65 committed, according to the
information, on April 28, 1936. He was therein sentenced to
one month and one day of arresto mayor with accessory
penalties, to indemnify Francis Liwanag the value of the
umbrella which was not recovered, and being a habitual
delinquent, the additional penalty of two years, four months
and one day of prision correccional.

Habitual Delinquency

The accused plead guilty of the crime imputed to


him in the information filed against him. It was alleged in the
information that he is a habitual delinquent, having been
convicted by final judgments of the crimes of theft and
qualified theft on January 4, 1933 and November 18, 1935
respectively, the date of his last release being January 10,
1936.

People v. Espina (G.R. No. 43556)


Facts:
The appellant was charged in the lower court with
the crime of theft of articles valued at P 585.15 and, having
pleaded guilty, was sentenced to six months and one day of
prision correccional and, being a habitual delinquent, to an
additional penalty of two years, four months and one day of
prision correccional. The appellant is a recidivist and plead

Due to his admission of guilt of the crime imputed to


him in the information, it is well settled in this jurisdiction
that when one pleads guilty of the crime imputed to him in
the information, it is understood that he admits all material

facts alleged therein, not excluding those alleging his former


convictions of other crimes.
Issue:
Whether or not the circumstance of recidivism can
be and must be twice taken into consideration, first as an
aggravating circumstance, and second as a qualifying
circumstance or one inherent in habitual delinquency?
Decision:
Yes, recidivism can be considered as an aggravating
circumstance in determining the principal penalty and as a
qualifying circumstance in habitual delinquency.
As to the principal penalty, there is the rule that in
cases in which the penalty prescribed by law contains three
periods, the courts must take into consideration, in the
application of said penalty, the aggravating or mitigating
circumstances established at the trial if they do not appear to
be compensated by other circumstance. It is reiterated in
People vs. Melendrez that the aggravating circumstance of
recidivism, even in cases of habitual delinquency, should be
taken into consideration in the application of the principal
penalty in the corresponding period.
The proposition that if recidivism is considered an
inherent or qualifying circumstance of habitual delinquency it
should not be taken into account in the imposition of the
principal penalty, seems to be untenable because it is based
upon the erroneous assumption that habitual delinquency is
a crime. It is simply a fact or circumstance which, if present in
a given case, gives rise to the imposition of the additional
penalties prescribed therein.
As to the additional penalty, if we must rely upon
the spirit and letter of the law, we would say that the purpose
of the latter in establishing it was to prevent those for the
second time or more commit the crimes from relapsing
thereafter at least during the period fixed thereby. The lower
court correctly ruled in imposing the additional penalty.

Arts. 89-93: Total Extinction of Criminal Liability


Death of the Accused
De Guzman v. People (G.R. No. 154579)
Facts:
On February 8, 1995, in the City of Makati, petitioner
De Guzman, stole several pieces of jewelry valued
at P4,600,000.00 belonging to one Jasmine Gongora. The trial
court rendered its decision finding de Guzman guilty beyond
reasonable doubt and imposed a penalty of imprisonment, as
well as the penalties accessory thereto. The Court further
finds the accused De Guzman civilly liable and orders her to
pay the private offended party. On appeal, the CA affirmed
the conviction but reduced the award of damages. During the

appeal in the Supreme Court, on January 30, 2003, counsel


for the petitioner filed a Manifestation informing the Court
that the petitioner passed away on January 13, 2003. The
death of the petitioner resulted from a vehicular accident, as
indicated in the Certificate of Death attached thereto.
Issue:
Whether or not the criminal and civil liability of the
petitioner is extinguished by reason of her death?
Decision:
Upon death of the accused pending appeal of his
conviction, the criminal action is extinguished inasmuch as
there is no longer a defendant to stand as the accused; the
civil action instituted therein for recovery of civil liability ex
delicto is ipso facto extinguished, grounded as it is on the
criminal. Although both the trial and the appellate courts
found petitioner guilty beyond reasonable doubt, she had the
right to appeal her case to this Court of last resort and
challenge the findings of the two courts below. The judgment
of conviction was pending review until her untimely demise.
It has, therefore, not yet attained finality. Thus, pursuant to
Article 89 of the Revised Penal Code, it is incumbent upon the
Court to dismiss the instant petition for review. The Court is
dismissing the case because there is no longer a need to
continue with the review of the appeal. The lower courts
decision has thus become ineffectual.
People v. Bayotas (G.R. No. 102007)
Facts:
In Criminal Case No. C-3217 filed before Branch 16,
RTC Roxas City, Rogelio Bayotas y Cordova was charged with
Rape and eventually convicted thereof on June 19, 1991.
Pending appeal of his conviction, Bayotas died on
February 4, 1992 at the National Bilibid Hospital due to cardio
respiratory arrest secondary to hepatic encephalopathy
secondary to hipato carcinoma gastric malingering.
Consequently, the Supreme Court in its Resolution of
May 20, 1992 dismissed the criminal aspect of the appeal.
However, it required the Solicitor General to file its comment
with regard to Bayotas' civil liability arising from his
commission of the offense charged.
In his comment, the Solicitor General expressed his
view that the death of accused-appellant did not extinguish
his civil liability as a result of his commission of the offense
charged.
Issue:Whether or not the death of the accused pending
appeal of his conviction extinguish his civil liability?
Decision:

Death of the accused pending appeal of his conviction


extinguishes his criminal liability as well as the civil liability
based solely thereon. As opined by Justice Regalado, in this
regard, "the death of the accused prior to final judgment
terminates his criminal liability and only the civil
liability directly arising from and based solely on the offense
committed, i.e., civil liability ex delicto in senso strictiore."
Corollarily, the claim for civil liability survives
notwithstanding the death of accused, if the same may also
be predicated on a source of obligation other than
delict. Article 1157 of the Civil Code enumerates these other
sources of obligation from which the civil liability may arise as
a result of the same act or omission:
a) Law
b) Contracts
c) Quasi-contracts
d) . . .
e) Quasi-delicts
Where the civil liability survives, as explained in Number
2 above, an action for recovery therefor may be pursued but
only by way of filing a separate civil action and subject to
Section 1, Rule 111 of the 1985 Rules on Criminal Procedure
as amended. This separate civil action may be enforced either
against the executor/administrator or the estate of the
accused, depending on the source of obligation upon which
the same is based as explained above.
Finally, the private offended party need not fear a
forfeiture of his right to file this separate civil action by
prescription, in cases where during the prosecution of the
criminal action and prior to its extinction, the privateoffended party instituted together therewith the civil action.
In such case, the statute of limitations on the civil liability is
deemed interrupted during the pendency of the criminal
case, conformably with provisions of Article 1155 of the Civil
Code, that should thereby avoid any apprehension on a
possible privation of right by prescription.
Applying this set of rules to the case at bench, we
hold that the death of appellant Bayotas extinguished his
criminal liability and the civil liability based solely on the act
complained of, i.e., rape. Consequently, the appeal is hereby
dismissed without qualification.

People v. Abungan (G.R. No. 136843)


Facts:
On August 4, 1992, at Capulaan, Villasis, Pangasinan,
the accused conspiring, confederating and mutually helping
one another, armed with long firearms, attack, assault and
shoot Camilo Dirilo, [Sr.] y Pajarito, inflicting upon him
wounds on the different parts of his body which directly
caused his death.

An Information, dated March 9, 1993, was filed


charging appellant Pedro Abungan, together with Randy
Pascua and Ernesto Ragonton Jr. (both at large), with murder.
The trial court rendered a decision finding Abungan
guilty beyond reasonable doubt of the crime of murder.
During appeal, in a letter dated August 7,
2000, however, Joselito A. Fajardo, assistant director of the
Bureau of Corrections, informed the Court that Appellant
Abungan had died on July 19, 2000 at the NBP Hospital.
Issue:
Whether or not the criminal and civil liability of the
appellant is extinguished by reason of her death?
Decision:
In the present case, it is clear that, following the case
of People vs. Bayotas, the death of appellant extinguished his
criminal liability. Moreover, because he died during the
pendency of the appeal and before the finality of the
judgment against him, his civil liability arising from the crime
or delict (civil liability ex delicto) was also extinguished. It
must be added, though, that his civil liability may be based on
sources of obligation other than delict. For this reason, the
victims may file a separate civil action against his estate, as
may be warranted by law and procedural rules.
Brian Bonifacio Dela Cruz
2007-0388
Prescription of Offenses
Panaguiton v. DOJ (G.R. No. 167571)
Facts:
Cawili and his business associate Tongson borrowed
from Panaguiton (petitioner) sums amounting to 1,979,459.
They issued checks signed by both of them to Panaguiton but
these were dishonored upon presentation. Panaguiton made
demands to pay but to no avail. He formally filed a complaint
on August 24, 1995 for violating BP 22 before the City
Prosecutors Office.
Tongson moved to drop his name from the case as his
signatures were allegedly falsified. Case against him was
dismissed but afterwards upon finding that Tongson might
have indeed signed the checks, the chief state prosecutor
directed the city prosecutor to conduct a reinvestigation.
Tongson moved for reconsideration but denied.
In 1999 assistant prosecutor dismissed the complaint
for the action has prescribed pursuant to Act 3326, which
provides for the prescriptive periods of statutes without their
own (4 years for BP22). She claims that the filing of the
complaint on August 24, 1995 did not interrupt the running of

the period as the law refers to judicial and not administrative


proceedings.
Issue:Whether or not the filing of the complaint in the
prosecutors office tolled the prescriptive period?
Decision:
Yes. Filing of the complaint in the prosecutors office
tolls the prescriptive period for violations of BP22. When Act
3326 was passed into law, preliminary investigation of cases
was done by the justices of peace, and not by agents of the
executive department (i.e. prosecutors). Thus, the prevailing
rule at that time is that prescription is tolled once filed with
the justice of peace (a judicial process). However, since then,
the conduction of a preliminary investigation was moved to
the function of the executive department.
Today, the term proceedings must be understood to
mean either executive or judicial proceedings. With this
interpretation, any type of investigation may ultimately lead
to sufficiently toll prescription.
To rule otherwise would deprive the injured party
the right to obtain vindication on account of delays not under
his control. As seen in this case, various conflicting opinions
of the DOJ delayed his cause. Aggrieved parties who do not
sleep on their right should not be allowed to suffer simply
because of circumstances beyond their control.

Recebido v. People (346 SCRA 881)


Facts:
Sometime in April 1985, Caridad Dorol mortgaged her
property--an agricultural land located in Bacon, Sorsogon to
her cousin Recebido. Dorol and Recebido did not execute any
mortgage document, but instead, the former gave to the
latter a copy of the Deed of Sale dated June 16, 1973 which
was done by Juan Dorol (father of Caridad).
On September 9, 1990, Caridad Dorol went to the house
of Recebido to redeem such property, wherein Recebido
refused to allow claiming that Dorol has already sold to him
the land on 1979. Dorol, on the other hand, insisted that the
transaction between them was not a sale, but a mere
mortgage.
Caridad Dorol, then, went to the Office of the Assessor in
Sorsogon and verified the existence of a file Deed of Sale
dated August 13, 1979 in which she knew that the property
was already registered in Recebidos name. A comparison of
the specimen signatures of Caridads other documents and
that in the questioned Deed of Sale was done, and NBI
Document Examiner Antonio Magbojas found out that in the
latters signature was falsified.

The Office of the Provincial Prosecutor of Sorsogon filed


the information indicting Recebido for Falsification of Public
Document with the Regional Trial Court of Sorsogon.
The trial court rendered the decision convicting the
petitioner of the crime and sentenced to an indeterminate
penalty of one (1) year to three (3) years and six (6) months
of prision correccional as maximum and to pay a fine of
Three Thousand (P3,000.00) Pesos, with subsidiary
imprisonment.
The defense of prescription was raised only during the
motion for reconsideration of the Court of Appeals.

Issue: Whether or not the crime charged had already


prescribed at the time the information was filed?
Decision:
No. Prescription, although not invoked in the trial,
may, as in this case, be invoked on appeal. Hence, the failure to
raise this defense in the motion to quash the information does
not give rise to the waiver of the petitioner-accused to raise
the same anytime thereafter including during appeal.
Nonetheless, we hold that the crime charged has not
prescribed. The petitioner is correct in stating that whether or
not the offense charged has already prescribed when the
information was filed would depend on the penalty imposable
therefore, which in this case is prision correccional in its
medium and maximum periods and a fine of not more than
5,000.00 pesos.
Under the Revised Penal Code, said penalty is a
correctional penalty in the same way that the fine imposed is
categorized as correctional. Both the penalty and fine being
correctional, the offense shall prescribe in ten years. The issue
that the petitioner has missed, however, is the reckoning point
of the prescriptive period. The petitioner is of the impression
that the ten-year prescriptive period necessarily started at the
time the crime was committed. This is inaccurate. Under
Article 91 of the Revised Penal Code, the period of prescription
shall commence to run from the day on which the crime is
discovered by the offended party, the authorities, or their
agents.

Amnesty
People v. Patriarcha (G.R. No. 135457)
Facts:
On August 16, 1990, an Information for murder was
filed against Jose Patriarca, Jr., alias "Ka Django," "Carlos
Narra", "Ka Jessie," et al., for killing Alfredo Arevalo.

Accused-appellant Jose Patriarca, Jr. was also


charged with Murder for the killing of one Rudy de Borja and
a certain Elmer Cadag under Informations docketed as
Criminal Cases Nos. 2665 and 2672, respectively.
On January 20, 1998, the lower court rendered its
decision convicting the herein accused-appellant.
Thus, Accused-Appellant filed his appeal. However,
while his appeal was pending, he applied for amnesty under
Proclamation No. 724 amending Proclamation No. 347, dated
March 25, 1994, entitled "Granting Amnesty to Rebels,
Insurgents, and All Other Persons Who Have or May Have
Committed Crimes Against Public Order, Other Crimes
Committed in Furtherance of Political Ends, and Violations of
the Article of War, and Creating a National Amnesty
Commission." His application was favorably granted by the
National Amnesty Board.
After a careful verification and evaluation on the
claims of the applicant, the Local Amnesty Board concluded
that his activities were done in the pursuit of his political
beliefs. It, thus, recommended on 20 May 1998 the grant of
his application for amnesty.
The Commission, in its deliberation on the
application on 22 October 1999, resolved to approve the
recommendation of the Local Amnesty Board.
The Office of the Solicitor General, in its letter dated
June 23, 2000 to the National Amnesty Commission,
requested information as to whether or not a motion for
reconsideration was filed by any party, and the action, if
there was any, taken by the NAC.
In his reply dated June 28, 2000, NAC Chairman
Tadiar wrote, among other things, that there has been no
motion for reconsideration filed by any party.
Accused-appellant
Jose
N.
Patriarca, Jr. was granted amnesty under
Proclamation No. 724 on May 17, 1996.
Issue: Whether or not the grant of amnesty
in favor of Jose Patriarca, Jr. - while the
various criminal cases filed against him
were pending
- shall
completely
extinguished his criminal liability?

released by amnesty stands before the law precisely as


though he had committed no offense.
Paragraph 3 of Article 89 of the Revised Penal Code
provides that criminal liability is totally extinguished by
amnesty, which completely extinguishes the penalty and all
its effects.
In the case of People vs. Casido, the difference
between pardon and amnesty is given:
"Pardon is granted by the Chief Executive and as
such it is a private act which must be pleaded and proved by
the person pardoned, because the courts take no notice
thereof; while amnesty by Proclamation of the Chief
Executive with the concurrence of Congress, is a public act of
which the courts should take judicial notice. Pardon is
granted to one after conviction; while amnesty is granted to
classes of persons or communities who may be guilty of
political offenses, generally before or after the institution of
the criminal prosecution and sometimes after conviction.
Pardon looks forward and relieves the offender from the
consequences of an offense of which he has been convicted,
that is, it abolishes or forgives the punishment, and for that
reason it does 'not work the restoration of the rights to hold
public office, or the right of suffrage, unless such rights be
expressly restored by the terms of the pardon,' and it 'in no
case exempts the culprit from the payment of the civil
indemnity imposed upon him by the sentence' (Article 36,
Revised Penal Code). While amnesty looks backward and
abolishes and puts into oblivion the offense itself, it so
overlooks and obliterates the offense with which he is
charged that the person released by amnesty stands before
the law precisely as though he had committed no offense."
This Court takes judicial notice of the grant of
amnesty upon accused-appellant Jose N. Patriarca, Jr. Once
granted, it is binding and effective. It serves to put an end to
the appeal.

Arts. 100-103: Civil Liability


Subsidiary Civil Liability of Other Persons
Nueva Espana v. People (460 SCRA 547)
Facts:

Decision:
Amnesty commonly denotes a general pardon to
rebels for their treason or other high political offenses, or the
forgiveness which one sovereign grant to the subjects of
another, who have offended, by some breach, the law of
nations. Amnesty looks backward, and abolishes and puts
into oblivion, the offense itself; it so overlooks and obliterates
the offense with which he is charged, that the person

Petitioner Nueva Espana was found guilty of reckless


imprudence resulting into double homicide when the
passenger bus he was driving rammed into a Honda
motorcycle driven by Reynard So with Nilo Castro as
passenger resulting into the death of both. An aggravating
circumstance was also imposed as Espana also left the scene
of the crime without lending assistance to the victims.

During trial, the father of So and the mother of Castro


were both called on to testify as to the earning capacity of
the two. Sos father claimed that his son was earning P80,000
a month while Castros mother said that his son was bringing
in P8,000 a month. Sos father additionally testified that the
funeral expenses incurred by them was P87,000 while
Castros mom stated that they spent P30,000 for the funeral.

for the liability of the petitioner as its driver. Thus, this


petition for review with the SC.

As a result the trial court, besides imprisonment,


awarded the following amounts to the heirs of the victims:
TO THE HEIRS OF THE VICTIM REYNARD SO
1)
P2,997,000.00 indemnity for loss of earning
capacity of victim
2)
14,200.00 for expenses of the wake
3)
20,000.00 for funeral parlor
4)
12,000.00 for the tomb
5)
53,000.00 for cost of burial site
6)
30,000.00 for attorneys fees
7)
200,000.00 for moral damages
8)
100,000.00 for exemplary damages
P3,429,200.00 TOTAL AMOUNT
TO THE HEIRS OF VICTIM NILO CASTRO
1)
P1,728,000.00 indemnity for loss of earning
capacity
2)
20,000.00 for funeral expenses
3)
200,000.00 for moral damages
4)
50,000.00 for exemplary damages
P1,998,000.00 TOTAL AMOUNT
The court based the amount of loss of earning capacity
based on the formula used by the Supreme Court as
illustrated:
As to the civil liability, particularly the indemnity for
the loss of the earning capacity of the victims, the formula
last enunciated by the Supreme Court is:
Net earning capacity (x) = life expectancy x grossliving
expenses annual (50% of
gross annual
income)
Thusly, since the victim Reynard So was earning
P80,000 a month at the time of his death when he was thirty
(30) years old, his lost earning capacity should be computed
as follows:
x = 2 (80 30) x [P960,000.00 P480,000.00)
3
x = 33.4 x P480,000.00
x=
x P16,032,000.00
With respect to the victim Nilo Castro, he was
earning P8,000.00 a month when he died at the age of
twenty-six (26). His lost earnings were:
x = 2 (80 26) [P96,000.00 P48,000.00]
3
x = 36 x P48,000.00
x = P1,728,000.00
As a result, petitioner appeals to the CA but the
appellate court affirmed the decision of the trial court
regarding the damages, Consequently, the CA declared that
Vallacar Transit Inc., should not yet be held subsidiary liable

Decision:

Issue:
Was the award of damages amounting to P8 million
proper?

The SC modifies the award of damages mostly to the


fact that loss of earning capacity should be properly adduced
and supported by competent evidence to prove the same.
This rule also applies to the funeral and burial expenses. In
the case at bar, the lower courts based their award for
damages solely on the testimony of SOs father and Castros
mother, even though both of them never substantiated the
amounts claimed with receipts, papers and other evidence.
And so the award is modified as follows:
To summarize, the heirs of the deceased Reynard So are
entitled to the following:
P 50,000 civil indemnity ex delicto
73,000 actual damages
25,000 temperate damages
50,000 moral damages
25,000 exemplary damages
30,000 attorneys fees
P 253,000 TOTAL
The heirs of Nilo Castro are also entitled to the following:
P 50,000 civil indemnity ex delicto
50,000 temperate damages
50,000 moral damages
25,000 exemplary damages
30,000 attorneys fees
P 205,000 TOTAL
The SC meanwhile adopts the pronouncement of the
Court of Appeals regarding the subsidiary liability of
petitioners employer, Vallacar Transit Inc., under Article 103
of the Revised Penal Code. An employer may be subsidiarily
liable for the employees civil liability in the criminal action if
it can be shown that: (1) the employer is engaged in any kind
of industry; (2) the employee committed the offense in the
discharge of his duties and (3) the accused is insolvent.
However, subject to prevailing jurisprudence, the subsidiary
liability may be enforced only upon a motion for subsidiary
writ of execution against Vallacar Transit, Inc. and upon proof
that petitioner is insolvent.
Pangonorom v. People (455 SCRA 211)
Facts:
Pangonorom was the driver of a passenger bus
owned and operated by MMTC which collided with a Gemini
Isuzu car driven by Carlos Berba, resulting into the damage of
the car and physical injuries obtained by Berba.

Pangonorom was found guilty of reckless


imprudence resulting into damage to property and physical
injuries.

Quinto v. Andres (453 SCRA 511)

The trial court awarded damages amounting to


P42,000 but was silent as to the subsidiary liability of MMTC.
Consequently, the CA affirmed the decision of the trial court
and also found MMTC subsidiary liable for the amount
notwithstanding the fact that the judgment of the trial court
was silent as to said matter.

Petitioner Quinto is the mother of an 11-year old


boy named Wilson who died while going inside a drainage
with the respondents Andres and Pacheco, who were also of
the same age.

Issue:
Did the CA err in not holding MMTC not subsidiary
liable despite the fact that the RTC did not mention anything
to that effect?
Decision:
The SC ruled that even when the dispositive portion
of an RTC decision does not expressly pronounce subsidiary
liability of the employer, they are deemed written into the
judgment whenever applicable.
But, he subsidiary liability of the employer arises
only after conviction of the employee in the criminal action.
In the present case, there exists an employer-employee
relationship between petitioners, the MMTC is engaged in
the transportation industry, and Olimpio has been adjudged
guilty of a wrongful act and found to have committed the
offense in the discharge of his duties. However, there is no
proof here of Olimpios insolvency. The judgment of
conviction against Olimpio has not attained finality. This
being so, no writ of execution can issue against him to satisfy
his civil liability. Only after proof of the accused-employees
insolvency may the subsidiary liability of his employer be
enforced.
In short, there is as yet no occasion to speak of
enforcing the employers subsidiary civil liability unless it
appears that the accused-employees primary liability cannot
in the first instance be satisfied because of insolvency. This
fact cannot be known until sometime after the verdict of
conviction shall have become final. And even if it appears
prima facie that execution against the employee cannot be
satisfied, execution against the employer will not issue as a
matter of course. The procedure for the enforcement of a
judgment will have to be followed. Once the judgment of
conviction against Olimpio becomes final and executory, and
after the writ of execution issued against him is returned
unsatisfied because of his insolvency, only then can a
subsidiary writ of execution be issued against the MMTC after
a hearing set for that precise purpose. It is still too early to
hold the MMTC subsidiarily liable with its accused-employee
considering that there is no proof yet of Olimpios insolvency.

Facts:

What was clear according to a witness who was a


friend of the victim was that the three of them (Wilson and
the respondents) went inside the drainage filled with water.
First to emerge was Pacheco who immediately went home,
and then next to come out was Andres who was already
carrying the dead body of Wilson.
After being charged with homicide, the trial court
the respondents not guilty and also found the same not civilly
liable because of the absence of preponderance of evidence
to prove liability. QUinto appealed the civil aspect of the
decision which the CA affirmed.
Issue:
Petitioner comes to the Court and raises the
following issues: 1) Does extinction of criminal liability carry
with it extinction of the civil liability; and
2) was the
prosecution able to establish preponderance of evidence.
Decision:
The civil action based on delict is not extinguished
unless the court itself finds that civil liability did not arise. In
the case at bar, the trial court was very clear that the
prosecution was not able to establish a preponderance of
evidence to find the respondents liable.
As to whether preponderance of evidence should
have been considered, the trial court and the CA was correct
in their findings. Preponderance of evidence should not be
based on the fact that the evidence of the defense is weaker.
The evidence presented must be strong enough
to
SUFFICIENTLY SUSTAIN THE CAUSE OF ACTION.
In the case at bar, the prosecution single prosecution
witness testified that the hematomas on the alleged victim
may have been caused by either hitting with a blunt object or
slipping and falling on the hard pavement.
Even the friend of the deceased testified that the
drainage was so dark and this was the reason that he did not
come with the other boys inside.
And so, the possibility of slippage by Wilson was very
much a possibility.

The multiple prison terms are distinct from each


other, and if none of the terms exceeds the limit set out in
the P.D. No. 603, then he is entitled to probation, unless he is
otherwise specifically disqualified.

Probation Law (P.D. No. 968)


Francisco v. CA (G.R. No. 108747)
Facts:
Petitioner, as President and General Manager of the
company, humiliated his employees and blurted out
invectives against the latter. He was charged with multip[le
grave oral defamation by 5 of his employees who were
allegedly the recipient of the said invectives.
He was found guilty of oral defamation in 4 out of 5
cases filed against him. Petitioner elevated the judgment
from the MeTC to the RTC; however, the latter affirmed his
conviction, with modification, accrediting to him the
mitigating circumstance of passion or obfuscation. His appeal
to the CA was to no avail also.
Issue: Whether or not petitioner is still qualified to avail of
probation?
Decision: NO
Probation is a mere privilege, not a right. Its benefits
cannot extend to those not expressly included. Probation is
not a right of an accused, but rather an act of grace and
clemency or immunity conferred by the state which may be
granted by the court to a seemingly deserving defendant who
thereby escapes the extreme rigors of the penalty imposed
by law for the offense of which he stands convicted.
The Probation Law should not therefore be
permitted to divest the state or its government of any of the
latter's prerogatives, rights or remedies, unless the intention
of the legislature to this end is clearly expressed, and no
person should benefit from the terms of the law who is not
clearly within them.
That an appeal should not bar the accused from
applying for probation if the appeal is taken solely to reduce
the penalty is simply contrary to the clear and express
mandate of Sec, 4 of P.D. No. 603, which states that no
application for probation shall be entertained or granted if
the defendant has perfected the appeal from the judgment of
conviction.
The penalties imposed by the MeTC were already
probationable. Hence, there was no need to appeal if only to
reduce the penalties to within the probationable period.

Fixing the cut-off point at a maximum term of six (6)


years imprisonment for probation is based on the assumption
that those sentenced to higher penalties pose too great a risk
to society, not just because of their demonstrated capability
for serious wrong doing but because of the gravity and
serious consequences of the offense they might further
commit.
Considering that the multiple prison terms should
not be summed up but taken separately as the totality of all
the penalties is not the test, petitioner should have
immediately filed an application for probation as he was
already qualified after being convicted by the MeTC, if indeed
thereafter he felt humbled, was ready to unconditionally
accept the verdict of the court and admit his liability.
Consequently, in appealing the Decision of the MeTC to the
RTC, petitioner lost his right to probation. For, plainly, the law
considers appeal and probation mutually exclusive remedies.
Anti-Fencing Law (P.D. No. 1612)
Francisco v. People (434 SCRA 122)
Facts:
Pacita Linghon was the helper of Jovita Rodriguez.
Pacita, through her brother Macarion, sold to petitioner
Ernesto Linghon several pieces of jewelry stolen from
Rodriguez. The Regional Trial Court of Malolos, Bulacan,
Branch 22, found petitioner Ernesto Francisco guilty of
violating Presidential Decree No. 1612, otherwise known as
the Anti-Fencing Law, sentencing him to suffer the penalty of
ten (10) years and one (1) day of prision mayor maximum, as
minimum, to twenty (20) years of reclusion temporal
maximum, as maximum, with the accessory penalties
corresponding to the latter, and to pay the corresponding
value of the subject pieces of jewelry. The petitioner asserts
that the prosecution failed to prove his guilt for the crime
charged beyond reasonable doubt. He avers that the
prosecution failed to prove that Pacita stole the jewelry
subject of the charge, and that Macario sold the said pieces
of jewelry to him.
Issue: Whether the Court of Appeals erred in sustaining the
trial courts decision finding petitioner guilty beyond
reasonable doubt of violation of the (sic) Presidential Decree
No. 1612, otherwise known as the Anti-Fencing Law?
Decision:
The essential elements of the crime of fencing are as
follows: (1) a crime of robbery or theft has been committed;
(2) the accused, who is not a principal or accomplice in the
commission of the crime of robbery or theft, buys, receives,

possesses, keeps, acquires, conceals, sells or disposes, or


buys and sells, or in any manner deals in any article, item,
object or anything of value, which has been derived from the
proceeds of the crime of robbery or theft; (3) the accused
knew or should have shown that the said article, item, object
or anything of value has been derived from the proceeds of
the crime of robbery or theft; and, (4) there is, on the part of
the accused, intent to gain for himself or for another. Macario
Linghon testified that he sold the jewelry to petitioner.
Although the well-entrenched rule is that the testimony of a
single witness is sufficient on which to anchor a judgment of
conviction, it is required that such testimony must be credible
and reliable. In this case, we find the testimony of Macario to
be dubious; hence, barren of probative weight. The Court
further held It bears stressing that, in the absence of direct
evidence that the accused had knowledge that the jewelry
was stolen, the prosecution is burdened to prove facts and
circumstances from which it can be concluded that the
accused should have known that the property sold to him
were stolen. This requirement serves two basic purposes: (a)
to prove one of the elements of the crime of fencing; and, (b)
to enable the trial court to determine the imposable penalty
for the crime, since the penalty depends on the value of the
property.

Tan v. People (313 SCRA 220)


Facts:
Rosita Lim is the proprietor of Bueno Metal
Industries; upon inventory, she found that several pieces of
equiptment were missing. Manuelito Mendez was a former
employee of Lim, who left her employment before Lim found
out that her goods were missing. Subsequently, Manuelito
Mendez was arrested in the Visayas and he admitted that he
and his companion Gaudencio Dayop stole from the
complainants warehouse some boat spare parts such as
bronze and stainless propellers and brass screws. Manuelito
Mendez asked for complainants forgiveness. He pointed to
petitioner Ramon C. Tan as the one who bought the stolen
items and who paid the amount of P13,000.00, in cash to
Mendez and Dayop, and they split the amount with one
another. Tan was found guilty beyond reasonable doubt of
violating the Anti-Fencing Law of 1979, otherwise known as
Presidential Decree No. 1612, and sentences him to suffer the
penalty of imprisonment of SIX (6) YEARS and ONE (1) DAY to
TEN (10) YEARS of prision mayor and to indemnify Rosita Lim
the value of the stolen merchandise purchased by him in the
sum of P18,000.00.
Issue:
Whether or not the prosecution has successfully
established the elements of fencing as against petitioner?
Decision:

Fencing, as defined in Section 2 of P.D. No. 1612 is


the act of any person who, with intent to gain for himself or
for another, shall buy, receive, possess, keep, acquire,
conceal, sell or dispose of, or shall buy and sell, or in any
manner deal in any article, item, object or anything of value
which he knows, or should be known to him, to have been
derived from the proceeds of the crime of robbery or theft.
The essential elements of the crime of fencing are as follows:
(1) a crime of robbery or theft has been committed; (2) the
accused, who is not a principal or accomplice in the
commission of the crime of robbery or theft, buys, receives,
possesses, keeps, acquires, conceals, sells or disposes, or
buys and sells, or in any manner deals in any article, item,
object or anything of value, which has been derived from the
proceeds of the crime of robbery or theft; (3) the accused
knew or should have shown that the said article, item, object
or anything of value has been derived from the proceeds of
the crime of robbery or theft; and, (4) there is, on the part of
the accused, intent to gain for himself or for another. The
Supreme Court stated that there was no sufficient proof of
the unlawful taking of anothers property. The theft was not
proved because complainant Rosita Lim did not complain to
the public authorities of the felonious taking of her property.
She sought out her former employee Manuelito Mendez, who
confessed that he stole certain articles from the warehouse
of the complainant and sold them to petitioner. Such
confession is insufficient to convict, without evidence of
corpus delicti.The Court held that accused Tan could not be
held guilty because there was no showing at all that the
accused knew or should have known that the very stolen
articles were the ones sold to him.

You might also like